Oops, Wrong Bottle

Article Type
Changed
Tue, 01/15/2019 - 11:36
Display Headline
Oops, Wrong Bottle

A 42-year-old man presented to the ED with a cut to his left forearm from a piece of metal. The patient only complained of pain at the site of injury; he had no numbness or weakness of the left hand. The patient was otherwise in good health, was taking no medications, and was current with his tetanus immunization.

On physical examination, the patient’s vital signs were normal. The emergency physician (EP) documented a vertical laceration of the mid-left forearm on the dorsal aspect, measuring 6 x 2 cm. The wound edges could be easily approximated. The distal motor and sensory exams were normal.

The EP anesthetized the area with local infiltration using 1% plain xylocaine. The EP then picked up a bottle of CaviCide that had been sitting on the counter and sprayed it on the patient’s wound. The patient immediately complained of burning pain, but the EP continued to spray the wound before suturing it closed with 4.0 nylon.

The patient, however, stated the pain was unbearable. He showed the ED manager the bottle of CaviCide and asked if it was an appropriate sterilizing solution for wounds. When informed it was not, the patient demanded the sutures be removed and the wound re-opened and irrigated with an appropriate solution. The EP re-opened the wound, irrigated it with sterile normal saline, and closed it once again using 4.0 nylon. The EP apologized to the patient, admitted that he made a mistake, and discharged the patient home with instructions to have the sutures removed in 10 days.

The patient developed severe pain at the site a few hours later, prompting him to go to a different ED. They applied lidocaine gel to the area and recommended ibuprofen by mouth for pain. The patient was discharged home.

The patient sued the EP, the nurse, and the hospital for negligence. The plaintiff alleged that under no circumstances should CaviCide be used on humans. The plaintiff’s EM expert testified that the error represented gross negligence. The hospital admitted the nurse violated the standard of care for not properly storing the CaviCide. The EP expert for the defense argued the patient did not suffer any new injury or pain, and that his symptoms were due to the laceration. A second defense expert (toxicology) explained that CaviCide is not toxic and that it would only cause short-term irritation. The plaintiff’s counsel asked for $172,800 in damages, explaining that he was requesting $1 per second for the time the patient experienced intense pain.  After deliberating for five hours, the jury found in favor of the defense.

 

 

DISCUSSION

Over the years, I have seen variations of this case: hemoccult solution placed in the eye under the impression it was a topical anesthetic, and 1:1000 epinephrine given intravenously (IV) when it was thought to be 1:10,000 concentration.

The way to avoid this mistake is to force yourself to take a good look at whatever medication you are administering to a patient, be it by mouth or IV, on the eye or skin, in a muscle, or up the rectum. Read the name of the medication before giving it. It is fortunate for all involved in this case that no serious or permanent injury occurred.

According to the manufacturer of CaviCide (Metrex), it is a “convenient, ready-to-use, intermediate-level surface disinfectant which is effective against tuberculosis, HBV, HCV, viruses (hydrophilic and lipophilic), bacteria (including MRSA and VRE), and fungi. It is safe for use on non-porous surfaces, and for cleaning environmental and medical device surfaces.” While it sounds great for cleaning surfaces and objects, it is clearly not the right product to spray on a wound.

This accident falls under the general heading of a medication error. This category includes: selecting the wrong medication or dosage; giving the medication at the wrong frequency; administration to the wrong patient or via the wrong route; or failure to monitor the patients’ response to the medication. In the risk management world, it is recommended that providers consistently perform the “five rights” of medication administration: right patient; right drug; right dosage; right time; and right route. This case illustrates the problem of “right drug.” Clearly, CaviCide was not the right drug for this patient. Given different circumstances, the harm could have been significant.

SUMMARY

Fortunately, this is a relatively simple take-home message: know what drug you are giving your patient, always.

Article PDF
Author and Disclosure Information

The author reports no actual or potential conflict of interest in relation to this article.

Issue
Emergency Medicine - 50(11)
Publications
Sections
Author and Disclosure Information

The author reports no actual or potential conflict of interest in relation to this article.

Author and Disclosure Information

The author reports no actual or potential conflict of interest in relation to this article.

Article PDF
Article PDF

A 42-year-old man presented to the ED with a cut to his left forearm from a piece of metal. The patient only complained of pain at the site of injury; he had no numbness or weakness of the left hand. The patient was otherwise in good health, was taking no medications, and was current with his tetanus immunization.

On physical examination, the patient’s vital signs were normal. The emergency physician (EP) documented a vertical laceration of the mid-left forearm on the dorsal aspect, measuring 6 x 2 cm. The wound edges could be easily approximated. The distal motor and sensory exams were normal.

The EP anesthetized the area with local infiltration using 1% plain xylocaine. The EP then picked up a bottle of CaviCide that had been sitting on the counter and sprayed it on the patient’s wound. The patient immediately complained of burning pain, but the EP continued to spray the wound before suturing it closed with 4.0 nylon.

The patient, however, stated the pain was unbearable. He showed the ED manager the bottle of CaviCide and asked if it was an appropriate sterilizing solution for wounds. When informed it was not, the patient demanded the sutures be removed and the wound re-opened and irrigated with an appropriate solution. The EP re-opened the wound, irrigated it with sterile normal saline, and closed it once again using 4.0 nylon. The EP apologized to the patient, admitted that he made a mistake, and discharged the patient home with instructions to have the sutures removed in 10 days.

The patient developed severe pain at the site a few hours later, prompting him to go to a different ED. They applied lidocaine gel to the area and recommended ibuprofen by mouth for pain. The patient was discharged home.

The patient sued the EP, the nurse, and the hospital for negligence. The plaintiff alleged that under no circumstances should CaviCide be used on humans. The plaintiff’s EM expert testified that the error represented gross negligence. The hospital admitted the nurse violated the standard of care for not properly storing the CaviCide. The EP expert for the defense argued the patient did not suffer any new injury or pain, and that his symptoms were due to the laceration. A second defense expert (toxicology) explained that CaviCide is not toxic and that it would only cause short-term irritation. The plaintiff’s counsel asked for $172,800 in damages, explaining that he was requesting $1 per second for the time the patient experienced intense pain.  After deliberating for five hours, the jury found in favor of the defense.

 

 

DISCUSSION

Over the years, I have seen variations of this case: hemoccult solution placed in the eye under the impression it was a topical anesthetic, and 1:1000 epinephrine given intravenously (IV) when it was thought to be 1:10,000 concentration.

The way to avoid this mistake is to force yourself to take a good look at whatever medication you are administering to a patient, be it by mouth or IV, on the eye or skin, in a muscle, or up the rectum. Read the name of the medication before giving it. It is fortunate for all involved in this case that no serious or permanent injury occurred.

According to the manufacturer of CaviCide (Metrex), it is a “convenient, ready-to-use, intermediate-level surface disinfectant which is effective against tuberculosis, HBV, HCV, viruses (hydrophilic and lipophilic), bacteria (including MRSA and VRE), and fungi. It is safe for use on non-porous surfaces, and for cleaning environmental and medical device surfaces.” While it sounds great for cleaning surfaces and objects, it is clearly not the right product to spray on a wound.

This accident falls under the general heading of a medication error. This category includes: selecting the wrong medication or dosage; giving the medication at the wrong frequency; administration to the wrong patient or via the wrong route; or failure to monitor the patients’ response to the medication. In the risk management world, it is recommended that providers consistently perform the “five rights” of medication administration: right patient; right drug; right dosage; right time; and right route. This case illustrates the problem of “right drug.” Clearly, CaviCide was not the right drug for this patient. Given different circumstances, the harm could have been significant.

SUMMARY

Fortunately, this is a relatively simple take-home message: know what drug you are giving your patient, always.

A 42-year-old man presented to the ED with a cut to his left forearm from a piece of metal. The patient only complained of pain at the site of injury; he had no numbness or weakness of the left hand. The patient was otherwise in good health, was taking no medications, and was current with his tetanus immunization.

On physical examination, the patient’s vital signs were normal. The emergency physician (EP) documented a vertical laceration of the mid-left forearm on the dorsal aspect, measuring 6 x 2 cm. The wound edges could be easily approximated. The distal motor and sensory exams were normal.

The EP anesthetized the area with local infiltration using 1% plain xylocaine. The EP then picked up a bottle of CaviCide that had been sitting on the counter and sprayed it on the patient’s wound. The patient immediately complained of burning pain, but the EP continued to spray the wound before suturing it closed with 4.0 nylon.

The patient, however, stated the pain was unbearable. He showed the ED manager the bottle of CaviCide and asked if it was an appropriate sterilizing solution for wounds. When informed it was not, the patient demanded the sutures be removed and the wound re-opened and irrigated with an appropriate solution. The EP re-opened the wound, irrigated it with sterile normal saline, and closed it once again using 4.0 nylon. The EP apologized to the patient, admitted that he made a mistake, and discharged the patient home with instructions to have the sutures removed in 10 days.

The patient developed severe pain at the site a few hours later, prompting him to go to a different ED. They applied lidocaine gel to the area and recommended ibuprofen by mouth for pain. The patient was discharged home.

The patient sued the EP, the nurse, and the hospital for negligence. The plaintiff alleged that under no circumstances should CaviCide be used on humans. The plaintiff’s EM expert testified that the error represented gross negligence. The hospital admitted the nurse violated the standard of care for not properly storing the CaviCide. The EP expert for the defense argued the patient did not suffer any new injury or pain, and that his symptoms were due to the laceration. A second defense expert (toxicology) explained that CaviCide is not toxic and that it would only cause short-term irritation. The plaintiff’s counsel asked for $172,800 in damages, explaining that he was requesting $1 per second for the time the patient experienced intense pain.  After deliberating for five hours, the jury found in favor of the defense.

 

 

DISCUSSION

Over the years, I have seen variations of this case: hemoccult solution placed in the eye under the impression it was a topical anesthetic, and 1:1000 epinephrine given intravenously (IV) when it was thought to be 1:10,000 concentration.

The way to avoid this mistake is to force yourself to take a good look at whatever medication you are administering to a patient, be it by mouth or IV, on the eye or skin, in a muscle, or up the rectum. Read the name of the medication before giving it. It is fortunate for all involved in this case that no serious or permanent injury occurred.

According to the manufacturer of CaviCide (Metrex), it is a “convenient, ready-to-use, intermediate-level surface disinfectant which is effective against tuberculosis, HBV, HCV, viruses (hydrophilic and lipophilic), bacteria (including MRSA and VRE), and fungi. It is safe for use on non-porous surfaces, and for cleaning environmental and medical device surfaces.” While it sounds great for cleaning surfaces and objects, it is clearly not the right product to spray on a wound.

This accident falls under the general heading of a medication error. This category includes: selecting the wrong medication or dosage; giving the medication at the wrong frequency; administration to the wrong patient or via the wrong route; or failure to monitor the patients’ response to the medication. In the risk management world, it is recommended that providers consistently perform the “five rights” of medication administration: right patient; right drug; right dosage; right time; and right route. This case illustrates the problem of “right drug.” Clearly, CaviCide was not the right drug for this patient. Given different circumstances, the harm could have been significant.

SUMMARY

Fortunately, this is a relatively simple take-home message: know what drug you are giving your patient, always.

Issue
Emergency Medicine - 50(11)
Issue
Emergency Medicine - 50(11)
Publications
Publications
Article Type
Display Headline
Oops, Wrong Bottle
Display Headline
Oops, Wrong Bottle
Sections
Disallow All Ads
Content Gating
No Gating (article Unlocked/Free)
Alternative CME
Disqus Comments
Default
Gate On Date
Tue, 11/27/2018 - 13:30
Un-Gate On Date
Tue, 11/27/2018 - 13:30
Use ProPublica
CFC Schedule Remove Status
Tue, 11/27/2018 - 13:30
Article PDF Media

Malpractice Counsel: Diverticulitis

Article Type
Changed
Wed, 12/12/2018 - 21:12

A 44-year-old woman presented to the ED complaining of crampy lower abdominal pain with nausea and vomiting. The patient described gradual onset 2 days prior, with symptoms worsening over the previous 12 hours. She denied diarrhea, constipation, or blood in her stool. She did admit to frequency of urination, but no dysuria or hematuria. She was gravida 2, para 2, aborta 0, with a last menstrual period 3 weeks prior. She denied vaginal bleeding or discharge. Her past medical history was unremarkable, she was on no medications, and denied alcohol use. She did admit to smoking one pack of cigarettes per day.

On physical examination, the patient’s vital signs were: blood pressure, 132/68 mm Hg; heart rate, 96 beats/min; respiratory rate, 18 breaths/min; and temperature, 99.8°F. Oxygen saturation was 99% on room air.

The head, ears, eyes, nose, and throat (HEENT) examination was completely normal, as was the heart and lung examination. The patient was tender to palpation in the lower abdomen, but without guarding or rebound. Bowel sounds were present and normoactive. A pelvic examination, including a bimanual examination, demonstrated mild left ovarian tenderness but without mass or cervical motion tenderness. The patient did not exhibit any costovertebral angle tenderness bilaterally. No rectal examination was performed.

The emergency physicians (EPs) ordered a complete blood count (CBC), basic metabolic profile (BMP), urinalysis, urine pregnancy test, and a vaginal wet preparation. In addition, the patient was administered 500 cc’s of normal saline intravenously (IV) and ondansetron (Zofran) 4 mg IV.

The urine pregnancy test result came back negative. The urinalysis was remarkable for positive leukocyte esterase, with five to 10 white cells and bacteria present. The CBC showed a mild leukocytosis, but with a normal hemoglobin and hematocrit. The BMP and vaginal wet preparation were completely normal.

The EP was concerned the patient might have something more serious than a simple urinary tract infection (UTI), so she ordered a computed tomography (CT) scan of the abdomen and pelvis with IV contrast.

The radiologist interpreted the CT scan as normal. The patient was discharged home with a prescription for an antibiotic for her UTI, encouraged to drink liquids, and instructed to follow-up with her primary care physician in 1 week.

The patient returned to the same ED approximately 48 hours later with worsening abdominal pain. On this presentation, she was tachycardic (110 beats/min) with a temperature of 101°F. The abdominal examination was remarkable for diffuse tenderness and voluntary guarding. The patient was administered IV fluids, morphine, and ondansetron. A repeat CT scan of the abdomen and pelvis with IV contrast showed a perforated sigmoid colon, with leakage of bowel contents into the peritoneum. The EP immediately started IV fluid resuscitation and administered IV antibiotics. The patient was taken emergently to the operating room by general surgery. The colon was repaired and a colostomy placed. The patient was able to be discharged home on day number 5.

The patient sued the hospital and the treating EP for failure to make the proper diagnosis on the initial ED visit, resulting in the patient having a long and difficult recovery, and the need for a colostomy. At trial, the jury returned a defense verdict.

 

 

Discussion

Diverticulitis, and its complications, account for a significant number of ED visits. It is the third most common inpatient gastrointestinal diagnosis in the United States, costing two billion dollars annually.1It is defined as clinically evident microscopic inflammation of a diverticulum or diverticula, and occurs in approximately 4% of patients with diverticulosis.1It is estimated that roughly 15% of these patients will experience a complication, defined as an abscess, perforation, fistula, or colonic obstruction; 15% to 30% will experience a recurrence.

The mean age of patients admitted to the hospital for diverticulitis is 63 years. While considered a disease of older patients, it should be included in the differential diagnosis for younger patients, as approximately 16% of admissions for acute diverticulitis are in patients less than 45 years.2Risk factors include poor diet (ie, low fiber, high fat, red meat), obesity, and smoking. The clinical presentation of diverticulitis has sometimes been referred to as “left-sided appendicitis” because of the similarities between the two entities. Patients will frequently complain of anorexia, change in bowel habits (either diarrhea or constipation), crampy abdominal pain (primarily in the left lower quadrant), low grade fever, and nausea with vomiting. Interestingly, 10% to 15% of patients with acute diverticulitis will complain of dysuria, urgency, or frequency (as in this patient) due to irritation of the bladder from an inflamed sigmoid colon.

Physical examination may reveal a low grade fever and tachycardia, if significant vomiting has been present. The abdomen is tender primarily in the left lower quadrant. The presence of severe tachycardia, hypotension, or a rigid abdomen with guarding and rebound suggests perforation. A pelvic examination should be performed on all women of child-bearing age. The rectal examination may reveal hemoccult positive stool; gross blood is rare.

Laboratory testing should include a CBC, BMP, urinalysis, and a urine pregnancy test (for women of child-bearing age). The CBC will usually reveal a mild leukocytosis. The urinalysis may reveal sterile pyuria for the reason previously described. Additional testing may be indicated by the history and physical examination.

A CT scan of the abdomen and pelvis is considered the gold standard with regards to imaging, with a reported sensitivity of 94% and specificity of 99%.3 Ideally, the CT scan should include both oral and IV contrast; however, IV alone is frequently used. In addition to identifying diverticulitis, CT can also visualize complications, including abscesses, perforation, and bowel obstruction. Ultrasound using high-resolution, graded compassion has a similar sensitivity and specificity as CT, with the advantage of less cost, can be performed at the bedside, and avoids radiation exposure.3 However, it is operator dependent and inferior to CT regarding visualizing complications.

Historically, antibiotics have been considered the treatment of choice for patients with acute uncomplicated diverticulitis, usually as an outpatient. Typically, this involves prescribing ciprofloxacin (or trimethoprim-sulfamethoxazole) plus metronidazole for 7 to 10 days. Monotherapy consisting of either moxifloxacin or amoxicillin/clavulanic acid is also acceptable. However, as our understanding of the important role of inflammation in this disease process, combined with the negative effects associated with antibiotic use, the role of antibiotics in uncomplicated diverticulitis has been called into question. In one recent study of 155 patients with acute uncomplicated diverticulitis, 97% were managed successfully as outpatients without antibiotics, admission, or complications.4 The American Gastroenterological Association (AGA) recommends that antibiotics should be used selectively, rather than routinely, in patients with acute uncomplicated diverticulitis. However, this is considered a “conditional recommendation with a low quality of evidence.”1In other words, this recommendation could easily change based on newer studies. Similarly, other “conditional recommendations” by the AGA include suggesting a fiber-rich diet, or fiber supplementation, and no need to avoid the consumption of nuts and popcorn.1 The majority of these patients begin to feel better in 2 to 3 days and have a good outcome.

Summary

For patients that appear ill, have significant comorbidities, are immunocompromised, or have a complication of acute diverticulitis, admission to the hospital with surgery consultation is recommended. For abscesses, interventional radiology has been used with success for CT-guided percutaneous drainage of diverticular abscesses. Intravenous antibiotics should be initiated; appropriate medications include metronidazole plus a third-generation cephalosporin (such as ceftriaxone or cefotaxime) or a fluoroquinolone (such as ciprofloxacin or levofloxacin). Monotherapy for the moderately ill patient includes piperacillin/tazobactam, ampicillin/sulbactam, ticarcillin/clavulanic acid, and imipenem. In addition, these patients should be placed at bowel rest (ie, nothing by mouth) with IV fluid resuscitation and hydration.

References

1. Stollman N, Smalley W, Hirano I; AGA Institute Clinical Guidelines Committee. American Gastroenterological Association Institute guideline on the management of acute diverticulitis. Gastroenterology. 2015;149(7):1944-1949. doi:10.1053/j.gastro.2015.10.003.

2. Nguyen GC, Sam J, Anand N. Epidemiological trends and geographic variation in hospital admissions for diverticulitis in the United States. World J Gastroenterol. 2011;17(12):1600-1605. doi:10.3748/wjg.v17.i12.1600.

3. Laméris W, van Randen A, Bipat S, Bossuyt PM, Boermeester MA, Stoker J. Graded compression ultrasonography and computed tomography in acute colonic diverticulitis: meta-analysis of test accuracy. Eur Radiol. 2008;18(11):2498-2511. doi:10.1007/s00330-008-1018-6.

4. Isacson D, Thorisson A, Andreasson K, Nikberg M, Smedh K, Chabok A. Outpatient, non-antibiotic management in acute uncomplicated diverticulitis: a prospective study. Int J Colorectal Dis. 2015;30(9):1229-1234. doi:10.1007/s00384-015-2258-y.

Article PDF
Author and Disclosure Information

Author’s Disclosure Statement: The author reports no actual or potential conflict of interest in relation to this article.

Issue
Emergency Medicine - 50(9)
Publications
Topics
Page Number
172-174
Sections
Author and Disclosure Information

Author’s Disclosure Statement: The author reports no actual or potential conflict of interest in relation to this article.

Author and Disclosure Information

Author’s Disclosure Statement: The author reports no actual or potential conflict of interest in relation to this article.

Article PDF
Article PDF

A 44-year-old woman presented to the ED complaining of crampy lower abdominal pain with nausea and vomiting. The patient described gradual onset 2 days prior, with symptoms worsening over the previous 12 hours. She denied diarrhea, constipation, or blood in her stool. She did admit to frequency of urination, but no dysuria or hematuria. She was gravida 2, para 2, aborta 0, with a last menstrual period 3 weeks prior. She denied vaginal bleeding or discharge. Her past medical history was unremarkable, she was on no medications, and denied alcohol use. She did admit to smoking one pack of cigarettes per day.

On physical examination, the patient’s vital signs were: blood pressure, 132/68 mm Hg; heart rate, 96 beats/min; respiratory rate, 18 breaths/min; and temperature, 99.8°F. Oxygen saturation was 99% on room air.

The head, ears, eyes, nose, and throat (HEENT) examination was completely normal, as was the heart and lung examination. The patient was tender to palpation in the lower abdomen, but without guarding or rebound. Bowel sounds were present and normoactive. A pelvic examination, including a bimanual examination, demonstrated mild left ovarian tenderness but without mass or cervical motion tenderness. The patient did not exhibit any costovertebral angle tenderness bilaterally. No rectal examination was performed.

The emergency physicians (EPs) ordered a complete blood count (CBC), basic metabolic profile (BMP), urinalysis, urine pregnancy test, and a vaginal wet preparation. In addition, the patient was administered 500 cc’s of normal saline intravenously (IV) and ondansetron (Zofran) 4 mg IV.

The urine pregnancy test result came back negative. The urinalysis was remarkable for positive leukocyte esterase, with five to 10 white cells and bacteria present. The CBC showed a mild leukocytosis, but with a normal hemoglobin and hematocrit. The BMP and vaginal wet preparation were completely normal.

The EP was concerned the patient might have something more serious than a simple urinary tract infection (UTI), so she ordered a computed tomography (CT) scan of the abdomen and pelvis with IV contrast.

The radiologist interpreted the CT scan as normal. The patient was discharged home with a prescription for an antibiotic for her UTI, encouraged to drink liquids, and instructed to follow-up with her primary care physician in 1 week.

The patient returned to the same ED approximately 48 hours later with worsening abdominal pain. On this presentation, she was tachycardic (110 beats/min) with a temperature of 101°F. The abdominal examination was remarkable for diffuse tenderness and voluntary guarding. The patient was administered IV fluids, morphine, and ondansetron. A repeat CT scan of the abdomen and pelvis with IV contrast showed a perforated sigmoid colon, with leakage of bowel contents into the peritoneum. The EP immediately started IV fluid resuscitation and administered IV antibiotics. The patient was taken emergently to the operating room by general surgery. The colon was repaired and a colostomy placed. The patient was able to be discharged home on day number 5.

The patient sued the hospital and the treating EP for failure to make the proper diagnosis on the initial ED visit, resulting in the patient having a long and difficult recovery, and the need for a colostomy. At trial, the jury returned a defense verdict.

 

 

Discussion

Diverticulitis, and its complications, account for a significant number of ED visits. It is the third most common inpatient gastrointestinal diagnosis in the United States, costing two billion dollars annually.1It is defined as clinically evident microscopic inflammation of a diverticulum or diverticula, and occurs in approximately 4% of patients with diverticulosis.1It is estimated that roughly 15% of these patients will experience a complication, defined as an abscess, perforation, fistula, or colonic obstruction; 15% to 30% will experience a recurrence.

The mean age of patients admitted to the hospital for diverticulitis is 63 years. While considered a disease of older patients, it should be included in the differential diagnosis for younger patients, as approximately 16% of admissions for acute diverticulitis are in patients less than 45 years.2Risk factors include poor diet (ie, low fiber, high fat, red meat), obesity, and smoking. The clinical presentation of diverticulitis has sometimes been referred to as “left-sided appendicitis” because of the similarities between the two entities. Patients will frequently complain of anorexia, change in bowel habits (either diarrhea or constipation), crampy abdominal pain (primarily in the left lower quadrant), low grade fever, and nausea with vomiting. Interestingly, 10% to 15% of patients with acute diverticulitis will complain of dysuria, urgency, or frequency (as in this patient) due to irritation of the bladder from an inflamed sigmoid colon.

Physical examination may reveal a low grade fever and tachycardia, if significant vomiting has been present. The abdomen is tender primarily in the left lower quadrant. The presence of severe tachycardia, hypotension, or a rigid abdomen with guarding and rebound suggests perforation. A pelvic examination should be performed on all women of child-bearing age. The rectal examination may reveal hemoccult positive stool; gross blood is rare.

Laboratory testing should include a CBC, BMP, urinalysis, and a urine pregnancy test (for women of child-bearing age). The CBC will usually reveal a mild leukocytosis. The urinalysis may reveal sterile pyuria for the reason previously described. Additional testing may be indicated by the history and physical examination.

A CT scan of the abdomen and pelvis is considered the gold standard with regards to imaging, with a reported sensitivity of 94% and specificity of 99%.3 Ideally, the CT scan should include both oral and IV contrast; however, IV alone is frequently used. In addition to identifying diverticulitis, CT can also visualize complications, including abscesses, perforation, and bowel obstruction. Ultrasound using high-resolution, graded compassion has a similar sensitivity and specificity as CT, with the advantage of less cost, can be performed at the bedside, and avoids radiation exposure.3 However, it is operator dependent and inferior to CT regarding visualizing complications.

Historically, antibiotics have been considered the treatment of choice for patients with acute uncomplicated diverticulitis, usually as an outpatient. Typically, this involves prescribing ciprofloxacin (or trimethoprim-sulfamethoxazole) plus metronidazole for 7 to 10 days. Monotherapy consisting of either moxifloxacin or amoxicillin/clavulanic acid is also acceptable. However, as our understanding of the important role of inflammation in this disease process, combined with the negative effects associated with antibiotic use, the role of antibiotics in uncomplicated diverticulitis has been called into question. In one recent study of 155 patients with acute uncomplicated diverticulitis, 97% were managed successfully as outpatients without antibiotics, admission, or complications.4 The American Gastroenterological Association (AGA) recommends that antibiotics should be used selectively, rather than routinely, in patients with acute uncomplicated diverticulitis. However, this is considered a “conditional recommendation with a low quality of evidence.”1In other words, this recommendation could easily change based on newer studies. Similarly, other “conditional recommendations” by the AGA include suggesting a fiber-rich diet, or fiber supplementation, and no need to avoid the consumption of nuts and popcorn.1 The majority of these patients begin to feel better in 2 to 3 days and have a good outcome.

Summary

For patients that appear ill, have significant comorbidities, are immunocompromised, or have a complication of acute diverticulitis, admission to the hospital with surgery consultation is recommended. For abscesses, interventional radiology has been used with success for CT-guided percutaneous drainage of diverticular abscesses. Intravenous antibiotics should be initiated; appropriate medications include metronidazole plus a third-generation cephalosporin (such as ceftriaxone or cefotaxime) or a fluoroquinolone (such as ciprofloxacin or levofloxacin). Monotherapy for the moderately ill patient includes piperacillin/tazobactam, ampicillin/sulbactam, ticarcillin/clavulanic acid, and imipenem. In addition, these patients should be placed at bowel rest (ie, nothing by mouth) with IV fluid resuscitation and hydration.

A 44-year-old woman presented to the ED complaining of crampy lower abdominal pain with nausea and vomiting. The patient described gradual onset 2 days prior, with symptoms worsening over the previous 12 hours. She denied diarrhea, constipation, or blood in her stool. She did admit to frequency of urination, but no dysuria or hematuria. She was gravida 2, para 2, aborta 0, with a last menstrual period 3 weeks prior. She denied vaginal bleeding or discharge. Her past medical history was unremarkable, she was on no medications, and denied alcohol use. She did admit to smoking one pack of cigarettes per day.

On physical examination, the patient’s vital signs were: blood pressure, 132/68 mm Hg; heart rate, 96 beats/min; respiratory rate, 18 breaths/min; and temperature, 99.8°F. Oxygen saturation was 99% on room air.

The head, ears, eyes, nose, and throat (HEENT) examination was completely normal, as was the heart and lung examination. The patient was tender to palpation in the lower abdomen, but without guarding or rebound. Bowel sounds were present and normoactive. A pelvic examination, including a bimanual examination, demonstrated mild left ovarian tenderness but without mass or cervical motion tenderness. The patient did not exhibit any costovertebral angle tenderness bilaterally. No rectal examination was performed.

The emergency physicians (EPs) ordered a complete blood count (CBC), basic metabolic profile (BMP), urinalysis, urine pregnancy test, and a vaginal wet preparation. In addition, the patient was administered 500 cc’s of normal saline intravenously (IV) and ondansetron (Zofran) 4 mg IV.

The urine pregnancy test result came back negative. The urinalysis was remarkable for positive leukocyte esterase, with five to 10 white cells and bacteria present. The CBC showed a mild leukocytosis, but with a normal hemoglobin and hematocrit. The BMP and vaginal wet preparation were completely normal.

The EP was concerned the patient might have something more serious than a simple urinary tract infection (UTI), so she ordered a computed tomography (CT) scan of the abdomen and pelvis with IV contrast.

The radiologist interpreted the CT scan as normal. The patient was discharged home with a prescription for an antibiotic for her UTI, encouraged to drink liquids, and instructed to follow-up with her primary care physician in 1 week.

The patient returned to the same ED approximately 48 hours later with worsening abdominal pain. On this presentation, she was tachycardic (110 beats/min) with a temperature of 101°F. The abdominal examination was remarkable for diffuse tenderness and voluntary guarding. The patient was administered IV fluids, morphine, and ondansetron. A repeat CT scan of the abdomen and pelvis with IV contrast showed a perforated sigmoid colon, with leakage of bowel contents into the peritoneum. The EP immediately started IV fluid resuscitation and administered IV antibiotics. The patient was taken emergently to the operating room by general surgery. The colon was repaired and a colostomy placed. The patient was able to be discharged home on day number 5.

The patient sued the hospital and the treating EP for failure to make the proper diagnosis on the initial ED visit, resulting in the patient having a long and difficult recovery, and the need for a colostomy. At trial, the jury returned a defense verdict.

 

 

Discussion

Diverticulitis, and its complications, account for a significant number of ED visits. It is the third most common inpatient gastrointestinal diagnosis in the United States, costing two billion dollars annually.1It is defined as clinically evident microscopic inflammation of a diverticulum or diverticula, and occurs in approximately 4% of patients with diverticulosis.1It is estimated that roughly 15% of these patients will experience a complication, defined as an abscess, perforation, fistula, or colonic obstruction; 15% to 30% will experience a recurrence.

The mean age of patients admitted to the hospital for diverticulitis is 63 years. While considered a disease of older patients, it should be included in the differential diagnosis for younger patients, as approximately 16% of admissions for acute diverticulitis are in patients less than 45 years.2Risk factors include poor diet (ie, low fiber, high fat, red meat), obesity, and smoking. The clinical presentation of diverticulitis has sometimes been referred to as “left-sided appendicitis” because of the similarities between the two entities. Patients will frequently complain of anorexia, change in bowel habits (either diarrhea or constipation), crampy abdominal pain (primarily in the left lower quadrant), low grade fever, and nausea with vomiting. Interestingly, 10% to 15% of patients with acute diverticulitis will complain of dysuria, urgency, or frequency (as in this patient) due to irritation of the bladder from an inflamed sigmoid colon.

Physical examination may reveal a low grade fever and tachycardia, if significant vomiting has been present. The abdomen is tender primarily in the left lower quadrant. The presence of severe tachycardia, hypotension, or a rigid abdomen with guarding and rebound suggests perforation. A pelvic examination should be performed on all women of child-bearing age. The rectal examination may reveal hemoccult positive stool; gross blood is rare.

Laboratory testing should include a CBC, BMP, urinalysis, and a urine pregnancy test (for women of child-bearing age). The CBC will usually reveal a mild leukocytosis. The urinalysis may reveal sterile pyuria for the reason previously described. Additional testing may be indicated by the history and physical examination.

A CT scan of the abdomen and pelvis is considered the gold standard with regards to imaging, with a reported sensitivity of 94% and specificity of 99%.3 Ideally, the CT scan should include both oral and IV contrast; however, IV alone is frequently used. In addition to identifying diverticulitis, CT can also visualize complications, including abscesses, perforation, and bowel obstruction. Ultrasound using high-resolution, graded compassion has a similar sensitivity and specificity as CT, with the advantage of less cost, can be performed at the bedside, and avoids radiation exposure.3 However, it is operator dependent and inferior to CT regarding visualizing complications.

Historically, antibiotics have been considered the treatment of choice for patients with acute uncomplicated diverticulitis, usually as an outpatient. Typically, this involves prescribing ciprofloxacin (or trimethoprim-sulfamethoxazole) plus metronidazole for 7 to 10 days. Monotherapy consisting of either moxifloxacin or amoxicillin/clavulanic acid is also acceptable. However, as our understanding of the important role of inflammation in this disease process, combined with the negative effects associated with antibiotic use, the role of antibiotics in uncomplicated diverticulitis has been called into question. In one recent study of 155 patients with acute uncomplicated diverticulitis, 97% were managed successfully as outpatients without antibiotics, admission, or complications.4 The American Gastroenterological Association (AGA) recommends that antibiotics should be used selectively, rather than routinely, in patients with acute uncomplicated diverticulitis. However, this is considered a “conditional recommendation with a low quality of evidence.”1In other words, this recommendation could easily change based on newer studies. Similarly, other “conditional recommendations” by the AGA include suggesting a fiber-rich diet, or fiber supplementation, and no need to avoid the consumption of nuts and popcorn.1 The majority of these patients begin to feel better in 2 to 3 days and have a good outcome.

Summary

For patients that appear ill, have significant comorbidities, are immunocompromised, or have a complication of acute diverticulitis, admission to the hospital with surgery consultation is recommended. For abscesses, interventional radiology has been used with success for CT-guided percutaneous drainage of diverticular abscesses. Intravenous antibiotics should be initiated; appropriate medications include metronidazole plus a third-generation cephalosporin (such as ceftriaxone or cefotaxime) or a fluoroquinolone (such as ciprofloxacin or levofloxacin). Monotherapy for the moderately ill patient includes piperacillin/tazobactam, ampicillin/sulbactam, ticarcillin/clavulanic acid, and imipenem. In addition, these patients should be placed at bowel rest (ie, nothing by mouth) with IV fluid resuscitation and hydration.

References

1. Stollman N, Smalley W, Hirano I; AGA Institute Clinical Guidelines Committee. American Gastroenterological Association Institute guideline on the management of acute diverticulitis. Gastroenterology. 2015;149(7):1944-1949. doi:10.1053/j.gastro.2015.10.003.

2. Nguyen GC, Sam J, Anand N. Epidemiological trends and geographic variation in hospital admissions for diverticulitis in the United States. World J Gastroenterol. 2011;17(12):1600-1605. doi:10.3748/wjg.v17.i12.1600.

3. Laméris W, van Randen A, Bipat S, Bossuyt PM, Boermeester MA, Stoker J. Graded compression ultrasonography and computed tomography in acute colonic diverticulitis: meta-analysis of test accuracy. Eur Radiol. 2008;18(11):2498-2511. doi:10.1007/s00330-008-1018-6.

4. Isacson D, Thorisson A, Andreasson K, Nikberg M, Smedh K, Chabok A. Outpatient, non-antibiotic management in acute uncomplicated diverticulitis: a prospective study. Int J Colorectal Dis. 2015;30(9):1229-1234. doi:10.1007/s00384-015-2258-y.

References

1. Stollman N, Smalley W, Hirano I; AGA Institute Clinical Guidelines Committee. American Gastroenterological Association Institute guideline on the management of acute diverticulitis. Gastroenterology. 2015;149(7):1944-1949. doi:10.1053/j.gastro.2015.10.003.

2. Nguyen GC, Sam J, Anand N. Epidemiological trends and geographic variation in hospital admissions for diverticulitis in the United States. World J Gastroenterol. 2011;17(12):1600-1605. doi:10.3748/wjg.v17.i12.1600.

3. Laméris W, van Randen A, Bipat S, Bossuyt PM, Boermeester MA, Stoker J. Graded compression ultrasonography and computed tomography in acute colonic diverticulitis: meta-analysis of test accuracy. Eur Radiol. 2008;18(11):2498-2511. doi:10.1007/s00330-008-1018-6.

4. Isacson D, Thorisson A, Andreasson K, Nikberg M, Smedh K, Chabok A. Outpatient, non-antibiotic management in acute uncomplicated diverticulitis: a prospective study. Int J Colorectal Dis. 2015;30(9):1229-1234. doi:10.1007/s00384-015-2258-y.

Issue
Emergency Medicine - 50(9)
Issue
Emergency Medicine - 50(9)
Page Number
172-174
Page Number
172-174
Publications
Publications
Topics
Article Type
Sections
Disallow All Ads
Content Gating
No Gating (article Unlocked/Free)
Alternative CME
Disqus Comments
Default
Use ProPublica
Article PDF Media

The times they are a-changin’ —Bob Dylan, 1964

Article Type
Changed
Thu, 03/28/2019 - 14:42
Display Headline
The times they are a-changin’ —Bob Dylan, 1964

The beginning of a new year is always associated with changes, accompanied by new challenges and opportunities. This year is no different and, in fact, begins with some significant changes. First, I am incredibly honored, and humbled, to be named your new editor-in-chief. By way of background and introduction, I am residency-trained and board-certified in emergency medicine (EM). I founded the first academic department of EM in Virginia in 1992, and continue to serve in the role of chair. From 1990 to 2010, I served as the program director of our 3-year EM residency program, which I still consider the best job in EM. Most importantly, I continue to see and care for patients in the ED primarily, in addition to supervising and teaching EM residents and fellows in the delivery of care in the clinical arena. I know first-hand the needs of practicing emergency physicians (EPs).

I feel very fortunate to have been associated with Emergency Medicine (EM) since 1988, the year the journal published my very first manuscript. I served on the editorial board from 1999 to 2006, and for the past 11 years, have served as the associate editor-in-chief. I hold a very special regard and respect for this journal, and its role in our specialty. My goal is to continue to publish high-quality content and ensure we consistently provide timely and clinically useful information to the practicing EP. We will invite the very best in our specialty to share their knowledge and clinical tips. We will of course continue some of your favorite sections, like “Emergency Ultrasound,” “Diagnosis at a Glance,” and “Case Studies in Toxicology.” We will also encourage our readers to submit interesting and informative case reports, review articles, and interesting images. While I plan to write a few editorials each year, I will invite thought leaders in EM to write on their area(s) of expertise.

Come writers and critics who prophesize with your pen.

Another major change has to do with the journal itself. This will be the last paper copy of EM (so think about keeping this one for posterity, or eBay). Starting with the February issue, all future issues will be digital and online-only. This decision was not an easy one, and has been in the making for some time. Thanks to the growth in our Web site traffic, it is clear that many of you have already become “digital-first” readers. This fact, combined with the added financial challenge of publishing a large-circulation journal within an environment of declining print advertising, convinced us that this is the right time to make the leap to the digital-only format. While some of you (including myself), will miss physically holding and reading a hard copy of EM, you may simply continue to access the journal as you have for years, on your desktop, laptop, or iPad, and never further away than your cell phone. This change has the advantage of providing opportunities to deliver valuable clinical content in new ways, through increased use of audio and video, as well as text. To ensure that you receive your copy, please e-mail our Editor, Kellie DeSantis (kdesantis@frontlinemedcom.com) to make sure we have your correct and preferred e-mail address. While our goal is to push each issue out to you via e-mail, you will always be able to access the most recent articles by going to our Web site, www.emed-journal.com.

And don’t criticize

What you can’t understand.

Finally, 2018 promises to be a very interesting year, with the unknown implications of tax reform, the repeal of the individual mandate for health insurance, the opioid crisis, and the curious mergers within the health insurance industry (ie, CVS and Aetna). It is too soon for anyone to say how these changes will affect EM on the national stage. What will not change however, is that EPs will continue to provide outstanding care to any and every patient who presents to the ED. Emergency physicians will ensure that all patients receive the care they need (but not necessarily the care they want) and will do so without regard to gender, religion, national origin, race, age, sexual preference, or insurance status.

I wish each and every one of you a happy and healthy 2018.

Article PDF
Author and Disclosure Information

Author’s Disclosure Statement: The author reports no actual or potential conflict of interest in relation to this article.

Issue
Emergency Medicine - 50(1)
Publications
Topics
Page Number
5
Sections
Author and Disclosure Information

Author’s Disclosure Statement: The author reports no actual or potential conflict of interest in relation to this article.

Author and Disclosure Information

Author’s Disclosure Statement: The author reports no actual or potential conflict of interest in relation to this article.

Article PDF
Article PDF

The beginning of a new year is always associated with changes, accompanied by new challenges and opportunities. This year is no different and, in fact, begins with some significant changes. First, I am incredibly honored, and humbled, to be named your new editor-in-chief. By way of background and introduction, I am residency-trained and board-certified in emergency medicine (EM). I founded the first academic department of EM in Virginia in 1992, and continue to serve in the role of chair. From 1990 to 2010, I served as the program director of our 3-year EM residency program, which I still consider the best job in EM. Most importantly, I continue to see and care for patients in the ED primarily, in addition to supervising and teaching EM residents and fellows in the delivery of care in the clinical arena. I know first-hand the needs of practicing emergency physicians (EPs).

I feel very fortunate to have been associated with Emergency Medicine (EM) since 1988, the year the journal published my very first manuscript. I served on the editorial board from 1999 to 2006, and for the past 11 years, have served as the associate editor-in-chief. I hold a very special regard and respect for this journal, and its role in our specialty. My goal is to continue to publish high-quality content and ensure we consistently provide timely and clinically useful information to the practicing EP. We will invite the very best in our specialty to share their knowledge and clinical tips. We will of course continue some of your favorite sections, like “Emergency Ultrasound,” “Diagnosis at a Glance,” and “Case Studies in Toxicology.” We will also encourage our readers to submit interesting and informative case reports, review articles, and interesting images. While I plan to write a few editorials each year, I will invite thought leaders in EM to write on their area(s) of expertise.

Come writers and critics who prophesize with your pen.

Another major change has to do with the journal itself. This will be the last paper copy of EM (so think about keeping this one for posterity, or eBay). Starting with the February issue, all future issues will be digital and online-only. This decision was not an easy one, and has been in the making for some time. Thanks to the growth in our Web site traffic, it is clear that many of you have already become “digital-first” readers. This fact, combined with the added financial challenge of publishing a large-circulation journal within an environment of declining print advertising, convinced us that this is the right time to make the leap to the digital-only format. While some of you (including myself), will miss physically holding and reading a hard copy of EM, you may simply continue to access the journal as you have for years, on your desktop, laptop, or iPad, and never further away than your cell phone. This change has the advantage of providing opportunities to deliver valuable clinical content in new ways, through increased use of audio and video, as well as text. To ensure that you receive your copy, please e-mail our Editor, Kellie DeSantis (kdesantis@frontlinemedcom.com) to make sure we have your correct and preferred e-mail address. While our goal is to push each issue out to you via e-mail, you will always be able to access the most recent articles by going to our Web site, www.emed-journal.com.

And don’t criticize

What you can’t understand.

Finally, 2018 promises to be a very interesting year, with the unknown implications of tax reform, the repeal of the individual mandate for health insurance, the opioid crisis, and the curious mergers within the health insurance industry (ie, CVS and Aetna). It is too soon for anyone to say how these changes will affect EM on the national stage. What will not change however, is that EPs will continue to provide outstanding care to any and every patient who presents to the ED. Emergency physicians will ensure that all patients receive the care they need (but not necessarily the care they want) and will do so without regard to gender, religion, national origin, race, age, sexual preference, or insurance status.

I wish each and every one of you a happy and healthy 2018.

The beginning of a new year is always associated with changes, accompanied by new challenges and opportunities. This year is no different and, in fact, begins with some significant changes. First, I am incredibly honored, and humbled, to be named your new editor-in-chief. By way of background and introduction, I am residency-trained and board-certified in emergency medicine (EM). I founded the first academic department of EM in Virginia in 1992, and continue to serve in the role of chair. From 1990 to 2010, I served as the program director of our 3-year EM residency program, which I still consider the best job in EM. Most importantly, I continue to see and care for patients in the ED primarily, in addition to supervising and teaching EM residents and fellows in the delivery of care in the clinical arena. I know first-hand the needs of practicing emergency physicians (EPs).

I feel very fortunate to have been associated with Emergency Medicine (EM) since 1988, the year the journal published my very first manuscript. I served on the editorial board from 1999 to 2006, and for the past 11 years, have served as the associate editor-in-chief. I hold a very special regard and respect for this journal, and its role in our specialty. My goal is to continue to publish high-quality content and ensure we consistently provide timely and clinically useful information to the practicing EP. We will invite the very best in our specialty to share their knowledge and clinical tips. We will of course continue some of your favorite sections, like “Emergency Ultrasound,” “Diagnosis at a Glance,” and “Case Studies in Toxicology.” We will also encourage our readers to submit interesting and informative case reports, review articles, and interesting images. While I plan to write a few editorials each year, I will invite thought leaders in EM to write on their area(s) of expertise.

Come writers and critics who prophesize with your pen.

Another major change has to do with the journal itself. This will be the last paper copy of EM (so think about keeping this one for posterity, or eBay). Starting with the February issue, all future issues will be digital and online-only. This decision was not an easy one, and has been in the making for some time. Thanks to the growth in our Web site traffic, it is clear that many of you have already become “digital-first” readers. This fact, combined with the added financial challenge of publishing a large-circulation journal within an environment of declining print advertising, convinced us that this is the right time to make the leap to the digital-only format. While some of you (including myself), will miss physically holding and reading a hard copy of EM, you may simply continue to access the journal as you have for years, on your desktop, laptop, or iPad, and never further away than your cell phone. This change has the advantage of providing opportunities to deliver valuable clinical content in new ways, through increased use of audio and video, as well as text. To ensure that you receive your copy, please e-mail our Editor, Kellie DeSantis (kdesantis@frontlinemedcom.com) to make sure we have your correct and preferred e-mail address. While our goal is to push each issue out to you via e-mail, you will always be able to access the most recent articles by going to our Web site, www.emed-journal.com.

And don’t criticize

What you can’t understand.

Finally, 2018 promises to be a very interesting year, with the unknown implications of tax reform, the repeal of the individual mandate for health insurance, the opioid crisis, and the curious mergers within the health insurance industry (ie, CVS and Aetna). It is too soon for anyone to say how these changes will affect EM on the national stage. What will not change however, is that EPs will continue to provide outstanding care to any and every patient who presents to the ED. Emergency physicians will ensure that all patients receive the care they need (but not necessarily the care they want) and will do so without regard to gender, religion, national origin, race, age, sexual preference, or insurance status.

I wish each and every one of you a happy and healthy 2018.

Issue
Emergency Medicine - 50(1)
Issue
Emergency Medicine - 50(1)
Page Number
5
Page Number
5
Publications
Publications
Topics
Article Type
Display Headline
The times they are a-changin’ —Bob Dylan, 1964
Display Headline
The times they are a-changin’ —Bob Dylan, 1964
Sections
Disallow All Ads
Content Gating
No Gating (article Unlocked/Free)
Alternative CME
Disqus Comments
Default
Article PDF Media

Malpractice Counsel: Don’t Miss Popeye

Article Type
Changed
Wed, 12/12/2018 - 21:08

A 42-year-old man presented to the ED with left arm pain secondary to an injury he sustained at work. The patient stated that he had been helping to lift a heavy steel beam at a construction site when he experienced abrupt onset of pain in his left arm. He further noted that his left arm felt slightly weaker than normal after the injury.

The patient was left-hand dominant, denied any other injury, was otherwise in good health, and on no medications. With the exception of an appendectomy at age 12 years, his medical history was unremarkable. Regarding his social history, he admitted to smoking one pack of cigarettes per day, and to occasional alcohol consumption. He had no known drug allergies.

On physical examination, the patient’s vital signs were: blood pressure, 125/76 mm Hg; heart rate, 78 beats/min; respiratory rate, 16 breaths/min; and temperature, 98.6°F. Oxygen saturation was 99% on room air.

Examination of the patient’s left shoulder revealed no swelling or tenderness; he was able to fully internally/externally rotate the left shoulder, and lift his left hand above his head. The patient did have tenderness along the biceps area of the left arm, but no tenderness in the triceps area. The left elbow was tender in the antecubital fossa, but without swelling. He had full range of motion of the left elbow but with some pain. He likewise had full range of motion in his left wrist, but no tenderness or swelling. The left radial pulse was 2+. The patient had 5/5 grip strength with the left hand and good capillary refill.

The physician assistant (PA) evaluating the patient diagnosed an arm strain. At discharge, he referred the patient to an occupational health physician (OHP) for follow-up. He also instructed the patient to take ibuprofen 400 mg every 6 to 8 hours, and to limit use of his left arm for 3 days.

The patient followed up with the OHP approximately 3 weeks after discharge from the ED. The OHP was concerned the patient had experienced a distal biceps tendon rupture and referred the patient emergently to an orthopedic surgeon. The orthopedic surgeon saw the patient the next day, agreed with the diagnosis of a distal biceps tendon rupture, and attempted surgical repair the following day. The orthopedic surgeon informed the patient prior to the surgery that the delay in the referral and surgery could result in a poor functional outcome. The patient did have a difficult recovery period, and a second surgery was required, which did not result in any significant functional improvement.

The plaintiff sued the treating PA and supervising emergency physician (EP) for failure to properly diagnose the biceps tendon rupture, failure to appreciate the existence of a 3-week window of opportunity to repair the distal biceps tendon rupture, and failure to obtain an urgent orthopedic referral. The experts for the defense argued that the poor outcome was not a consequence of any delay in diagnosis or surgical repair. In addition, the defense disputed the existence of a 3-week window of opportunity for successful repair of a distal biceps tendon rupture. The jury returned a defense verdict.

Discussion

Proximal and Distal Biceps Tendon Ruptures

While both proximal and distal biceps tendon ruptures involve the biceps brachii, they are managed differently and have the potential for very different outcomes.1 At its proximal attachment, the biceps has two distinct tendinous insertions—the long head and the short head. For the distal attachment, the two muscle bellies unite at the midshaft of the humerus and attach as a single tendon on the radial tuberosity. In general, 96% of biceps tendon ruptures involve the long head, 1% involve the short head, and only 3% involve the distal tendon.1 Biceps tendon ruptures occur more commonly in men, patients who use anabolic steroids, cigarette smokers, patient history of tendinopathy, or patients who have a rotator cuff tear.1 Biceps tendon ruptures have not been found to be associated with statin use.2 The mechanism of injury includes heavy-lifting activities, such as weight lifting and rock climbing. However, when associated with a tendinopathy, minimal force may be involved.1

Signs and Symptoms

For proximal biceps tendon rupture, patients usually present with an acute or gradual onset of pain, swelling, and bruising of the upper arm and shoulder. Occasionally, if there is an inciting event, the patient may describe hearing or feeling a “popping” or “snapping” sound. On physical examination, the patient may exhibit a “Popeye” sign—a bulge in the distal biceps area due to the retracted biceps muscle belly. There is also tenderness along the biceps.

 

 

On testing, it has been estimated that patients can experience strength loss of approximately 30% with elbow flexion.1 In contrast, patients with distal biceps tendon ruptures usually complain of pain, swelling, and possibly bruising in the antecubital fossa, as was the case with this patient. Similar to proximal ruptures, the patient may admit to hearing or feeling a “popping” sound if there is an inciting event. The patient may exhibit a “reverse Popeye” deformity, with a bulge in the proximal arm secondary to retraction of the biceps muscle belly proximally.1

Diagnosis

There are two tests that can be performed to assist in making the diagnosis—the biceps squeeze test and the hook test.

Biceps Squeeze Test. The first test to assess for distal biceps tendon rupture is the biceps squeeze test, in which the clinician forcefully squeezes the patient’s biceps muscle to observe for forearm flexion/supination. This test is similar in principle to the Thompson test for Achilles tendon rupture. If there is no forearm movement, the injury is suspicious for a complete distal biceps tendon rupture. In one observational study of this test, 21 of 22 patients with a positive biceps squeeze test were found to have a complete distal biceps tendon tear at surgery.3

Hook Test. The second test is the hook test. While the patient actively supinates with the elbow flexed at 900, an intact hook test permits the examiner to “hook” his or her index finger under the intact biceps tendon from the lateral side. The absence of a “hook” means that there is no cord-like structure under which the examiner can hook a finger, indicating distal avulsion.4 In one study comparing the hook test to magnetic resonance imaging (MRI) in 33 patients with this suspected injury, the hook test had 100% sensitivity and specificity, while MRI only demonstrated a 92% sensitivity and 85% specificity.4

Imaging Techniques

The need for diagnostic imaging is based somewhat on the location of the rupture—proximal or distal. Ultrasound has been shown to have a high sensitivity and specificity for identifying normal tendons and complete tears of the long head biceps tendon (ie, proximal). It is not sensitive at identifying proximal partial tears, however. For distal ruptures, ultrasound imaging of the distal biceps tendon is technically difficult and not reliable. For patients with suspected distal biceps tendon ruptures, the EP should consult with orthopedic services prior to ordering an MRI. While MRI is considered the gold standard imaging test, it is neither 100% sensitive nor specific. The bottom line is that the absence of pathologic findings on MRI is not sufficient enough to exclude biceps tendon pathology.5

Treatment and Management

Regarding management, the majority of patients with proximal biceps tendon ruptures tend to do well with conservative management. The exception is for younger, active patients who are less willing to accept the cosmetic deformity, or patients whose occupation makes them unable to tolerate minimal weakness or fatigue cramping (eg, carpenters), in which case referral for a surgical repair (tenodesis) may be appropriate.1 However, multiple systematic reviews examining tenotomy vs tenodesis have not shown any functional improvement, only cosmetic.1,6,7

Distal biceps tendon ruptures are usually treated surgically, since conservative management results in a decrease of 30% to 50% supination strength and 20% flexion strength.1,8 This surgery, however, is not without complications. Approximately 20% of the patients will have a minor complication and 5% will have major complications following surgery on the distal biceps tendon.9 It is preferable to operate on distal ruptures less than 4 weeks from the initial injury; otherwise, these injuries may be more difficult to fix, require a graft, and have less predictable outcomes.1 Nonoperative management should be reserved for the elderly or less active patients with multiple comorbidities, especially if the nondominant arm is involved.10

Summary

The PA clearly missed the correct diagnosis on this patient. A more thorough history and focused physical examination would have led to the correct diagnosis sooner, along with earlier surgical repair. It is impossible, however, to know if the outcome would have been any different in this uncommon injury.

References

1. Smith D. Proximal versus distal biceps tendon ruptures: when to refer. BCMJ. 2017;59(2):85.

2. Spoendlin J, Layton JB, Mundkur M, Meier C, Jick SS, Meier CR. The risk of achilles or biceps tendon rupture in new statin users: a propensity score-matched sequential cohort study. Drug Safety. 2016;39(12):1229-1237. doi:10.1007/s40264-016-0462-5.

3. Ruland RT, Dunbar RP, Bowen JD. The biceps squeeze test for diagnosis of distal biceps tendon ruptures. Clin Orthop Relat Res. 2005;437:128-131.

4. O’Driscoll SW, Goncalves LBJ, Dietz P. The hook test for distal biceps tendon avulsion. Am J Sports Med. 2007;35(11):1865-1969. doi:10.1177/0363546507305016.

5. Malavolta EA, Assunção JH, Guglielmetti CL, de Souza FF, Gracitelli ME, Ferreira Neto AA. Accuracy of preoperative MRI in the diagnosis of disorders of the long head of the biceps tendon. Eur J Radiol. 2015;84(11):2250-2254. doi:10.1016/j.ejrad.2015.07.031.

6. Tangari M, Carbone S, Gallo M, Campi A. Long head of the biceps tendon rupture in professional wrestlers: treatment with a mini-open tenodesis. J Shoulder Elbow Surg. 2011;20(3):409-413. doi:10.1016/j.jse.2010.08.008.

7. Eakin JL, Bailey JR, Dewing CB, Provencher MT. Subpectoral biceps tenodesis. Oper Tech Sports Med. 2012;20(3):244-252.

8. Thomas JR, Lawton JN. Biceps and triceps ruptures in athletes. Hand Clin. 2017;33(1):35-46. doi:10.1016/j.hcl.2016.08.019.

9. Beks RB, Claessen FM, Oh LS, Ring D, Chen NC. Factors associated with adverse events after distal biceps tendon repair or reconstruction. J Shoulder Elbow Surg. 2016;25(8):1229-1234. doi:10.1016/j.jse.2016.02.032.

10. Savin DD, Watson J, Youderian AR, et al. Surgical management of acute distal biceps tendon ruptures. J Bone Joint Surg. 2017;3(9):785-796. doi:0.2106/JBJS.17.00080.

Article PDF
Author and Disclosure Information

Author’s Disclosure Statement: The author reports no actual or potential conflict of interest in relation to this article.

Issue
Emergency Medicine - 49(12)
Publications
Topics
Page Number
560-562
Sections
Author and Disclosure Information

Author’s Disclosure Statement: The author reports no actual or potential conflict of interest in relation to this article.

Author and Disclosure Information

Author’s Disclosure Statement: The author reports no actual or potential conflict of interest in relation to this article.

Article PDF
Article PDF

A 42-year-old man presented to the ED with left arm pain secondary to an injury he sustained at work. The patient stated that he had been helping to lift a heavy steel beam at a construction site when he experienced abrupt onset of pain in his left arm. He further noted that his left arm felt slightly weaker than normal after the injury.

The patient was left-hand dominant, denied any other injury, was otherwise in good health, and on no medications. With the exception of an appendectomy at age 12 years, his medical history was unremarkable. Regarding his social history, he admitted to smoking one pack of cigarettes per day, and to occasional alcohol consumption. He had no known drug allergies.

On physical examination, the patient’s vital signs were: blood pressure, 125/76 mm Hg; heart rate, 78 beats/min; respiratory rate, 16 breaths/min; and temperature, 98.6°F. Oxygen saturation was 99% on room air.

Examination of the patient’s left shoulder revealed no swelling or tenderness; he was able to fully internally/externally rotate the left shoulder, and lift his left hand above his head. The patient did have tenderness along the biceps area of the left arm, but no tenderness in the triceps area. The left elbow was tender in the antecubital fossa, but without swelling. He had full range of motion of the left elbow but with some pain. He likewise had full range of motion in his left wrist, but no tenderness or swelling. The left radial pulse was 2+. The patient had 5/5 grip strength with the left hand and good capillary refill.

The physician assistant (PA) evaluating the patient diagnosed an arm strain. At discharge, he referred the patient to an occupational health physician (OHP) for follow-up. He also instructed the patient to take ibuprofen 400 mg every 6 to 8 hours, and to limit use of his left arm for 3 days.

The patient followed up with the OHP approximately 3 weeks after discharge from the ED. The OHP was concerned the patient had experienced a distal biceps tendon rupture and referred the patient emergently to an orthopedic surgeon. The orthopedic surgeon saw the patient the next day, agreed with the diagnosis of a distal biceps tendon rupture, and attempted surgical repair the following day. The orthopedic surgeon informed the patient prior to the surgery that the delay in the referral and surgery could result in a poor functional outcome. The patient did have a difficult recovery period, and a second surgery was required, which did not result in any significant functional improvement.

The plaintiff sued the treating PA and supervising emergency physician (EP) for failure to properly diagnose the biceps tendon rupture, failure to appreciate the existence of a 3-week window of opportunity to repair the distal biceps tendon rupture, and failure to obtain an urgent orthopedic referral. The experts for the defense argued that the poor outcome was not a consequence of any delay in diagnosis or surgical repair. In addition, the defense disputed the existence of a 3-week window of opportunity for successful repair of a distal biceps tendon rupture. The jury returned a defense verdict.

Discussion

Proximal and Distal Biceps Tendon Ruptures

While both proximal and distal biceps tendon ruptures involve the biceps brachii, they are managed differently and have the potential for very different outcomes.1 At its proximal attachment, the biceps has two distinct tendinous insertions—the long head and the short head. For the distal attachment, the two muscle bellies unite at the midshaft of the humerus and attach as a single tendon on the radial tuberosity. In general, 96% of biceps tendon ruptures involve the long head, 1% involve the short head, and only 3% involve the distal tendon.1 Biceps tendon ruptures occur more commonly in men, patients who use anabolic steroids, cigarette smokers, patient history of tendinopathy, or patients who have a rotator cuff tear.1 Biceps tendon ruptures have not been found to be associated with statin use.2 The mechanism of injury includes heavy-lifting activities, such as weight lifting and rock climbing. However, when associated with a tendinopathy, minimal force may be involved.1

Signs and Symptoms

For proximal biceps tendon rupture, patients usually present with an acute or gradual onset of pain, swelling, and bruising of the upper arm and shoulder. Occasionally, if there is an inciting event, the patient may describe hearing or feeling a “popping” or “snapping” sound. On physical examination, the patient may exhibit a “Popeye” sign—a bulge in the distal biceps area due to the retracted biceps muscle belly. There is also tenderness along the biceps.

 

 

On testing, it has been estimated that patients can experience strength loss of approximately 30% with elbow flexion.1 In contrast, patients with distal biceps tendon ruptures usually complain of pain, swelling, and possibly bruising in the antecubital fossa, as was the case with this patient. Similar to proximal ruptures, the patient may admit to hearing or feeling a “popping” sound if there is an inciting event. The patient may exhibit a “reverse Popeye” deformity, with a bulge in the proximal arm secondary to retraction of the biceps muscle belly proximally.1

Diagnosis

There are two tests that can be performed to assist in making the diagnosis—the biceps squeeze test and the hook test.

Biceps Squeeze Test. The first test to assess for distal biceps tendon rupture is the biceps squeeze test, in which the clinician forcefully squeezes the patient’s biceps muscle to observe for forearm flexion/supination. This test is similar in principle to the Thompson test for Achilles tendon rupture. If there is no forearm movement, the injury is suspicious for a complete distal biceps tendon rupture. In one observational study of this test, 21 of 22 patients with a positive biceps squeeze test were found to have a complete distal biceps tendon tear at surgery.3

Hook Test. The second test is the hook test. While the patient actively supinates with the elbow flexed at 900, an intact hook test permits the examiner to “hook” his or her index finger under the intact biceps tendon from the lateral side. The absence of a “hook” means that there is no cord-like structure under which the examiner can hook a finger, indicating distal avulsion.4 In one study comparing the hook test to magnetic resonance imaging (MRI) in 33 patients with this suspected injury, the hook test had 100% sensitivity and specificity, while MRI only demonstrated a 92% sensitivity and 85% specificity.4

Imaging Techniques

The need for diagnostic imaging is based somewhat on the location of the rupture—proximal or distal. Ultrasound has been shown to have a high sensitivity and specificity for identifying normal tendons and complete tears of the long head biceps tendon (ie, proximal). It is not sensitive at identifying proximal partial tears, however. For distal ruptures, ultrasound imaging of the distal biceps tendon is technically difficult and not reliable. For patients with suspected distal biceps tendon ruptures, the EP should consult with orthopedic services prior to ordering an MRI. While MRI is considered the gold standard imaging test, it is neither 100% sensitive nor specific. The bottom line is that the absence of pathologic findings on MRI is not sufficient enough to exclude biceps tendon pathology.5

Treatment and Management

Regarding management, the majority of patients with proximal biceps tendon ruptures tend to do well with conservative management. The exception is for younger, active patients who are less willing to accept the cosmetic deformity, or patients whose occupation makes them unable to tolerate minimal weakness or fatigue cramping (eg, carpenters), in which case referral for a surgical repair (tenodesis) may be appropriate.1 However, multiple systematic reviews examining tenotomy vs tenodesis have not shown any functional improvement, only cosmetic.1,6,7

Distal biceps tendon ruptures are usually treated surgically, since conservative management results in a decrease of 30% to 50% supination strength and 20% flexion strength.1,8 This surgery, however, is not without complications. Approximately 20% of the patients will have a minor complication and 5% will have major complications following surgery on the distal biceps tendon.9 It is preferable to operate on distal ruptures less than 4 weeks from the initial injury; otherwise, these injuries may be more difficult to fix, require a graft, and have less predictable outcomes.1 Nonoperative management should be reserved for the elderly or less active patients with multiple comorbidities, especially if the nondominant arm is involved.10

Summary

The PA clearly missed the correct diagnosis on this patient. A more thorough history and focused physical examination would have led to the correct diagnosis sooner, along with earlier surgical repair. It is impossible, however, to know if the outcome would have been any different in this uncommon injury.

A 42-year-old man presented to the ED with left arm pain secondary to an injury he sustained at work. The patient stated that he had been helping to lift a heavy steel beam at a construction site when he experienced abrupt onset of pain in his left arm. He further noted that his left arm felt slightly weaker than normal after the injury.

The patient was left-hand dominant, denied any other injury, was otherwise in good health, and on no medications. With the exception of an appendectomy at age 12 years, his medical history was unremarkable. Regarding his social history, he admitted to smoking one pack of cigarettes per day, and to occasional alcohol consumption. He had no known drug allergies.

On physical examination, the patient’s vital signs were: blood pressure, 125/76 mm Hg; heart rate, 78 beats/min; respiratory rate, 16 breaths/min; and temperature, 98.6°F. Oxygen saturation was 99% on room air.

Examination of the patient’s left shoulder revealed no swelling or tenderness; he was able to fully internally/externally rotate the left shoulder, and lift his left hand above his head. The patient did have tenderness along the biceps area of the left arm, but no tenderness in the triceps area. The left elbow was tender in the antecubital fossa, but without swelling. He had full range of motion of the left elbow but with some pain. He likewise had full range of motion in his left wrist, but no tenderness or swelling. The left radial pulse was 2+. The patient had 5/5 grip strength with the left hand and good capillary refill.

The physician assistant (PA) evaluating the patient diagnosed an arm strain. At discharge, he referred the patient to an occupational health physician (OHP) for follow-up. He also instructed the patient to take ibuprofen 400 mg every 6 to 8 hours, and to limit use of his left arm for 3 days.

The patient followed up with the OHP approximately 3 weeks after discharge from the ED. The OHP was concerned the patient had experienced a distal biceps tendon rupture and referred the patient emergently to an orthopedic surgeon. The orthopedic surgeon saw the patient the next day, agreed with the diagnosis of a distal biceps tendon rupture, and attempted surgical repair the following day. The orthopedic surgeon informed the patient prior to the surgery that the delay in the referral and surgery could result in a poor functional outcome. The patient did have a difficult recovery period, and a second surgery was required, which did not result in any significant functional improvement.

The plaintiff sued the treating PA and supervising emergency physician (EP) for failure to properly diagnose the biceps tendon rupture, failure to appreciate the existence of a 3-week window of opportunity to repair the distal biceps tendon rupture, and failure to obtain an urgent orthopedic referral. The experts for the defense argued that the poor outcome was not a consequence of any delay in diagnosis or surgical repair. In addition, the defense disputed the existence of a 3-week window of opportunity for successful repair of a distal biceps tendon rupture. The jury returned a defense verdict.

Discussion

Proximal and Distal Biceps Tendon Ruptures

While both proximal and distal biceps tendon ruptures involve the biceps brachii, they are managed differently and have the potential for very different outcomes.1 At its proximal attachment, the biceps has two distinct tendinous insertions—the long head and the short head. For the distal attachment, the two muscle bellies unite at the midshaft of the humerus and attach as a single tendon on the radial tuberosity. In general, 96% of biceps tendon ruptures involve the long head, 1% involve the short head, and only 3% involve the distal tendon.1 Biceps tendon ruptures occur more commonly in men, patients who use anabolic steroids, cigarette smokers, patient history of tendinopathy, or patients who have a rotator cuff tear.1 Biceps tendon ruptures have not been found to be associated with statin use.2 The mechanism of injury includes heavy-lifting activities, such as weight lifting and rock climbing. However, when associated with a tendinopathy, minimal force may be involved.1

Signs and Symptoms

For proximal biceps tendon rupture, patients usually present with an acute or gradual onset of pain, swelling, and bruising of the upper arm and shoulder. Occasionally, if there is an inciting event, the patient may describe hearing or feeling a “popping” or “snapping” sound. On physical examination, the patient may exhibit a “Popeye” sign—a bulge in the distal biceps area due to the retracted biceps muscle belly. There is also tenderness along the biceps.

 

 

On testing, it has been estimated that patients can experience strength loss of approximately 30% with elbow flexion.1 In contrast, patients with distal biceps tendon ruptures usually complain of pain, swelling, and possibly bruising in the antecubital fossa, as was the case with this patient. Similar to proximal ruptures, the patient may admit to hearing or feeling a “popping” sound if there is an inciting event. The patient may exhibit a “reverse Popeye” deformity, with a bulge in the proximal arm secondary to retraction of the biceps muscle belly proximally.1

Diagnosis

There are two tests that can be performed to assist in making the diagnosis—the biceps squeeze test and the hook test.

Biceps Squeeze Test. The first test to assess for distal biceps tendon rupture is the biceps squeeze test, in which the clinician forcefully squeezes the patient’s biceps muscle to observe for forearm flexion/supination. This test is similar in principle to the Thompson test for Achilles tendon rupture. If there is no forearm movement, the injury is suspicious for a complete distal biceps tendon rupture. In one observational study of this test, 21 of 22 patients with a positive biceps squeeze test were found to have a complete distal biceps tendon tear at surgery.3

Hook Test. The second test is the hook test. While the patient actively supinates with the elbow flexed at 900, an intact hook test permits the examiner to “hook” his or her index finger under the intact biceps tendon from the lateral side. The absence of a “hook” means that there is no cord-like structure under which the examiner can hook a finger, indicating distal avulsion.4 In one study comparing the hook test to magnetic resonance imaging (MRI) in 33 patients with this suspected injury, the hook test had 100% sensitivity and specificity, while MRI only demonstrated a 92% sensitivity and 85% specificity.4

Imaging Techniques

The need for diagnostic imaging is based somewhat on the location of the rupture—proximal or distal. Ultrasound has been shown to have a high sensitivity and specificity for identifying normal tendons and complete tears of the long head biceps tendon (ie, proximal). It is not sensitive at identifying proximal partial tears, however. For distal ruptures, ultrasound imaging of the distal biceps tendon is technically difficult and not reliable. For patients with suspected distal biceps tendon ruptures, the EP should consult with orthopedic services prior to ordering an MRI. While MRI is considered the gold standard imaging test, it is neither 100% sensitive nor specific. The bottom line is that the absence of pathologic findings on MRI is not sufficient enough to exclude biceps tendon pathology.5

Treatment and Management

Regarding management, the majority of patients with proximal biceps tendon ruptures tend to do well with conservative management. The exception is for younger, active patients who are less willing to accept the cosmetic deformity, or patients whose occupation makes them unable to tolerate minimal weakness or fatigue cramping (eg, carpenters), in which case referral for a surgical repair (tenodesis) may be appropriate.1 However, multiple systematic reviews examining tenotomy vs tenodesis have not shown any functional improvement, only cosmetic.1,6,7

Distal biceps tendon ruptures are usually treated surgically, since conservative management results in a decrease of 30% to 50% supination strength and 20% flexion strength.1,8 This surgery, however, is not without complications. Approximately 20% of the patients will have a minor complication and 5% will have major complications following surgery on the distal biceps tendon.9 It is preferable to operate on distal ruptures less than 4 weeks from the initial injury; otherwise, these injuries may be more difficult to fix, require a graft, and have less predictable outcomes.1 Nonoperative management should be reserved for the elderly or less active patients with multiple comorbidities, especially if the nondominant arm is involved.10

Summary

The PA clearly missed the correct diagnosis on this patient. A more thorough history and focused physical examination would have led to the correct diagnosis sooner, along with earlier surgical repair. It is impossible, however, to know if the outcome would have been any different in this uncommon injury.

References

1. Smith D. Proximal versus distal biceps tendon ruptures: when to refer. BCMJ. 2017;59(2):85.

2. Spoendlin J, Layton JB, Mundkur M, Meier C, Jick SS, Meier CR. The risk of achilles or biceps tendon rupture in new statin users: a propensity score-matched sequential cohort study. Drug Safety. 2016;39(12):1229-1237. doi:10.1007/s40264-016-0462-5.

3. Ruland RT, Dunbar RP, Bowen JD. The biceps squeeze test for diagnosis of distal biceps tendon ruptures. Clin Orthop Relat Res. 2005;437:128-131.

4. O’Driscoll SW, Goncalves LBJ, Dietz P. The hook test for distal biceps tendon avulsion. Am J Sports Med. 2007;35(11):1865-1969. doi:10.1177/0363546507305016.

5. Malavolta EA, Assunção JH, Guglielmetti CL, de Souza FF, Gracitelli ME, Ferreira Neto AA. Accuracy of preoperative MRI in the diagnosis of disorders of the long head of the biceps tendon. Eur J Radiol. 2015;84(11):2250-2254. doi:10.1016/j.ejrad.2015.07.031.

6. Tangari M, Carbone S, Gallo M, Campi A. Long head of the biceps tendon rupture in professional wrestlers: treatment with a mini-open tenodesis. J Shoulder Elbow Surg. 2011;20(3):409-413. doi:10.1016/j.jse.2010.08.008.

7. Eakin JL, Bailey JR, Dewing CB, Provencher MT. Subpectoral biceps tenodesis. Oper Tech Sports Med. 2012;20(3):244-252.

8. Thomas JR, Lawton JN. Biceps and triceps ruptures in athletes. Hand Clin. 2017;33(1):35-46. doi:10.1016/j.hcl.2016.08.019.

9. Beks RB, Claessen FM, Oh LS, Ring D, Chen NC. Factors associated with adverse events after distal biceps tendon repair or reconstruction. J Shoulder Elbow Surg. 2016;25(8):1229-1234. doi:10.1016/j.jse.2016.02.032.

10. Savin DD, Watson J, Youderian AR, et al. Surgical management of acute distal biceps tendon ruptures. J Bone Joint Surg. 2017;3(9):785-796. doi:0.2106/JBJS.17.00080.

References

1. Smith D. Proximal versus distal biceps tendon ruptures: when to refer. BCMJ. 2017;59(2):85.

2. Spoendlin J, Layton JB, Mundkur M, Meier C, Jick SS, Meier CR. The risk of achilles or biceps tendon rupture in new statin users: a propensity score-matched sequential cohort study. Drug Safety. 2016;39(12):1229-1237. doi:10.1007/s40264-016-0462-5.

3. Ruland RT, Dunbar RP, Bowen JD. The biceps squeeze test for diagnosis of distal biceps tendon ruptures. Clin Orthop Relat Res. 2005;437:128-131.

4. O’Driscoll SW, Goncalves LBJ, Dietz P. The hook test for distal biceps tendon avulsion. Am J Sports Med. 2007;35(11):1865-1969. doi:10.1177/0363546507305016.

5. Malavolta EA, Assunção JH, Guglielmetti CL, de Souza FF, Gracitelli ME, Ferreira Neto AA. Accuracy of preoperative MRI in the diagnosis of disorders of the long head of the biceps tendon. Eur J Radiol. 2015;84(11):2250-2254. doi:10.1016/j.ejrad.2015.07.031.

6. Tangari M, Carbone S, Gallo M, Campi A. Long head of the biceps tendon rupture in professional wrestlers: treatment with a mini-open tenodesis. J Shoulder Elbow Surg. 2011;20(3):409-413. doi:10.1016/j.jse.2010.08.008.

7. Eakin JL, Bailey JR, Dewing CB, Provencher MT. Subpectoral biceps tenodesis. Oper Tech Sports Med. 2012;20(3):244-252.

8. Thomas JR, Lawton JN. Biceps and triceps ruptures in athletes. Hand Clin. 2017;33(1):35-46. doi:10.1016/j.hcl.2016.08.019.

9. Beks RB, Claessen FM, Oh LS, Ring D, Chen NC. Factors associated with adverse events after distal biceps tendon repair or reconstruction. J Shoulder Elbow Surg. 2016;25(8):1229-1234. doi:10.1016/j.jse.2016.02.032.

10. Savin DD, Watson J, Youderian AR, et al. Surgical management of acute distal biceps tendon ruptures. J Bone Joint Surg. 2017;3(9):785-796. doi:0.2106/JBJS.17.00080.

Issue
Emergency Medicine - 49(12)
Issue
Emergency Medicine - 49(12)
Page Number
560-562
Page Number
560-562
Publications
Publications
Topics
Article Type
Sections
Disallow All Ads
Content Gating
No Gating (article Unlocked/Free)
Alternative CME
Disqus Comments
Default
Article PDF Media

Malpractice Counsel: A Pain in the…Scrotum

Article Type
Changed
Wed, 12/12/2018 - 21:07

Case

A 52-year-old man presented to the ED for evaluation of right scrotal pain and swelling. The patient stated that the pain started several hours prior to presentation and had gradually worsened. He denied any trauma or inciting event to the affected area; he further denied abdominal pain, nausea, vomiting, dysuria, polyuria, or fever. The patient’s remote medical history was significant for type 2 diabetes mellitus (DM), which he managed through dietary modification-only as he had refused pharmacological therapy. The patient admitted to smoking one half-pack of cigarettes per week, but denied alcohol or illicit drug use.

At presentation, the patient’s vital signs were all within normal range. The physical examination was remarkable only for right testicular tenderness and mild scrotal swelling, and there were no hernias or lymphadenopathy present.

The emergency physician (EP) ordered a urinalysis and color-flow Doppler ultrasound study of both testes, which the radiologist interpreted as an enlarged right epididymis with hyperemia; the left testicle was normal. The urinalysis was normal.

The patient was diagnosed with epididymitis and discharged home with a prescription for oral levofloxacin 500 mg daily for 10 days. He also was instructed to take ibuprofen for pain, apply ice to the affected area, keep the scrotal area elevated, and follow-up with a urologist in 1 week.

Approximately 8 hours after discharge, the patient returned to the same ED with complaints of increasing right testicular pain and swelling. The history and physical examination at this visit were essentially unchanged from his initial presentation. No laboratory evaluation, imaging studies, or other tests were ordered at the second visit.

The patient was discharged home with a prescription for a narcotic analgesic, which he was instructed to take in addition to the ibuprofen; he was also instructed to follow-up with a urologist within the next 2 to 3 days, instead of in 1 week.

The patient returned the following morning to the same ED with complaints of increased swelling and pain of the right testicle. In addition to the worsening testicular pain and swelling, he also had right inguinal pain, nausea, vomiting, and fever. Vital signs at this third presentation were: blood pressure (BP), 124/64 mm Hg; heart rate (HR), 110 beats/min; respiratory rate, 20 breaths/min; and temperature, 99.8o F. Oxygen saturation was 98% on room air.

The patient was tachycardic on heart examination, but with regular rhythm and no murmurs, rubs, or gallops. The lung and abdominal examinations were normal. The genital examination revealed marked right scrotal swelling and tenderness, as well as tender right inguinal lymphadenopathy.

The EP ordered an intravenous (IV) bolus of 1 L normal saline and laboratory studies, which included lactic acid, blood cultures, urinalysis, and urine culture and sensitivity. The EP was concerned for a scrotal abscess and ordered a testicular Doppler color-flow ultrasound study. The laboratory studies revealed an elevated white blood count of 16.5 K/uL, elevated blood glucose of 364 mg/dL, and elevated lactate of 2.8 mg/dL. As demonstrated on the ultrasound study performed at the patient’s first presentation, the ultrasound again showed an enlarged right epididymis, but without orchitis or abscess. The scrotal wall had significant thickening, consistent with cellulitis. The EP ordered broad spectrum IV antibiotics and admitted the patient to the hospitalist with a consult request for urology services.

The patient continued to receive IV fluids and antibiotics throughout the evening. In the morning, he was seen by the same hospitalist/admitting physician from the previous evening. Upon physical examination, the hospitalist noted tenderness, swelling, and erythema in the patient’s perineal area. The patient’s BP had dropped to 100/60 mm Hg, and his HR had increased to 115 beats/min despite receiving nearly 2 L of normal saline IV throughout the previous evening and night.

The urologist examined the patient soon after the consult request and diagnosed him with Fournier’s gangrene. He started the patient on aggressive IV fluid resuscitation, after which the patient was immediately taken to the operating room for extensive surgical debridement and scrotectomy. The patient’s postoperative course was complicated by acute kidney injury, respiratory failure requiring ventilator support, and sepsis. After a lengthy hospital stay, the patient was discharged home, but required a scrotal skin graft, and experienced erectile dysfunction and depression.

The patient sued all of the EPs involved in his care, the hospital, the hospitalist/admitting physician, and the urologist for negligence. The plaintiff’s attorney argued that since the patient progressively deteriorated over the 24 to 36 hours during his three presentations to the ED, urology services should have been consulted earlier, and that the urologist should have seen the patient immediately at the time of hospital admission.

The attorneys for the defendants claimed the patient denied dysuria, penile lesions, or urethral discharge and that the history, physical examination, and testicular ultrasound were all consistent with the diagnosis of epididymitis. For this reason, they argued, there was no indication for an emergent consultation with urology services. The jury returned a defense verdict.

 

 

Discussion

It is easy for a busy EP to have a differential diagnosis of only two disorders when evaluating a patient for unilateral testicular pain and swelling—in this case, testicular torsion and epididymitis. While these are the most common causes of testicular pain and swelling, this case emphasizes the need to also consider Fournier’s gangrene in the differential. A thorough history and physical examination, coupled with appropriate testing, will usually identify the correct diagnosis. While the differential diagnosis is broader than just these three disease processes (see the Box), we will review the evaluation and management of the three most serious: epididymitis, testicular torsion, and Fournier’s gangrene.

Box.

Noninfectious and Bacterial Epididymitis

Epididymitis is the most common cause of acute scrotal pain among US adults, accounting for approximately 600,000 cases each year.1 Infectious epididymitis is typically classified as acute (symptom duration of <6 weeks) or chronic (symptom duration of ≥6 weeks).2

Cases of noninfectious epididymitis are typically due to a chronic condition, such as autoimmune disease, cancer, or vasculitis. Although not as common, noninfectious epididymitis can also occur due to testicular trauma or amiodarone therapy.3,4

Patients with acute bacterial epididymitis typically present with scrotal pain and swelling ranging from mild to marked. These patients may also exhibit fever and chills, along with dysuria, frequency, and urgency, if associated with a urinary tract infection.2 The chronic presentation is more common though, and usually not associated with voiding issues.

Chronic epididymis is frequently seen in postpubertal boys and men following sexual activity, heavy physical exertion, and bicycle/motorcycle riding.2 On physical examination, palpation reveals induration and swelling of the involved epididymis with exquisite tenderness.2 Testicular swelling and pain, along with scrotal wall erythema, may be present in more advanced cases.2 The cremasteric reflex should be intact (ie, scratching the medial proximal thigh will cause ipsilateral testicle retraction). Similarly, the lie of both testicles while the patient is standing should be equal and symmetrical—ie, both testicles descended equally. However, in the presence of moderate-to-severe scrotal swelling, both of these physical findings may be impossible to confirm.

A urinalysis and urine culture should be ordered if there is any suspicion of epididymitis; pyuria will be present in approximately 50% of cases. However, since pyuria is neither sensitive nor specific for epididymitis, in most cases, a testicular ultrasound with Doppler flow is required to exclude testicular torsion. In cases of epididymitis, ultrasound usually demonstrates increased flow on the affected side, whereas in testicular torsion, there is decreased or absent blood flow.

The treatment for epididymitis involves antibiotics and symptomatic care. If epididymitis from chlamydia and/or gonorrhea is the suspected cause, or if the patient is younger than age 35 years, he should be given ceftriaxone 250 mg intramuscularly plus oral doxycycline 100 mg twice a day for 10 days. Patients who practice insertive anal sex should be treated with ceftriaxone, plus either oral ofloxacin 300 mg twice a day or oral levofloxacin 500 mg daily for 10 days.

In cases in which enteric organisms are suspected, the patient is older than age 35 years, or if patient status is posturinary tract instrumentation or vasectomy, he should be treated with either oral ofloxacin 300 mg twice a day or oral levofloxacin 500 mg daily for 10 days.2

For symptomatic relief, scrotal elevation, ice application, and nonsteroidal anti-inflammatory drugs are recommended.

Patients with epididymitis, regardless of etiology, should be instructed to follow-up with a urologist within 1 week. If the patient appears ill, septic, or in significant pain, admission to the hospital with IV antibiotics, IV fluids, and an urgent consult with urology services is required.

Testicular Torsion

Testicular torsion is a time-sensitive issue, requiring early diagnosis and rapid treatment to preserve the patient’s fertility. Most clinicians recommend detorsion within 6 hours of torsion onset because salvage rates are excellent when performed within this timeframe; after 12 hours, the testis will likely suffer irreversible damage due to ischemia.5,6

Testicular torsion can occur at any age, but is most commonly seen in a bimodal distribution—ie, neonates and postpubertal boys. The prevalence of testicular torsion in adult patients hospitalized with acute scrotal pain is approximately 25% to 50%.2

Patients with testicular torsion usually describe a sudden onset of severe, acute pain. The pain frequently occurs a few hours after vigorous physical activity or minor testicular trauma.2 Occasionally, the patient may complain of lower quadrant abdominal pain rather than testicular or scrotal pain. Nausea with vomiting can also be present.

On physical examination, significant testicular swelling is usually present. Examining the patient in the standing position will often reveal an asymmetrical, high-riding testis with a transverse lie on the affected side. The cremasteric reflex is usually absent in patients with testicular torsion.

Because of the significant overlap in history and physical examination findings for epididymitis and testicular torsion, a testicular ultrasound with color Doppler should be ordered. Multiple studies have confirmed the high sensitivity and specificity of ultrasound in the diagnosis of testicular torsion.

The treatment for suspected or confirmed testicular torsion is immediate surgical exploration with intraoperative detorsion and fixation of the testes. The EP can attempt manual detorsion (ie, performed in a medial to lateral motion, similar to opening a book). However, this should not delay the EP from consulting with urology services.

Pediatric patients with testicular torsion usually have a more favorable outcome than do adults. In one retrospective study, patients younger than age 21 years had a 70% testicular salvage rate compared to only 41% of patients aged 21 years and older.7 Regardless of age, better outcomes are associated with shorter periods of torsion.

 

 

Fournier’s Gangrene

Fournier’s gangrene is a polymicrobial necrotizing fasciitis of the perineum and scrotum that typically develops initially as a benign infection or abscess but quickly spreads. Risk factors for Fournier’s gangrene include DM, alcohol abuse, and any immunocompromised state (eg, HIV, cancer).

If the patient presents early in onset, there may be only mild tenderness, erythema, or swelling of the affected area; however, this infection progresses rapidly. Later findings include marked tenderness, swelling, crepitus, blisters, and ecchymoses. Patients with Fournier’s gangrene also develop systemic signs of infection, including fever, tachycardia, tachypnea, and hypotension. The key to diagnosis is careful examination of the perineal and scrotal area in any patient presenting with acute scrotal pain.

In the majority of cases, the diagnosis of Fournier’s gangrene is made clinically. Once the diagnosis is made, patients require immediate and aggressive IV fluid resuscitation, broad-spectrum IV antibiotics (typically vancomycin and piperacillin/tazobactam), and emergent evaluation by a urologist. It is essential that these patients undergo early and aggressive surgical exploration and debridement of necrotic tissue.2 Antibiotic therapy alone is associated with a 100% mortality rate, emphasizing the need for urgent surgery.2 Even with optimal medical and surgical management, the mortality rate remains significant.

Summary

This case emphasizes several important teaching points. The EP should be mindful of the patient who keeps returning to the ED with the same complaint—despite “appropriate” treatment—as the initial diagnosis may not be the correct one. Such returning patients require greater, not less, scrutiny. As with any patient, the EP should always take a complete history and perform a thorough physical examination at each presentation—as one would with a de novo patient. Finally, the EP should consider Fournier’s gangrene in addition to testicular torsion and epididymitis in the differential diagnosis for acute scrotal pain.

References

1. Trojian TH, Lishnak TS, Heiman D. Epididymitis and orchitis: an overview. Am Fam Physician. 2009;79(7):583-587.

2. Eyre RC. Evaluation of acute scrotal pain in adults. UpToDate Web site. https://www.uptodate.com/contents/evaluation-of-acute-scrotal-pain-in-adults. Updated July 31, 2017. Accessed September 7, 2017.

3. Shen Y, Liu H, Cheng J, Bu P. Amiodarone-induced epididymitis: a pathologically confirmed case report and review of the literature. Cardiology. 2014;128(4):349-351. doi:10.1159/000361038.

4. Tracy CR, Steers WD, Costabile R. Diagnosis and management of epididymitis. Urol Clin North Am. 2008;35(1):101-108. doi:10.1016/j.ucl.2007.09.013.

5. Wampler SM, Llanes M. Common scrotal and testicular problems. Prim Care. 2010;37(3):613-626. doi:10.1016/j.pop.2010.04.009.

6. Dunne PJ, O’Loughlin BS. Testicular torsion: time is the enemy. Aust NZ J Surg. 2000;70(6):441-442.

7. Cummings JM, Boullier JA, Sekhon D, Bose K. Adult testicular torsion. J Urol. 2002;167(5):2109-2110.

Article PDF
Author and Disclosure Information

Author’s Disclosure Statement: The author reports no actual or potential conflict of interest in relation to this article.

Issue
Emergency Medicine - 49(10)
Publications
Topics
Page Number
466-469
Sections
Author and Disclosure Information

Author’s Disclosure Statement: The author reports no actual or potential conflict of interest in relation to this article.

Author and Disclosure Information

Author’s Disclosure Statement: The author reports no actual or potential conflict of interest in relation to this article.

Article PDF
Article PDF

Case

A 52-year-old man presented to the ED for evaluation of right scrotal pain and swelling. The patient stated that the pain started several hours prior to presentation and had gradually worsened. He denied any trauma or inciting event to the affected area; he further denied abdominal pain, nausea, vomiting, dysuria, polyuria, or fever. The patient’s remote medical history was significant for type 2 diabetes mellitus (DM), which he managed through dietary modification-only as he had refused pharmacological therapy. The patient admitted to smoking one half-pack of cigarettes per week, but denied alcohol or illicit drug use.

At presentation, the patient’s vital signs were all within normal range. The physical examination was remarkable only for right testicular tenderness and mild scrotal swelling, and there were no hernias or lymphadenopathy present.

The emergency physician (EP) ordered a urinalysis and color-flow Doppler ultrasound study of both testes, which the radiologist interpreted as an enlarged right epididymis with hyperemia; the left testicle was normal. The urinalysis was normal.

The patient was diagnosed with epididymitis and discharged home with a prescription for oral levofloxacin 500 mg daily for 10 days. He also was instructed to take ibuprofen for pain, apply ice to the affected area, keep the scrotal area elevated, and follow-up with a urologist in 1 week.

Approximately 8 hours after discharge, the patient returned to the same ED with complaints of increasing right testicular pain and swelling. The history and physical examination at this visit were essentially unchanged from his initial presentation. No laboratory evaluation, imaging studies, or other tests were ordered at the second visit.

The patient was discharged home with a prescription for a narcotic analgesic, which he was instructed to take in addition to the ibuprofen; he was also instructed to follow-up with a urologist within the next 2 to 3 days, instead of in 1 week.

The patient returned the following morning to the same ED with complaints of increased swelling and pain of the right testicle. In addition to the worsening testicular pain and swelling, he also had right inguinal pain, nausea, vomiting, and fever. Vital signs at this third presentation were: blood pressure (BP), 124/64 mm Hg; heart rate (HR), 110 beats/min; respiratory rate, 20 breaths/min; and temperature, 99.8o F. Oxygen saturation was 98% on room air.

The patient was tachycardic on heart examination, but with regular rhythm and no murmurs, rubs, or gallops. The lung and abdominal examinations were normal. The genital examination revealed marked right scrotal swelling and tenderness, as well as tender right inguinal lymphadenopathy.

The EP ordered an intravenous (IV) bolus of 1 L normal saline and laboratory studies, which included lactic acid, blood cultures, urinalysis, and urine culture and sensitivity. The EP was concerned for a scrotal abscess and ordered a testicular Doppler color-flow ultrasound study. The laboratory studies revealed an elevated white blood count of 16.5 K/uL, elevated blood glucose of 364 mg/dL, and elevated lactate of 2.8 mg/dL. As demonstrated on the ultrasound study performed at the patient’s first presentation, the ultrasound again showed an enlarged right epididymis, but without orchitis or abscess. The scrotal wall had significant thickening, consistent with cellulitis. The EP ordered broad spectrum IV antibiotics and admitted the patient to the hospitalist with a consult request for urology services.

The patient continued to receive IV fluids and antibiotics throughout the evening. In the morning, he was seen by the same hospitalist/admitting physician from the previous evening. Upon physical examination, the hospitalist noted tenderness, swelling, and erythema in the patient’s perineal area. The patient’s BP had dropped to 100/60 mm Hg, and his HR had increased to 115 beats/min despite receiving nearly 2 L of normal saline IV throughout the previous evening and night.

The urologist examined the patient soon after the consult request and diagnosed him with Fournier’s gangrene. He started the patient on aggressive IV fluid resuscitation, after which the patient was immediately taken to the operating room for extensive surgical debridement and scrotectomy. The patient’s postoperative course was complicated by acute kidney injury, respiratory failure requiring ventilator support, and sepsis. After a lengthy hospital stay, the patient was discharged home, but required a scrotal skin graft, and experienced erectile dysfunction and depression.

The patient sued all of the EPs involved in his care, the hospital, the hospitalist/admitting physician, and the urologist for negligence. The plaintiff’s attorney argued that since the patient progressively deteriorated over the 24 to 36 hours during his three presentations to the ED, urology services should have been consulted earlier, and that the urologist should have seen the patient immediately at the time of hospital admission.

The attorneys for the defendants claimed the patient denied dysuria, penile lesions, or urethral discharge and that the history, physical examination, and testicular ultrasound were all consistent with the diagnosis of epididymitis. For this reason, they argued, there was no indication for an emergent consultation with urology services. The jury returned a defense verdict.

 

 

Discussion

It is easy for a busy EP to have a differential diagnosis of only two disorders when evaluating a patient for unilateral testicular pain and swelling—in this case, testicular torsion and epididymitis. While these are the most common causes of testicular pain and swelling, this case emphasizes the need to also consider Fournier’s gangrene in the differential. A thorough history and physical examination, coupled with appropriate testing, will usually identify the correct diagnosis. While the differential diagnosis is broader than just these three disease processes (see the Box), we will review the evaluation and management of the three most serious: epididymitis, testicular torsion, and Fournier’s gangrene.

Box.

Noninfectious and Bacterial Epididymitis

Epididymitis is the most common cause of acute scrotal pain among US adults, accounting for approximately 600,000 cases each year.1 Infectious epididymitis is typically classified as acute (symptom duration of <6 weeks) or chronic (symptom duration of ≥6 weeks).2

Cases of noninfectious epididymitis are typically due to a chronic condition, such as autoimmune disease, cancer, or vasculitis. Although not as common, noninfectious epididymitis can also occur due to testicular trauma or amiodarone therapy.3,4

Patients with acute bacterial epididymitis typically present with scrotal pain and swelling ranging from mild to marked. These patients may also exhibit fever and chills, along with dysuria, frequency, and urgency, if associated with a urinary tract infection.2 The chronic presentation is more common though, and usually not associated with voiding issues.

Chronic epididymis is frequently seen in postpubertal boys and men following sexual activity, heavy physical exertion, and bicycle/motorcycle riding.2 On physical examination, palpation reveals induration and swelling of the involved epididymis with exquisite tenderness.2 Testicular swelling and pain, along with scrotal wall erythema, may be present in more advanced cases.2 The cremasteric reflex should be intact (ie, scratching the medial proximal thigh will cause ipsilateral testicle retraction). Similarly, the lie of both testicles while the patient is standing should be equal and symmetrical—ie, both testicles descended equally. However, in the presence of moderate-to-severe scrotal swelling, both of these physical findings may be impossible to confirm.

A urinalysis and urine culture should be ordered if there is any suspicion of epididymitis; pyuria will be present in approximately 50% of cases. However, since pyuria is neither sensitive nor specific for epididymitis, in most cases, a testicular ultrasound with Doppler flow is required to exclude testicular torsion. In cases of epididymitis, ultrasound usually demonstrates increased flow on the affected side, whereas in testicular torsion, there is decreased or absent blood flow.

The treatment for epididymitis involves antibiotics and symptomatic care. If epididymitis from chlamydia and/or gonorrhea is the suspected cause, or if the patient is younger than age 35 years, he should be given ceftriaxone 250 mg intramuscularly plus oral doxycycline 100 mg twice a day for 10 days. Patients who practice insertive anal sex should be treated with ceftriaxone, plus either oral ofloxacin 300 mg twice a day or oral levofloxacin 500 mg daily for 10 days.

In cases in which enteric organisms are suspected, the patient is older than age 35 years, or if patient status is posturinary tract instrumentation or vasectomy, he should be treated with either oral ofloxacin 300 mg twice a day or oral levofloxacin 500 mg daily for 10 days.2

For symptomatic relief, scrotal elevation, ice application, and nonsteroidal anti-inflammatory drugs are recommended.

Patients with epididymitis, regardless of etiology, should be instructed to follow-up with a urologist within 1 week. If the patient appears ill, septic, or in significant pain, admission to the hospital with IV antibiotics, IV fluids, and an urgent consult with urology services is required.

Testicular Torsion

Testicular torsion is a time-sensitive issue, requiring early diagnosis and rapid treatment to preserve the patient’s fertility. Most clinicians recommend detorsion within 6 hours of torsion onset because salvage rates are excellent when performed within this timeframe; after 12 hours, the testis will likely suffer irreversible damage due to ischemia.5,6

Testicular torsion can occur at any age, but is most commonly seen in a bimodal distribution—ie, neonates and postpubertal boys. The prevalence of testicular torsion in adult patients hospitalized with acute scrotal pain is approximately 25% to 50%.2

Patients with testicular torsion usually describe a sudden onset of severe, acute pain. The pain frequently occurs a few hours after vigorous physical activity or minor testicular trauma.2 Occasionally, the patient may complain of lower quadrant abdominal pain rather than testicular or scrotal pain. Nausea with vomiting can also be present.

On physical examination, significant testicular swelling is usually present. Examining the patient in the standing position will often reveal an asymmetrical, high-riding testis with a transverse lie on the affected side. The cremasteric reflex is usually absent in patients with testicular torsion.

Because of the significant overlap in history and physical examination findings for epididymitis and testicular torsion, a testicular ultrasound with color Doppler should be ordered. Multiple studies have confirmed the high sensitivity and specificity of ultrasound in the diagnosis of testicular torsion.

The treatment for suspected or confirmed testicular torsion is immediate surgical exploration with intraoperative detorsion and fixation of the testes. The EP can attempt manual detorsion (ie, performed in a medial to lateral motion, similar to opening a book). However, this should not delay the EP from consulting with urology services.

Pediatric patients with testicular torsion usually have a more favorable outcome than do adults. In one retrospective study, patients younger than age 21 years had a 70% testicular salvage rate compared to only 41% of patients aged 21 years and older.7 Regardless of age, better outcomes are associated with shorter periods of torsion.

 

 

Fournier’s Gangrene

Fournier’s gangrene is a polymicrobial necrotizing fasciitis of the perineum and scrotum that typically develops initially as a benign infection or abscess but quickly spreads. Risk factors for Fournier’s gangrene include DM, alcohol abuse, and any immunocompromised state (eg, HIV, cancer).

If the patient presents early in onset, there may be only mild tenderness, erythema, or swelling of the affected area; however, this infection progresses rapidly. Later findings include marked tenderness, swelling, crepitus, blisters, and ecchymoses. Patients with Fournier’s gangrene also develop systemic signs of infection, including fever, tachycardia, tachypnea, and hypotension. The key to diagnosis is careful examination of the perineal and scrotal area in any patient presenting with acute scrotal pain.

In the majority of cases, the diagnosis of Fournier’s gangrene is made clinically. Once the diagnosis is made, patients require immediate and aggressive IV fluid resuscitation, broad-spectrum IV antibiotics (typically vancomycin and piperacillin/tazobactam), and emergent evaluation by a urologist. It is essential that these patients undergo early and aggressive surgical exploration and debridement of necrotic tissue.2 Antibiotic therapy alone is associated with a 100% mortality rate, emphasizing the need for urgent surgery.2 Even with optimal medical and surgical management, the mortality rate remains significant.

Summary

This case emphasizes several important teaching points. The EP should be mindful of the patient who keeps returning to the ED with the same complaint—despite “appropriate” treatment—as the initial diagnosis may not be the correct one. Such returning patients require greater, not less, scrutiny. As with any patient, the EP should always take a complete history and perform a thorough physical examination at each presentation—as one would with a de novo patient. Finally, the EP should consider Fournier’s gangrene in addition to testicular torsion and epididymitis in the differential diagnosis for acute scrotal pain.

Case

A 52-year-old man presented to the ED for evaluation of right scrotal pain and swelling. The patient stated that the pain started several hours prior to presentation and had gradually worsened. He denied any trauma or inciting event to the affected area; he further denied abdominal pain, nausea, vomiting, dysuria, polyuria, or fever. The patient’s remote medical history was significant for type 2 diabetes mellitus (DM), which he managed through dietary modification-only as he had refused pharmacological therapy. The patient admitted to smoking one half-pack of cigarettes per week, but denied alcohol or illicit drug use.

At presentation, the patient’s vital signs were all within normal range. The physical examination was remarkable only for right testicular tenderness and mild scrotal swelling, and there were no hernias or lymphadenopathy present.

The emergency physician (EP) ordered a urinalysis and color-flow Doppler ultrasound study of both testes, which the radiologist interpreted as an enlarged right epididymis with hyperemia; the left testicle was normal. The urinalysis was normal.

The patient was diagnosed with epididymitis and discharged home with a prescription for oral levofloxacin 500 mg daily for 10 days. He also was instructed to take ibuprofen for pain, apply ice to the affected area, keep the scrotal area elevated, and follow-up with a urologist in 1 week.

Approximately 8 hours after discharge, the patient returned to the same ED with complaints of increasing right testicular pain and swelling. The history and physical examination at this visit were essentially unchanged from his initial presentation. No laboratory evaluation, imaging studies, or other tests were ordered at the second visit.

The patient was discharged home with a prescription for a narcotic analgesic, which he was instructed to take in addition to the ibuprofen; he was also instructed to follow-up with a urologist within the next 2 to 3 days, instead of in 1 week.

The patient returned the following morning to the same ED with complaints of increased swelling and pain of the right testicle. In addition to the worsening testicular pain and swelling, he also had right inguinal pain, nausea, vomiting, and fever. Vital signs at this third presentation were: blood pressure (BP), 124/64 mm Hg; heart rate (HR), 110 beats/min; respiratory rate, 20 breaths/min; and temperature, 99.8o F. Oxygen saturation was 98% on room air.

The patient was tachycardic on heart examination, but with regular rhythm and no murmurs, rubs, or gallops. The lung and abdominal examinations were normal. The genital examination revealed marked right scrotal swelling and tenderness, as well as tender right inguinal lymphadenopathy.

The EP ordered an intravenous (IV) bolus of 1 L normal saline and laboratory studies, which included lactic acid, blood cultures, urinalysis, and urine culture and sensitivity. The EP was concerned for a scrotal abscess and ordered a testicular Doppler color-flow ultrasound study. The laboratory studies revealed an elevated white blood count of 16.5 K/uL, elevated blood glucose of 364 mg/dL, and elevated lactate of 2.8 mg/dL. As demonstrated on the ultrasound study performed at the patient’s first presentation, the ultrasound again showed an enlarged right epididymis, but without orchitis or abscess. The scrotal wall had significant thickening, consistent with cellulitis. The EP ordered broad spectrum IV antibiotics and admitted the patient to the hospitalist with a consult request for urology services.

The patient continued to receive IV fluids and antibiotics throughout the evening. In the morning, he was seen by the same hospitalist/admitting physician from the previous evening. Upon physical examination, the hospitalist noted tenderness, swelling, and erythema in the patient’s perineal area. The patient’s BP had dropped to 100/60 mm Hg, and his HR had increased to 115 beats/min despite receiving nearly 2 L of normal saline IV throughout the previous evening and night.

The urologist examined the patient soon after the consult request and diagnosed him with Fournier’s gangrene. He started the patient on aggressive IV fluid resuscitation, after which the patient was immediately taken to the operating room for extensive surgical debridement and scrotectomy. The patient’s postoperative course was complicated by acute kidney injury, respiratory failure requiring ventilator support, and sepsis. After a lengthy hospital stay, the patient was discharged home, but required a scrotal skin graft, and experienced erectile dysfunction and depression.

The patient sued all of the EPs involved in his care, the hospital, the hospitalist/admitting physician, and the urologist for negligence. The plaintiff’s attorney argued that since the patient progressively deteriorated over the 24 to 36 hours during his three presentations to the ED, urology services should have been consulted earlier, and that the urologist should have seen the patient immediately at the time of hospital admission.

The attorneys for the defendants claimed the patient denied dysuria, penile lesions, or urethral discharge and that the history, physical examination, and testicular ultrasound were all consistent with the diagnosis of epididymitis. For this reason, they argued, there was no indication for an emergent consultation with urology services. The jury returned a defense verdict.

 

 

Discussion

It is easy for a busy EP to have a differential diagnosis of only two disorders when evaluating a patient for unilateral testicular pain and swelling—in this case, testicular torsion and epididymitis. While these are the most common causes of testicular pain and swelling, this case emphasizes the need to also consider Fournier’s gangrene in the differential. A thorough history and physical examination, coupled with appropriate testing, will usually identify the correct diagnosis. While the differential diagnosis is broader than just these three disease processes (see the Box), we will review the evaluation and management of the three most serious: epididymitis, testicular torsion, and Fournier’s gangrene.

Box.

Noninfectious and Bacterial Epididymitis

Epididymitis is the most common cause of acute scrotal pain among US adults, accounting for approximately 600,000 cases each year.1 Infectious epididymitis is typically classified as acute (symptom duration of <6 weeks) or chronic (symptom duration of ≥6 weeks).2

Cases of noninfectious epididymitis are typically due to a chronic condition, such as autoimmune disease, cancer, or vasculitis. Although not as common, noninfectious epididymitis can also occur due to testicular trauma or amiodarone therapy.3,4

Patients with acute bacterial epididymitis typically present with scrotal pain and swelling ranging from mild to marked. These patients may also exhibit fever and chills, along with dysuria, frequency, and urgency, if associated with a urinary tract infection.2 The chronic presentation is more common though, and usually not associated with voiding issues.

Chronic epididymis is frequently seen in postpubertal boys and men following sexual activity, heavy physical exertion, and bicycle/motorcycle riding.2 On physical examination, palpation reveals induration and swelling of the involved epididymis with exquisite tenderness.2 Testicular swelling and pain, along with scrotal wall erythema, may be present in more advanced cases.2 The cremasteric reflex should be intact (ie, scratching the medial proximal thigh will cause ipsilateral testicle retraction). Similarly, the lie of both testicles while the patient is standing should be equal and symmetrical—ie, both testicles descended equally. However, in the presence of moderate-to-severe scrotal swelling, both of these physical findings may be impossible to confirm.

A urinalysis and urine culture should be ordered if there is any suspicion of epididymitis; pyuria will be present in approximately 50% of cases. However, since pyuria is neither sensitive nor specific for epididymitis, in most cases, a testicular ultrasound with Doppler flow is required to exclude testicular torsion. In cases of epididymitis, ultrasound usually demonstrates increased flow on the affected side, whereas in testicular torsion, there is decreased or absent blood flow.

The treatment for epididymitis involves antibiotics and symptomatic care. If epididymitis from chlamydia and/or gonorrhea is the suspected cause, or if the patient is younger than age 35 years, he should be given ceftriaxone 250 mg intramuscularly plus oral doxycycline 100 mg twice a day for 10 days. Patients who practice insertive anal sex should be treated with ceftriaxone, plus either oral ofloxacin 300 mg twice a day or oral levofloxacin 500 mg daily for 10 days.

In cases in which enteric organisms are suspected, the patient is older than age 35 years, or if patient status is posturinary tract instrumentation or vasectomy, he should be treated with either oral ofloxacin 300 mg twice a day or oral levofloxacin 500 mg daily for 10 days.2

For symptomatic relief, scrotal elevation, ice application, and nonsteroidal anti-inflammatory drugs are recommended.

Patients with epididymitis, regardless of etiology, should be instructed to follow-up with a urologist within 1 week. If the patient appears ill, septic, or in significant pain, admission to the hospital with IV antibiotics, IV fluids, and an urgent consult with urology services is required.

Testicular Torsion

Testicular torsion is a time-sensitive issue, requiring early diagnosis and rapid treatment to preserve the patient’s fertility. Most clinicians recommend detorsion within 6 hours of torsion onset because salvage rates are excellent when performed within this timeframe; after 12 hours, the testis will likely suffer irreversible damage due to ischemia.5,6

Testicular torsion can occur at any age, but is most commonly seen in a bimodal distribution—ie, neonates and postpubertal boys. The prevalence of testicular torsion in adult patients hospitalized with acute scrotal pain is approximately 25% to 50%.2

Patients with testicular torsion usually describe a sudden onset of severe, acute pain. The pain frequently occurs a few hours after vigorous physical activity or minor testicular trauma.2 Occasionally, the patient may complain of lower quadrant abdominal pain rather than testicular or scrotal pain. Nausea with vomiting can also be present.

On physical examination, significant testicular swelling is usually present. Examining the patient in the standing position will often reveal an asymmetrical, high-riding testis with a transverse lie on the affected side. The cremasteric reflex is usually absent in patients with testicular torsion.

Because of the significant overlap in history and physical examination findings for epididymitis and testicular torsion, a testicular ultrasound with color Doppler should be ordered. Multiple studies have confirmed the high sensitivity and specificity of ultrasound in the diagnosis of testicular torsion.

The treatment for suspected or confirmed testicular torsion is immediate surgical exploration with intraoperative detorsion and fixation of the testes. The EP can attempt manual detorsion (ie, performed in a medial to lateral motion, similar to opening a book). However, this should not delay the EP from consulting with urology services.

Pediatric patients with testicular torsion usually have a more favorable outcome than do adults. In one retrospective study, patients younger than age 21 years had a 70% testicular salvage rate compared to only 41% of patients aged 21 years and older.7 Regardless of age, better outcomes are associated with shorter periods of torsion.

 

 

Fournier’s Gangrene

Fournier’s gangrene is a polymicrobial necrotizing fasciitis of the perineum and scrotum that typically develops initially as a benign infection or abscess but quickly spreads. Risk factors for Fournier’s gangrene include DM, alcohol abuse, and any immunocompromised state (eg, HIV, cancer).

If the patient presents early in onset, there may be only mild tenderness, erythema, or swelling of the affected area; however, this infection progresses rapidly. Later findings include marked tenderness, swelling, crepitus, blisters, and ecchymoses. Patients with Fournier’s gangrene also develop systemic signs of infection, including fever, tachycardia, tachypnea, and hypotension. The key to diagnosis is careful examination of the perineal and scrotal area in any patient presenting with acute scrotal pain.

In the majority of cases, the diagnosis of Fournier’s gangrene is made clinically. Once the diagnosis is made, patients require immediate and aggressive IV fluid resuscitation, broad-spectrum IV antibiotics (typically vancomycin and piperacillin/tazobactam), and emergent evaluation by a urologist. It is essential that these patients undergo early and aggressive surgical exploration and debridement of necrotic tissue.2 Antibiotic therapy alone is associated with a 100% mortality rate, emphasizing the need for urgent surgery.2 Even with optimal medical and surgical management, the mortality rate remains significant.

Summary

This case emphasizes several important teaching points. The EP should be mindful of the patient who keeps returning to the ED with the same complaint—despite “appropriate” treatment—as the initial diagnosis may not be the correct one. Such returning patients require greater, not less, scrutiny. As with any patient, the EP should always take a complete history and perform a thorough physical examination at each presentation—as one would with a de novo patient. Finally, the EP should consider Fournier’s gangrene in addition to testicular torsion and epididymitis in the differential diagnosis for acute scrotal pain.

References

1. Trojian TH, Lishnak TS, Heiman D. Epididymitis and orchitis: an overview. Am Fam Physician. 2009;79(7):583-587.

2. Eyre RC. Evaluation of acute scrotal pain in adults. UpToDate Web site. https://www.uptodate.com/contents/evaluation-of-acute-scrotal-pain-in-adults. Updated July 31, 2017. Accessed September 7, 2017.

3. Shen Y, Liu H, Cheng J, Bu P. Amiodarone-induced epididymitis: a pathologically confirmed case report and review of the literature. Cardiology. 2014;128(4):349-351. doi:10.1159/000361038.

4. Tracy CR, Steers WD, Costabile R. Diagnosis and management of epididymitis. Urol Clin North Am. 2008;35(1):101-108. doi:10.1016/j.ucl.2007.09.013.

5. Wampler SM, Llanes M. Common scrotal and testicular problems. Prim Care. 2010;37(3):613-626. doi:10.1016/j.pop.2010.04.009.

6. Dunne PJ, O’Loughlin BS. Testicular torsion: time is the enemy. Aust NZ J Surg. 2000;70(6):441-442.

7. Cummings JM, Boullier JA, Sekhon D, Bose K. Adult testicular torsion. J Urol. 2002;167(5):2109-2110.

References

1. Trojian TH, Lishnak TS, Heiman D. Epididymitis and orchitis: an overview. Am Fam Physician. 2009;79(7):583-587.

2. Eyre RC. Evaluation of acute scrotal pain in adults. UpToDate Web site. https://www.uptodate.com/contents/evaluation-of-acute-scrotal-pain-in-adults. Updated July 31, 2017. Accessed September 7, 2017.

3. Shen Y, Liu H, Cheng J, Bu P. Amiodarone-induced epididymitis: a pathologically confirmed case report and review of the literature. Cardiology. 2014;128(4):349-351. doi:10.1159/000361038.

4. Tracy CR, Steers WD, Costabile R. Diagnosis and management of epididymitis. Urol Clin North Am. 2008;35(1):101-108. doi:10.1016/j.ucl.2007.09.013.

5. Wampler SM, Llanes M. Common scrotal and testicular problems. Prim Care. 2010;37(3):613-626. doi:10.1016/j.pop.2010.04.009.

6. Dunne PJ, O’Loughlin BS. Testicular torsion: time is the enemy. Aust NZ J Surg. 2000;70(6):441-442.

7. Cummings JM, Boullier JA, Sekhon D, Bose K. Adult testicular torsion. J Urol. 2002;167(5):2109-2110.

Issue
Emergency Medicine - 49(10)
Issue
Emergency Medicine - 49(10)
Page Number
466-469
Page Number
466-469
Publications
Publications
Topics
Article Type
Sections
Disallow All Ads
Content Gating
No Gating (article Unlocked/Free)
Alternative CME
Disqus Comments
Default
Article PDF Media

Malpractice Counsel: Never Too Young to Have a Heart Attack

Article Type
Changed
Wed, 12/12/2018 - 20:58
Display Headline
Malpractice Counsel: Never Too Young to Have a Heart Attack

Case

A 21-year-old woman presented to the ED for evaluation of severe chest pain radiating to her left arm, and associated shortness of breath, nausea, and vomiting. She stated that the pain started 2 hours earlier while she was resting and had become progressively worse. She denied any history of similar symptoms. The patient denied fever, chills, or cough. She stated that she was otherwise in good health and did not take any medication on a regular basis. Regarding her social history, she admitted to smoking one pack of cigarettes a day and drinking alcohol on occasion.

On physical examination, the patient appeared uncomfortable. Her vital signs were: blood pressure, 136/86 mm Hg; heart rate, 102 beats/min; respiratory rate, 22 breaths/min; and temperature, 98.60F. Oxygen saturation was 97% on room air. The head, eyes, ears, nose, and throat examination was unremarkable. Auscultation of the lungs revealed clear breath sounds bilaterally. The heart examination revealed tachycardia, but with regular rhythm and without murmurs, rubs, or gallops. The abdomen was soft and nontender. No lower extremity examination was documented.

The patient was seen by a physician assistant (PA) in the ED. An electrocardiogram (ECG), complete blood count (CBC), basic metabolic profile (BMP), troponin level, chest X-ray (CXR), and urine pregnancy test were ordered. The patient was given intravenous (IV) fluids and prochlorperazine 10 mg IV. The ECG and CXR were interpreted as normal. The urine pregnancy test was negative, and the remaining blood test results were within normal limits.

The PA believed the patient suffered from gastroenteritis, coupled with anxiety. He discharged the patient home with instructions to drink clear liquids for 24 hours, and take the prescribed prochlorperazine tablets as needed for continued nausea and vomiting.

At home, the patient continued to experience increasingly severe chest pain, shortness of breath, and vomiting. The next morning, she could no longer tolerate the pain and returned to the same ED via emergency medical services.

The patient’s history and physical examination remained unchanged from her presentation 16 hours earlier. At this ED visit, the patient was seen by an emergency physician (EP) who, concerned the patient had suffered an ischemic coronary event, ordered repeat ECG, CBC, BMP, and troponin evaluation. The EP also contacted cardiology services, but the cardiologist did not see the patient for several hours. When the cardiologist evaluated the patient and interpreted the ECG, he was concerned for an ST-segment elevation myocardial infarction (STEMI), and activated the catheterization lab.

Unfortunately, the patient had significant myocardial damage, with a resulting ejection fraction of only 10%. She was judged to be a candidate for heart transplantation, and received a left ventricular assist device (LVAD) as a bridge until a suitable donor heart could be identified. One month after implantation of the LVAD, the patient experienced an ischemic stroke that resulted in dense left-side weakness, leaving her confined to bed.

The patient sued the PA, the EP, the hospital, and the cardiologist for failing to identify and treat the acute STEMI in a timely manner. The plaintiff claimed the STEMI began at her first presentation to the ED, and that it should have been diagnosed and treated at that time. The plaintiff further argued that she should at least have been monitored and undergone repeat testing (ie, ECG and troponin level evaluation) at the first visit, stating that if she had received proper treatment, she would not have required an LVAD and therefore would not have had a stroke. The patient also alleged that at the second ED visit, there was a significant time delay before she was taken to the catheterization lab, which resulted in additional myocardial injury.

The defendants argued the patient was appropriately evaluated and treated at the first presentation, and that there was no evidence to suggest an MI. The EP argued that the delay in the patient’s care at the second visit was not his fault. All of the parties involved negotiated a settlement in the amount of $6 million in favor of the plaintiff.

Discussion

Myocardial infarction in adults younger than age 45 years is relatively rare, comprising only 2% to 10% of all MIs.1,2 The percentage of MI in patients younger than age 25 years must be even smaller, but no good data are available. In fact, age 40 years and younger is usually an exclusion criteria in many of the multicenter studies involving MI. Women are relatively spared from coronary artery disease (CAD) before menopause, thanks to the cardioprotective effects of estrogen. Young women who do experience an MI usually will have cardiovascular risk factors, especially smoking.

 

 

Risk Factors for MI in Young Patients

Cigarette Smoking. When examining common risk factors in young patients who had an MI (defined as patients younger than age 45 years), cigarette smoking is the most common risk factor.1,2 Between 76% and 91% of young patients with an MI are smokers, compared to only 40% incidence in older patients.1 It is thought that cigarette smoking produces endothelial dysfunction and can precipitate coronary spasm.1

Nonatherosclerotic Etiology. Interestingly, several studies of MI in young patients found a higher incidence of nonatherosclerotic causes of MI in women compared to men.2 One explanation for this finding is that women experience vasospastic syndromes and hypercoagulable states, secondary to oral contraceptive use or hereditary coagulation disorders.2 It has also been shown that young women have more active platelets following an MI and experience plaque erosions, rather than the plaque ruptures that occur in men and older women.2,3

Hyperlipidemia. Hyperlipidemia is an additional risk factor for MI in the younger adult patient population. In one study of young patients who had an MI, hyperlipidemia was the most important risk factor, in the absence of other obvious risk factors.1,4 In fact, some researchers think hyperlipidemia may be a more reliable predictor of MI in patients aged 30 to 39 years than in older patients.1,5 Unfortunately, many of these young adults are not aware that they have hyperlipidemia until they experience a complication such as an acute coronary syndrome. With respect to the patient in this case, it is not clear from the published report whether or not she had hyperlipidemia.

Family History. Another risk factor for MI in younger patients is a positive family history of CAD in a first-degree relative younger than age 55 years.1 Siblings of a young patient who experienced an MI have up to a 10-fold increase for developing CAD.1 It is currently not known why a positive family history increases the risk of MI in younger patients, but it may be related to inherited disorders of lipid metabolism, blood coagulation, or other genetic factors.1

Drug Abuse. Finally, drug abuse must be considered in young patients presenting with an MI. The use of cocaine, methamphetamine, marijuana, and K2 (synthetic marijuana) have all been associated with MI, especially in young patients,6-9 who typically do not have cardiac risk factors and do not show evidence of atherosclerotic disease on cardiac catheterization. As for the patient in this case, we do not know if she used any illicit drugs prior to presentation.

Summary

This case underscores the importance of not excluding MI in the differential diagnosis based simply on age or sex. While MI is uncommon in a 21-year-old woman, it can and does occur. In young patients presenting with chest pain, it is important to obtain a thorough history, including smoking, family history of MI, hyperlipidemia, and illicit drug use. While MI may be low on the differential diagnosis, it still needs to be considered.

References

1. Choudhury L, Marsh JD. Myocardial infarction in young patients. Am J Med. 1999;107(3):254-261.

2. Lawesson SS, Stenestrand U, Lagerqvist B, Wallentin L, Swahn E. Gender perspective on risk factors, coronary lesions and long-term outcome in young patients with ST-elevation myocardial infarction. Heart. 2010;96(6):453-459. doi:10.1136/hrt.2009.175463.

3. Burke AP, Farb A, Malcom G,Virmani R. Effect of menopause on plaque morphologic characteristics in coronary atherosclerosis. Am Heart J. 2001;141(2 Suppl):S58-S62.

4. Tomono S, Ohshima S, Murata K. The risk factors for ischemic heart disease in young adults. Jpn Circ J. 1990;54(4):436-441.

5. Gofman JW, Young W, Tandy R. Ischemic heart disease, atherosclerosis, and longevity. Circulation. 1966;34(4):679-697.

6. Zimmerman JL. Cocaine intoxication. Crit Care Clin. 2012;28(4):517-525. doi:10.1016/j.ccc.2012.07.003.

7. Hawley LA, Auten JD, Matteucci MJ, et al. Cardiac complications of adult methamphetamine exposures. J Emerg Med. 2013;45(6):821-827. doi:10.1016/j.jemermed.2013.04.061.

8. Bachs L, Mørland H. Acute cardiovascular fatalities following cannabis use. Forensic Sci Int. 2001;124(2-3):200-203.

9. Mir A, Obafemi A, Young A, Kane C. Myocardial infarction associated with use of the synthetic cannabinoid K2. Pediatrics. 2011;128(6):e1622-e1627. doi:10.1542 peds.2010-3823.

Article PDF
Author and Disclosure Information

Author’s Disclosure Statement: The author reports no actual or potential conflict of interest in relation to this article.

Issue
Emergency Medicine - 49(8)
Publications
Topics
Page Number
370-372
Sections
Author and Disclosure Information

Author’s Disclosure Statement: The author reports no actual or potential conflict of interest in relation to this article.

Author and Disclosure Information

Author’s Disclosure Statement: The author reports no actual or potential conflict of interest in relation to this article.

Article PDF
Article PDF

Case

A 21-year-old woman presented to the ED for evaluation of severe chest pain radiating to her left arm, and associated shortness of breath, nausea, and vomiting. She stated that the pain started 2 hours earlier while she was resting and had become progressively worse. She denied any history of similar symptoms. The patient denied fever, chills, or cough. She stated that she was otherwise in good health and did not take any medication on a regular basis. Regarding her social history, she admitted to smoking one pack of cigarettes a day and drinking alcohol on occasion.

On physical examination, the patient appeared uncomfortable. Her vital signs were: blood pressure, 136/86 mm Hg; heart rate, 102 beats/min; respiratory rate, 22 breaths/min; and temperature, 98.60F. Oxygen saturation was 97% on room air. The head, eyes, ears, nose, and throat examination was unremarkable. Auscultation of the lungs revealed clear breath sounds bilaterally. The heart examination revealed tachycardia, but with regular rhythm and without murmurs, rubs, or gallops. The abdomen was soft and nontender. No lower extremity examination was documented.

The patient was seen by a physician assistant (PA) in the ED. An electrocardiogram (ECG), complete blood count (CBC), basic metabolic profile (BMP), troponin level, chest X-ray (CXR), and urine pregnancy test were ordered. The patient was given intravenous (IV) fluids and prochlorperazine 10 mg IV. The ECG and CXR were interpreted as normal. The urine pregnancy test was negative, and the remaining blood test results were within normal limits.

The PA believed the patient suffered from gastroenteritis, coupled with anxiety. He discharged the patient home with instructions to drink clear liquids for 24 hours, and take the prescribed prochlorperazine tablets as needed for continued nausea and vomiting.

At home, the patient continued to experience increasingly severe chest pain, shortness of breath, and vomiting. The next morning, she could no longer tolerate the pain and returned to the same ED via emergency medical services.

The patient’s history and physical examination remained unchanged from her presentation 16 hours earlier. At this ED visit, the patient was seen by an emergency physician (EP) who, concerned the patient had suffered an ischemic coronary event, ordered repeat ECG, CBC, BMP, and troponin evaluation. The EP also contacted cardiology services, but the cardiologist did not see the patient for several hours. When the cardiologist evaluated the patient and interpreted the ECG, he was concerned for an ST-segment elevation myocardial infarction (STEMI), and activated the catheterization lab.

Unfortunately, the patient had significant myocardial damage, with a resulting ejection fraction of only 10%. She was judged to be a candidate for heart transplantation, and received a left ventricular assist device (LVAD) as a bridge until a suitable donor heart could be identified. One month after implantation of the LVAD, the patient experienced an ischemic stroke that resulted in dense left-side weakness, leaving her confined to bed.

The patient sued the PA, the EP, the hospital, and the cardiologist for failing to identify and treat the acute STEMI in a timely manner. The plaintiff claimed the STEMI began at her first presentation to the ED, and that it should have been diagnosed and treated at that time. The plaintiff further argued that she should at least have been monitored and undergone repeat testing (ie, ECG and troponin level evaluation) at the first visit, stating that if she had received proper treatment, she would not have required an LVAD and therefore would not have had a stroke. The patient also alleged that at the second ED visit, there was a significant time delay before she was taken to the catheterization lab, which resulted in additional myocardial injury.

The defendants argued the patient was appropriately evaluated and treated at the first presentation, and that there was no evidence to suggest an MI. The EP argued that the delay in the patient’s care at the second visit was not his fault. All of the parties involved negotiated a settlement in the amount of $6 million in favor of the plaintiff.

Discussion

Myocardial infarction in adults younger than age 45 years is relatively rare, comprising only 2% to 10% of all MIs.1,2 The percentage of MI in patients younger than age 25 years must be even smaller, but no good data are available. In fact, age 40 years and younger is usually an exclusion criteria in many of the multicenter studies involving MI. Women are relatively spared from coronary artery disease (CAD) before menopause, thanks to the cardioprotective effects of estrogen. Young women who do experience an MI usually will have cardiovascular risk factors, especially smoking.

 

 

Risk Factors for MI in Young Patients

Cigarette Smoking. When examining common risk factors in young patients who had an MI (defined as patients younger than age 45 years), cigarette smoking is the most common risk factor.1,2 Between 76% and 91% of young patients with an MI are smokers, compared to only 40% incidence in older patients.1 It is thought that cigarette smoking produces endothelial dysfunction and can precipitate coronary spasm.1

Nonatherosclerotic Etiology. Interestingly, several studies of MI in young patients found a higher incidence of nonatherosclerotic causes of MI in women compared to men.2 One explanation for this finding is that women experience vasospastic syndromes and hypercoagulable states, secondary to oral contraceptive use or hereditary coagulation disorders.2 It has also been shown that young women have more active platelets following an MI and experience plaque erosions, rather than the plaque ruptures that occur in men and older women.2,3

Hyperlipidemia. Hyperlipidemia is an additional risk factor for MI in the younger adult patient population. In one study of young patients who had an MI, hyperlipidemia was the most important risk factor, in the absence of other obvious risk factors.1,4 In fact, some researchers think hyperlipidemia may be a more reliable predictor of MI in patients aged 30 to 39 years than in older patients.1,5 Unfortunately, many of these young adults are not aware that they have hyperlipidemia until they experience a complication such as an acute coronary syndrome. With respect to the patient in this case, it is not clear from the published report whether or not she had hyperlipidemia.

Family History. Another risk factor for MI in younger patients is a positive family history of CAD in a first-degree relative younger than age 55 years.1 Siblings of a young patient who experienced an MI have up to a 10-fold increase for developing CAD.1 It is currently not known why a positive family history increases the risk of MI in younger patients, but it may be related to inherited disorders of lipid metabolism, blood coagulation, or other genetic factors.1

Drug Abuse. Finally, drug abuse must be considered in young patients presenting with an MI. The use of cocaine, methamphetamine, marijuana, and K2 (synthetic marijuana) have all been associated with MI, especially in young patients,6-9 who typically do not have cardiac risk factors and do not show evidence of atherosclerotic disease on cardiac catheterization. As for the patient in this case, we do not know if she used any illicit drugs prior to presentation.

Summary

This case underscores the importance of not excluding MI in the differential diagnosis based simply on age or sex. While MI is uncommon in a 21-year-old woman, it can and does occur. In young patients presenting with chest pain, it is important to obtain a thorough history, including smoking, family history of MI, hyperlipidemia, and illicit drug use. While MI may be low on the differential diagnosis, it still needs to be considered.

Case

A 21-year-old woman presented to the ED for evaluation of severe chest pain radiating to her left arm, and associated shortness of breath, nausea, and vomiting. She stated that the pain started 2 hours earlier while she was resting and had become progressively worse. She denied any history of similar symptoms. The patient denied fever, chills, or cough. She stated that she was otherwise in good health and did not take any medication on a regular basis. Regarding her social history, she admitted to smoking one pack of cigarettes a day and drinking alcohol on occasion.

On physical examination, the patient appeared uncomfortable. Her vital signs were: blood pressure, 136/86 mm Hg; heart rate, 102 beats/min; respiratory rate, 22 breaths/min; and temperature, 98.60F. Oxygen saturation was 97% on room air. The head, eyes, ears, nose, and throat examination was unremarkable. Auscultation of the lungs revealed clear breath sounds bilaterally. The heart examination revealed tachycardia, but with regular rhythm and without murmurs, rubs, or gallops. The abdomen was soft and nontender. No lower extremity examination was documented.

The patient was seen by a physician assistant (PA) in the ED. An electrocardiogram (ECG), complete blood count (CBC), basic metabolic profile (BMP), troponin level, chest X-ray (CXR), and urine pregnancy test were ordered. The patient was given intravenous (IV) fluids and prochlorperazine 10 mg IV. The ECG and CXR were interpreted as normal. The urine pregnancy test was negative, and the remaining blood test results were within normal limits.

The PA believed the patient suffered from gastroenteritis, coupled with anxiety. He discharged the patient home with instructions to drink clear liquids for 24 hours, and take the prescribed prochlorperazine tablets as needed for continued nausea and vomiting.

At home, the patient continued to experience increasingly severe chest pain, shortness of breath, and vomiting. The next morning, she could no longer tolerate the pain and returned to the same ED via emergency medical services.

The patient’s history and physical examination remained unchanged from her presentation 16 hours earlier. At this ED visit, the patient was seen by an emergency physician (EP) who, concerned the patient had suffered an ischemic coronary event, ordered repeat ECG, CBC, BMP, and troponin evaluation. The EP also contacted cardiology services, but the cardiologist did not see the patient for several hours. When the cardiologist evaluated the patient and interpreted the ECG, he was concerned for an ST-segment elevation myocardial infarction (STEMI), and activated the catheterization lab.

Unfortunately, the patient had significant myocardial damage, with a resulting ejection fraction of only 10%. She was judged to be a candidate for heart transplantation, and received a left ventricular assist device (LVAD) as a bridge until a suitable donor heart could be identified. One month after implantation of the LVAD, the patient experienced an ischemic stroke that resulted in dense left-side weakness, leaving her confined to bed.

The patient sued the PA, the EP, the hospital, and the cardiologist for failing to identify and treat the acute STEMI in a timely manner. The plaintiff claimed the STEMI began at her first presentation to the ED, and that it should have been diagnosed and treated at that time. The plaintiff further argued that she should at least have been monitored and undergone repeat testing (ie, ECG and troponin level evaluation) at the first visit, stating that if she had received proper treatment, she would not have required an LVAD and therefore would not have had a stroke. The patient also alleged that at the second ED visit, there was a significant time delay before she was taken to the catheterization lab, which resulted in additional myocardial injury.

The defendants argued the patient was appropriately evaluated and treated at the first presentation, and that there was no evidence to suggest an MI. The EP argued that the delay in the patient’s care at the second visit was not his fault. All of the parties involved negotiated a settlement in the amount of $6 million in favor of the plaintiff.

Discussion

Myocardial infarction in adults younger than age 45 years is relatively rare, comprising only 2% to 10% of all MIs.1,2 The percentage of MI in patients younger than age 25 years must be even smaller, but no good data are available. In fact, age 40 years and younger is usually an exclusion criteria in many of the multicenter studies involving MI. Women are relatively spared from coronary artery disease (CAD) before menopause, thanks to the cardioprotective effects of estrogen. Young women who do experience an MI usually will have cardiovascular risk factors, especially smoking.

 

 

Risk Factors for MI in Young Patients

Cigarette Smoking. When examining common risk factors in young patients who had an MI (defined as patients younger than age 45 years), cigarette smoking is the most common risk factor.1,2 Between 76% and 91% of young patients with an MI are smokers, compared to only 40% incidence in older patients.1 It is thought that cigarette smoking produces endothelial dysfunction and can precipitate coronary spasm.1

Nonatherosclerotic Etiology. Interestingly, several studies of MI in young patients found a higher incidence of nonatherosclerotic causes of MI in women compared to men.2 One explanation for this finding is that women experience vasospastic syndromes and hypercoagulable states, secondary to oral contraceptive use or hereditary coagulation disorders.2 It has also been shown that young women have more active platelets following an MI and experience plaque erosions, rather than the plaque ruptures that occur in men and older women.2,3

Hyperlipidemia. Hyperlipidemia is an additional risk factor for MI in the younger adult patient population. In one study of young patients who had an MI, hyperlipidemia was the most important risk factor, in the absence of other obvious risk factors.1,4 In fact, some researchers think hyperlipidemia may be a more reliable predictor of MI in patients aged 30 to 39 years than in older patients.1,5 Unfortunately, many of these young adults are not aware that they have hyperlipidemia until they experience a complication such as an acute coronary syndrome. With respect to the patient in this case, it is not clear from the published report whether or not she had hyperlipidemia.

Family History. Another risk factor for MI in younger patients is a positive family history of CAD in a first-degree relative younger than age 55 years.1 Siblings of a young patient who experienced an MI have up to a 10-fold increase for developing CAD.1 It is currently not known why a positive family history increases the risk of MI in younger patients, but it may be related to inherited disorders of lipid metabolism, blood coagulation, or other genetic factors.1

Drug Abuse. Finally, drug abuse must be considered in young patients presenting with an MI. The use of cocaine, methamphetamine, marijuana, and K2 (synthetic marijuana) have all been associated with MI, especially in young patients,6-9 who typically do not have cardiac risk factors and do not show evidence of atherosclerotic disease on cardiac catheterization. As for the patient in this case, we do not know if she used any illicit drugs prior to presentation.

Summary

This case underscores the importance of not excluding MI in the differential diagnosis based simply on age or sex. While MI is uncommon in a 21-year-old woman, it can and does occur. In young patients presenting with chest pain, it is important to obtain a thorough history, including smoking, family history of MI, hyperlipidemia, and illicit drug use. While MI may be low on the differential diagnosis, it still needs to be considered.

References

1. Choudhury L, Marsh JD. Myocardial infarction in young patients. Am J Med. 1999;107(3):254-261.

2. Lawesson SS, Stenestrand U, Lagerqvist B, Wallentin L, Swahn E. Gender perspective on risk factors, coronary lesions and long-term outcome in young patients with ST-elevation myocardial infarction. Heart. 2010;96(6):453-459. doi:10.1136/hrt.2009.175463.

3. Burke AP, Farb A, Malcom G,Virmani R. Effect of menopause on plaque morphologic characteristics in coronary atherosclerosis. Am Heart J. 2001;141(2 Suppl):S58-S62.

4. Tomono S, Ohshima S, Murata K. The risk factors for ischemic heart disease in young adults. Jpn Circ J. 1990;54(4):436-441.

5. Gofman JW, Young W, Tandy R. Ischemic heart disease, atherosclerosis, and longevity. Circulation. 1966;34(4):679-697.

6. Zimmerman JL. Cocaine intoxication. Crit Care Clin. 2012;28(4):517-525. doi:10.1016/j.ccc.2012.07.003.

7. Hawley LA, Auten JD, Matteucci MJ, et al. Cardiac complications of adult methamphetamine exposures. J Emerg Med. 2013;45(6):821-827. doi:10.1016/j.jemermed.2013.04.061.

8. Bachs L, Mørland H. Acute cardiovascular fatalities following cannabis use. Forensic Sci Int. 2001;124(2-3):200-203.

9. Mir A, Obafemi A, Young A, Kane C. Myocardial infarction associated with use of the synthetic cannabinoid K2. Pediatrics. 2011;128(6):e1622-e1627. doi:10.1542 peds.2010-3823.

References

1. Choudhury L, Marsh JD. Myocardial infarction in young patients. Am J Med. 1999;107(3):254-261.

2. Lawesson SS, Stenestrand U, Lagerqvist B, Wallentin L, Swahn E. Gender perspective on risk factors, coronary lesions and long-term outcome in young patients with ST-elevation myocardial infarction. Heart. 2010;96(6):453-459. doi:10.1136/hrt.2009.175463.

3. Burke AP, Farb A, Malcom G,Virmani R. Effect of menopause on plaque morphologic characteristics in coronary atherosclerosis. Am Heart J. 2001;141(2 Suppl):S58-S62.

4. Tomono S, Ohshima S, Murata K. The risk factors for ischemic heart disease in young adults. Jpn Circ J. 1990;54(4):436-441.

5. Gofman JW, Young W, Tandy R. Ischemic heart disease, atherosclerosis, and longevity. Circulation. 1966;34(4):679-697.

6. Zimmerman JL. Cocaine intoxication. Crit Care Clin. 2012;28(4):517-525. doi:10.1016/j.ccc.2012.07.003.

7. Hawley LA, Auten JD, Matteucci MJ, et al. Cardiac complications of adult methamphetamine exposures. J Emerg Med. 2013;45(6):821-827. doi:10.1016/j.jemermed.2013.04.061.

8. Bachs L, Mørland H. Acute cardiovascular fatalities following cannabis use. Forensic Sci Int. 2001;124(2-3):200-203.

9. Mir A, Obafemi A, Young A, Kane C. Myocardial infarction associated with use of the synthetic cannabinoid K2. Pediatrics. 2011;128(6):e1622-e1627. doi:10.1542 peds.2010-3823.

Issue
Emergency Medicine - 49(8)
Issue
Emergency Medicine - 49(8)
Page Number
370-372
Page Number
370-372
Publications
Publications
Topics
Article Type
Display Headline
Malpractice Counsel: Never Too Young to Have a Heart Attack
Display Headline
Malpractice Counsel: Never Too Young to Have a Heart Attack
Sections
Disallow All Ads
Content Gating
No Gating (article Unlocked/Free)
Alternative CME
Disqus Comments
Default
Article PDF Media

Malpractice Counsel: Missed Eye Injury: The Importance of the Visual Examination

Article Type
Changed
Wed, 12/12/2018 - 20:57
Display Headline
Malpractice Counsel: Missed Eye Injury: The Importance of the Visual Examination

Case

A 61-year-old woman presented to the ED for evaluation of left-side facial pain following a fall. The patient stated that she lost her balance as she was getting out of her car and fell to the ground, striking her left face and head. She denied any loss of consciousness, and complained of primarily left periorbital pain and swelling. She also denied neck or extremity pain, and was ambulatory after the fall. Her medical history was significant for hypertension and gastroesophageal reflux disease, for which she took medications. She admitted to a modest use of alcohol but denied tobacco use.

 

On physical examination, the patient’s vital signs were: blood pressure, 148/92 mm Hg; heart rate, 104 beats/min; respiratory rate, 18 breaths/min; and temperature, 98.8oF. Oxygen saturation was 98% on room air. Examination of the head and face revealed marked left periorbital bruising and swelling, and abrasions to the left forehead and anterior temporal area. The left eye was swollen shut. The right pupil was round and reactive to light, with intact extraocular muscle movement. The patient was tender to palpation around the left periorbital area, but not on any other areas of her face or cranium. The neck was nontender in the midline posteriorly, and the patient’s neurological examination was normal. Examination of the lungs, heart, and abdomen were likewise normal. No measurement of visual acuity was obtained.

The emergency physician (EP) ordered a computed tomography (CT) scan of the head and face without contrast. Because the patient could not remember the date of her last tetanus shot, a tetanus immunization was administered. The EP made several attempts to open the patient’s left eye to examine the pupil and anterior chamber, but was unable to do so because of the marked swelling and the patient’s discomfort.

Radiology services reported that the CT scan of the head was normal, while the CT scan of the face revealed a left orbital floor fracture. The patient was discharged home with instructions to place ice on the areas of swelling and to avoid blowing her nose. She was also given a prescription for hydrocodone/acetaminophen and instructed to follow-up with an ophthalmologist in 1 week.

Unfortunately, the patient suffered permanent and complete loss of sight in the left eye. She sued the hospital and the EP for failure to perform a complete physical examination and consult with an ophthalmologist to determine the extent of her injuries. In addition, an overread of the CT scan of the face revealed entrapment of the left inferior rectus muscle, which the original radiologist did not include in his report. The jury returned a defense verdict.

Discussion

This case is unfortunate because the critical injury, entrapment of the inferior rectus muscle, was missed by two physicians—the EP and the radiologist. While this injury can sometimes be detected on CT, most clinicians agree that orbital muscle entrapment is a clinical diagnosis. The most significant omission in this case is that the EP neither examined the affected eye nor tested the extraocular muscles. If the EP had done so, then in all likelihood this injury would have been identified and ophthalmology services would have been consulted.

Visual acuity should be considered a sixth vital sign in patients who present with an eye injury. This test can be performed using a wall, pocket, or mobile-app Snellen chart.1 If the patient is unable to perform an eye examination, the EP should assess for light and color perception.1 A complete loss of vision implies injury to the optic nerve or globe.1

When possible, it is best to attempt to examine the eyes prior to the onset of significant eyelid swelling. In the presence of significant swelling, lid retractors (eg, paper clip retractors) can be used to allow proper examination of the eye. The pupil, sclera, anterior chamber, and eye movement should all be assessed. Limited vertical movement of the globe, vertical diplopia, and pain in the inferior orbit on attempted vertical movement are consistent with entrapment of the inferior rectus muscle.2 The presence of enophthalmos (posterior displacement of the globe within the orbit) and globe ptosis (downward displacement of the globe within the orbit) should be noted because these often indicate a significant fracture.2

The majority of orbital floor fractures do not require surgical repair. Most are followed for 5 to 10 days to allow swelling and orbital hemorrhage to subside.2 Prednisone (1 mg/kg/d for 7 days) can decrease edema and may limit the risk of diplopia from inferior rectus muscle contractions and fibrosis. However, the presence of tight entrapment of the inferior rectus muscle, or CT scan demonstration of the inferior rectus muscle within the maxillary sinus, is an indication for immediate surgical intervention.2

As physicians, it is imperative that we thoroughly examine the area of primary complaint which, as this case demonstrates, is not always easy.

 

 

References

1. Walker RA, Adhikari S. Eye emergencies. In: Tintinalli JE, Stapczynski JS, Ma OJ, Yealy DM, Meckler GD, Cline DM, eds. Tintinalli’s Emergency Medicine: A Comprehensive Study Guide. 8th ed. New York, NY: McGraw Hill; 2016:1543-1569.

2. American Academy of Ophthalmology. Orbital floor fractures. www.aao.org/bcscsnippetdetail.aspx?id=415cdb9b-3308-4f90-bc33-dac1ebd676ce. Accessed May 3, 2017.

Article PDF
Author and Disclosure Information

Author’s Disclosure Statement: The author reports no actual or potential conflict of interest in relation to this article.

Issue
Emergency Medicine 49(6)
Publications
Topics
Page Number
270-271
Sections
Author and Disclosure Information

Author’s Disclosure Statement: The author reports no actual or potential conflict of interest in relation to this article.

Author and Disclosure Information

Author’s Disclosure Statement: The author reports no actual or potential conflict of interest in relation to this article.

Article PDF
Article PDF

Case

A 61-year-old woman presented to the ED for evaluation of left-side facial pain following a fall. The patient stated that she lost her balance as she was getting out of her car and fell to the ground, striking her left face and head. She denied any loss of consciousness, and complained of primarily left periorbital pain and swelling. She also denied neck or extremity pain, and was ambulatory after the fall. Her medical history was significant for hypertension and gastroesophageal reflux disease, for which she took medications. She admitted to a modest use of alcohol but denied tobacco use.

 

On physical examination, the patient’s vital signs were: blood pressure, 148/92 mm Hg; heart rate, 104 beats/min; respiratory rate, 18 breaths/min; and temperature, 98.8oF. Oxygen saturation was 98% on room air. Examination of the head and face revealed marked left periorbital bruising and swelling, and abrasions to the left forehead and anterior temporal area. The left eye was swollen shut. The right pupil was round and reactive to light, with intact extraocular muscle movement. The patient was tender to palpation around the left periorbital area, but not on any other areas of her face or cranium. The neck was nontender in the midline posteriorly, and the patient’s neurological examination was normal. Examination of the lungs, heart, and abdomen were likewise normal. No measurement of visual acuity was obtained.

The emergency physician (EP) ordered a computed tomography (CT) scan of the head and face without contrast. Because the patient could not remember the date of her last tetanus shot, a tetanus immunization was administered. The EP made several attempts to open the patient’s left eye to examine the pupil and anterior chamber, but was unable to do so because of the marked swelling and the patient’s discomfort.

Radiology services reported that the CT scan of the head was normal, while the CT scan of the face revealed a left orbital floor fracture. The patient was discharged home with instructions to place ice on the areas of swelling and to avoid blowing her nose. She was also given a prescription for hydrocodone/acetaminophen and instructed to follow-up with an ophthalmologist in 1 week.

Unfortunately, the patient suffered permanent and complete loss of sight in the left eye. She sued the hospital and the EP for failure to perform a complete physical examination and consult with an ophthalmologist to determine the extent of her injuries. In addition, an overread of the CT scan of the face revealed entrapment of the left inferior rectus muscle, which the original radiologist did not include in his report. The jury returned a defense verdict.

Discussion

This case is unfortunate because the critical injury, entrapment of the inferior rectus muscle, was missed by two physicians—the EP and the radiologist. While this injury can sometimes be detected on CT, most clinicians agree that orbital muscle entrapment is a clinical diagnosis. The most significant omission in this case is that the EP neither examined the affected eye nor tested the extraocular muscles. If the EP had done so, then in all likelihood this injury would have been identified and ophthalmology services would have been consulted.

Visual acuity should be considered a sixth vital sign in patients who present with an eye injury. This test can be performed using a wall, pocket, or mobile-app Snellen chart.1 If the patient is unable to perform an eye examination, the EP should assess for light and color perception.1 A complete loss of vision implies injury to the optic nerve or globe.1

When possible, it is best to attempt to examine the eyes prior to the onset of significant eyelid swelling. In the presence of significant swelling, lid retractors (eg, paper clip retractors) can be used to allow proper examination of the eye. The pupil, sclera, anterior chamber, and eye movement should all be assessed. Limited vertical movement of the globe, vertical diplopia, and pain in the inferior orbit on attempted vertical movement are consistent with entrapment of the inferior rectus muscle.2 The presence of enophthalmos (posterior displacement of the globe within the orbit) and globe ptosis (downward displacement of the globe within the orbit) should be noted because these often indicate a significant fracture.2

The majority of orbital floor fractures do not require surgical repair. Most are followed for 5 to 10 days to allow swelling and orbital hemorrhage to subside.2 Prednisone (1 mg/kg/d for 7 days) can decrease edema and may limit the risk of diplopia from inferior rectus muscle contractions and fibrosis. However, the presence of tight entrapment of the inferior rectus muscle, or CT scan demonstration of the inferior rectus muscle within the maxillary sinus, is an indication for immediate surgical intervention.2

As physicians, it is imperative that we thoroughly examine the area of primary complaint which, as this case demonstrates, is not always easy.

 

 

Case

A 61-year-old woman presented to the ED for evaluation of left-side facial pain following a fall. The patient stated that she lost her balance as she was getting out of her car and fell to the ground, striking her left face and head. She denied any loss of consciousness, and complained of primarily left periorbital pain and swelling. She also denied neck or extremity pain, and was ambulatory after the fall. Her medical history was significant for hypertension and gastroesophageal reflux disease, for which she took medications. She admitted to a modest use of alcohol but denied tobacco use.

 

On physical examination, the patient’s vital signs were: blood pressure, 148/92 mm Hg; heart rate, 104 beats/min; respiratory rate, 18 breaths/min; and temperature, 98.8oF. Oxygen saturation was 98% on room air. Examination of the head and face revealed marked left periorbital bruising and swelling, and abrasions to the left forehead and anterior temporal area. The left eye was swollen shut. The right pupil was round and reactive to light, with intact extraocular muscle movement. The patient was tender to palpation around the left periorbital area, but not on any other areas of her face or cranium. The neck was nontender in the midline posteriorly, and the patient’s neurological examination was normal. Examination of the lungs, heart, and abdomen were likewise normal. No measurement of visual acuity was obtained.

The emergency physician (EP) ordered a computed tomography (CT) scan of the head and face without contrast. Because the patient could not remember the date of her last tetanus shot, a tetanus immunization was administered. The EP made several attempts to open the patient’s left eye to examine the pupil and anterior chamber, but was unable to do so because of the marked swelling and the patient’s discomfort.

Radiology services reported that the CT scan of the head was normal, while the CT scan of the face revealed a left orbital floor fracture. The patient was discharged home with instructions to place ice on the areas of swelling and to avoid blowing her nose. She was also given a prescription for hydrocodone/acetaminophen and instructed to follow-up with an ophthalmologist in 1 week.

Unfortunately, the patient suffered permanent and complete loss of sight in the left eye. She sued the hospital and the EP for failure to perform a complete physical examination and consult with an ophthalmologist to determine the extent of her injuries. In addition, an overread of the CT scan of the face revealed entrapment of the left inferior rectus muscle, which the original radiologist did not include in his report. The jury returned a defense verdict.

Discussion

This case is unfortunate because the critical injury, entrapment of the inferior rectus muscle, was missed by two physicians—the EP and the radiologist. While this injury can sometimes be detected on CT, most clinicians agree that orbital muscle entrapment is a clinical diagnosis. The most significant omission in this case is that the EP neither examined the affected eye nor tested the extraocular muscles. If the EP had done so, then in all likelihood this injury would have been identified and ophthalmology services would have been consulted.

Visual acuity should be considered a sixth vital sign in patients who present with an eye injury. This test can be performed using a wall, pocket, or mobile-app Snellen chart.1 If the patient is unable to perform an eye examination, the EP should assess for light and color perception.1 A complete loss of vision implies injury to the optic nerve or globe.1

When possible, it is best to attempt to examine the eyes prior to the onset of significant eyelid swelling. In the presence of significant swelling, lid retractors (eg, paper clip retractors) can be used to allow proper examination of the eye. The pupil, sclera, anterior chamber, and eye movement should all be assessed. Limited vertical movement of the globe, vertical diplopia, and pain in the inferior orbit on attempted vertical movement are consistent with entrapment of the inferior rectus muscle.2 The presence of enophthalmos (posterior displacement of the globe within the orbit) and globe ptosis (downward displacement of the globe within the orbit) should be noted because these often indicate a significant fracture.2

The majority of orbital floor fractures do not require surgical repair. Most are followed for 5 to 10 days to allow swelling and orbital hemorrhage to subside.2 Prednisone (1 mg/kg/d for 7 days) can decrease edema and may limit the risk of diplopia from inferior rectus muscle contractions and fibrosis. However, the presence of tight entrapment of the inferior rectus muscle, or CT scan demonstration of the inferior rectus muscle within the maxillary sinus, is an indication for immediate surgical intervention.2

As physicians, it is imperative that we thoroughly examine the area of primary complaint which, as this case demonstrates, is not always easy.

 

 

References

1. Walker RA, Adhikari S. Eye emergencies. In: Tintinalli JE, Stapczynski JS, Ma OJ, Yealy DM, Meckler GD, Cline DM, eds. Tintinalli’s Emergency Medicine: A Comprehensive Study Guide. 8th ed. New York, NY: McGraw Hill; 2016:1543-1569.

2. American Academy of Ophthalmology. Orbital floor fractures. www.aao.org/bcscsnippetdetail.aspx?id=415cdb9b-3308-4f90-bc33-dac1ebd676ce. Accessed May 3, 2017.

References

1. Walker RA, Adhikari S. Eye emergencies. In: Tintinalli JE, Stapczynski JS, Ma OJ, Yealy DM, Meckler GD, Cline DM, eds. Tintinalli’s Emergency Medicine: A Comprehensive Study Guide. 8th ed. New York, NY: McGraw Hill; 2016:1543-1569.

2. American Academy of Ophthalmology. Orbital floor fractures. www.aao.org/bcscsnippetdetail.aspx?id=415cdb9b-3308-4f90-bc33-dac1ebd676ce. Accessed May 3, 2017.

Issue
Emergency Medicine 49(6)
Issue
Emergency Medicine 49(6)
Page Number
270-271
Page Number
270-271
Publications
Publications
Topics
Article Type
Display Headline
Malpractice Counsel: Missed Eye Injury: The Importance of the Visual Examination
Display Headline
Malpractice Counsel: Missed Eye Injury: The Importance of the Visual Examination
Sections
Disallow All Ads
Content Gating
No Gating (article Unlocked/Free)
Alternative CME
Article PDF Media

Malpractice Counsel: The Challenges of Cardioversion

Article Type
Changed
Wed, 12/12/2018 - 20:56
Display Headline
Malpractice Counsel: The Challenges of Cardioversion

Case

A 56-year-old woman presented to the ED with palpitations and lightheadedness, which began upon awakening that morning. The patient had a history of atrial fibrillation (AF), and believed this was the cause of her symptoms. Over the past 18 months, the patient had twice undergone successful cardioversion for AF with a rapid ventricular response (RVR); both cardioversions were performed by her cardiologist.

The patient denied experiencing any chest pain, shortness of breath, nausea, or vomiting. Her medical history was significant only for AF. Regarding her medication history, the patient had been prescribed metoprolol, but admitted that she frequently forgot to take it. She further stated that she was not taking aspirin or anticoagulation therapy for AF. She denied past or current alcohol consumption or tobacco use.

On physical examination, the patient’s vital signs were: heart rate (HR), 186 beats/min; blood pressure, 137/82 mm Hg; respiratory rate, 20 breaths/min, and temperature, afebrile. Oxygen saturation was 96% on room air. The head, eye, ears, nose, and throat examination was normal. Auscultation of the lungs revealed clear breath sounds bilaterally. On examination of the heart, the patient had an irregularly irregular rhythm that was tachycardic; no murmurs, rubs, or gallops were appreciated. The abdomen was soft and nontender. There was no edema or redness of the lower extremities.

The emergency physician (EP) placed the patient on a cardiac monitor and administered 2 L of oxygen via nasal cannula. An electrocardiogram (ECG), portable chest X-ray (CXR), and laboratory evaluation were ordered, and an intravenous (IV) line was established. The ECG revealed AF with RVR, without evidence of ischemia. The CXR was interpreted as normal. Laboratory studies, including complete blood count, basic metabolic profile, and serum troponin levels, were likewise within normal limits.

Based on the patient’s history and evaluation, the EP decided to cardiovert the patient rather than attempt rate control with IV medications. The patient consented to the cardioversion, based on the two previous successful cardioversions performed by her cardiologist. The EP gave the patient midazolam 2 mg IV and performed synchronized cardioversion at 200 joules. The patient converted to normal sinus rhythm with an HR of 86 beats/min. She was observed in the ED for 1 hour, given metoprolol 50 mg by mouth, and discharged home with instructions to follow up with her cardiologist the following week.

The next day, the patient suffered a large ischemic stroke in the distribution of the left middle cerebral artery, resulting in a dense right hemiparesis. The neurological deficit was significant, necessitating the patient’s placement in a nursing home.

The patient and her family sued the EP for malpractice for not anticoagulating the patient prior to and following cardioversion. A $3.3 million settlement was agreed upon prior to trial.

Discussion

Patients commonly present to the ED for complaints related to AF. In some cases, the EP is the first to diagnose the patient’s AF; in other cases, the patient has a history of AF and is presenting with a complication. The focus of this discussion is solely on the management of AF with RVR.

When managing a patient in AF with RVR, the EP must consider three issues: ventricular rate control (VRC), rhythm control, and anticoagulation. Selecting the best treatment strategy will depend on the patient’s hemodynamic stability, duration of her or his symptoms, local custom and preference, and the length of time the AF has been present.

Ventricular Rate Control and Cardioversion

For many stable patients, VRC is frequently the treatment of choice, with a goal HR of less than 100 beats/min. Intravenous diltiazem, esmolol, or metoprolol can be used to achieve VRC in patients in AF. Because these drugs only control ventricular rate and do not typically cardiovert, the risk of embolization is small.

Synchronized cardioversion has the benefit of providing both rate and rhythm control, but at the expense of the increased risk of arterial embolization. Some patients, including those with rheumatic heart disease, mitral stenosis, prosthetic heart valves, severe left ventricular dysfunction, or a history of thromboembolism, are at a constant high risk of developing a thromboembolism.1

Risk-Benefit Ratio and Anticoagulation Therapy

To help determine the risk-benefit ratio in patients without the risk factors mentioned above, the EP should calculate the CHADS2 (congestive heart failure [CHF], hypertension, age, diabetes mellitus [DM], prior stroke, transient ischemic attack [TIA], or thromboembolism [doubled]) score or CHA2DS2-VASC (CHF, hypertension, age 75 years or older [two scores], DM, previous stroke, TIA, or thromboembolism [doubled], vascular disease, age 65-74 years, sex [female]) score to help identify patients at risk for arterial embolic complications (Table).

Table.
Table.

 

 

For patients who have been in AF for less than 48 hours and who are at a very low-embolic risk (CHA2DS2-VASC score of 0), some experts suggest cardioversion without anticoagulation. However, other experts recommend anticoagulation prior to cardioversion—even in low-risk patients. Unfortunately, there is disagreement between professional organizations, with the American Heart Association/American College of Cardiology/Heart Rhythm Society stating that cardioversion may be performed with or without procedural anticoagulation,2 while the 2016 European Society of Cardiology guidelines recommend immediate initiation of anticoagulants in all such patients scheduled for cardioversion.3

The reasoning in favor of anticoagulation prior to cardioversion is supported by an observational study by Airaksinen et al4 of 2,481 patients undergoing cardioversion for AF of less than 48 hours duration. This study demonstrated a definite thromboembolic event in 38 (0.7%) of the patients within 30 days (median of 2 days). The thromboembolic event was stroke in 31 of the 38 patients.4 Airaksinen et al4 found that age older than 60 years, female sex, heart failure (HF), and DM were the strongest predictors of embolization. The risk of stroke in patients without HF and those younger than age 60 years was only 0.2%.4

In a similar observational study by Hansen et al5 of 16,274 patients in AF undergoing cardioversion with and without anticoagulation therapy, the absence of post­cardio­version anticoagulation increased the risk of thromboembolism 2-fold—regardless of CHA2DS2-VASC scores.

Summary

While the management of AF with a duration of more than 48 hours should always include some type of anticoagulation therapy (pre- or postcardioversion, or both), the role of anticoagulation in low-risk patients with AF of less than 48 hours is not as clear. As this situation is not uncommon, the emergency medicine and cardiology physicians should consider developing a mutually agreed upon protocol on how best to manage these patients at their institution. When considering cardioversion without pre- or postanticoagulation in low-risk patients with AF, EPs should always involve the patient in the decision-making process.

References

1. Phang R, Manning WJ. Prevention of embolization prior to and after restoration of sinus rhythm in atrial fibrillation. UptoDate Web site. http://www.uptodate.com/contents/prevention-of-embolization-prior-to-and-after-restoration-of-sinus-rhythm-in-atrial-fibrillation. Updated October 10, 2016. Accessed March 6, 2017.
2. January CT, Wann LS, Alpert JS, et al; ACC/AHA Task Force Members. 2014 AHA/ACC/HRS guideline for the management of patients with atrial fibrillation: a report of the American College of Cardiology/American Heart Association Task Force on practice guidelines and the Heart Rhythm Society. Circulation. 2014;130(23):e199-e267. Erratum in Circulation. 2014;130(23):e272-e274. doi:10.1161/CIR.0000000000000041.
3. Kirchhof P, Benussi S, Kotecha D, et al. 2016 ESC Guidelines for the management of atrial fibrillation developed in collaboration with EACTS. Eur Heart J. 2016:37(38):2893-2962. doi:10.1093/eurheartj/ehw210.
4. Airaksinen KE, Grönberg T, Nuotio I, et al. Thromboembolic complications after cardioversion of acute atrial fibrillation: the FinCV (Finnish CardioVersion) study. J Am Coll Cardiol. 2013;62(13):1187-1192. doi:10.1016/j.jacc.2013.04.089.
5. Hansen ML, Jepsen RM, Olesen JB, et al. Thromboembolic risk in 16 274 atrial fibrillation patients undergoing direct current cardioversion with and without oral anticoagulant therapy. Europace. 2015;17(1):18-23. doi:10.1093/europace/euu189.

Article PDF
Author and Disclosure Information

Author’s Disclosure Statement: The author reports no actual or potential conflict of interest in relation to this article.

Issue
Emergency Medicine - 49(4)
Publications
Topics
Page Number
172-174
Sections
Author and Disclosure Information

Author’s Disclosure Statement: The author reports no actual or potential conflict of interest in relation to this article.

Author and Disclosure Information

Author’s Disclosure Statement: The author reports no actual or potential conflict of interest in relation to this article.

Article PDF
Article PDF

Case

A 56-year-old woman presented to the ED with palpitations and lightheadedness, which began upon awakening that morning. The patient had a history of atrial fibrillation (AF), and believed this was the cause of her symptoms. Over the past 18 months, the patient had twice undergone successful cardioversion for AF with a rapid ventricular response (RVR); both cardioversions were performed by her cardiologist.

The patient denied experiencing any chest pain, shortness of breath, nausea, or vomiting. Her medical history was significant only for AF. Regarding her medication history, the patient had been prescribed metoprolol, but admitted that she frequently forgot to take it. She further stated that she was not taking aspirin or anticoagulation therapy for AF. She denied past or current alcohol consumption or tobacco use.

On physical examination, the patient’s vital signs were: heart rate (HR), 186 beats/min; blood pressure, 137/82 mm Hg; respiratory rate, 20 breaths/min, and temperature, afebrile. Oxygen saturation was 96% on room air. The head, eye, ears, nose, and throat examination was normal. Auscultation of the lungs revealed clear breath sounds bilaterally. On examination of the heart, the patient had an irregularly irregular rhythm that was tachycardic; no murmurs, rubs, or gallops were appreciated. The abdomen was soft and nontender. There was no edema or redness of the lower extremities.

The emergency physician (EP) placed the patient on a cardiac monitor and administered 2 L of oxygen via nasal cannula. An electrocardiogram (ECG), portable chest X-ray (CXR), and laboratory evaluation were ordered, and an intravenous (IV) line was established. The ECG revealed AF with RVR, without evidence of ischemia. The CXR was interpreted as normal. Laboratory studies, including complete blood count, basic metabolic profile, and serum troponin levels, were likewise within normal limits.

Based on the patient’s history and evaluation, the EP decided to cardiovert the patient rather than attempt rate control with IV medications. The patient consented to the cardioversion, based on the two previous successful cardioversions performed by her cardiologist. The EP gave the patient midazolam 2 mg IV and performed synchronized cardioversion at 200 joules. The patient converted to normal sinus rhythm with an HR of 86 beats/min. She was observed in the ED for 1 hour, given metoprolol 50 mg by mouth, and discharged home with instructions to follow up with her cardiologist the following week.

The next day, the patient suffered a large ischemic stroke in the distribution of the left middle cerebral artery, resulting in a dense right hemiparesis. The neurological deficit was significant, necessitating the patient’s placement in a nursing home.

The patient and her family sued the EP for malpractice for not anticoagulating the patient prior to and following cardioversion. A $3.3 million settlement was agreed upon prior to trial.

Discussion

Patients commonly present to the ED for complaints related to AF. In some cases, the EP is the first to diagnose the patient’s AF; in other cases, the patient has a history of AF and is presenting with a complication. The focus of this discussion is solely on the management of AF with RVR.

When managing a patient in AF with RVR, the EP must consider three issues: ventricular rate control (VRC), rhythm control, and anticoagulation. Selecting the best treatment strategy will depend on the patient’s hemodynamic stability, duration of her or his symptoms, local custom and preference, and the length of time the AF has been present.

Ventricular Rate Control and Cardioversion

For many stable patients, VRC is frequently the treatment of choice, with a goal HR of less than 100 beats/min. Intravenous diltiazem, esmolol, or metoprolol can be used to achieve VRC in patients in AF. Because these drugs only control ventricular rate and do not typically cardiovert, the risk of embolization is small.

Synchronized cardioversion has the benefit of providing both rate and rhythm control, but at the expense of the increased risk of arterial embolization. Some patients, including those with rheumatic heart disease, mitral stenosis, prosthetic heart valves, severe left ventricular dysfunction, or a history of thromboembolism, are at a constant high risk of developing a thromboembolism.1

Risk-Benefit Ratio and Anticoagulation Therapy

To help determine the risk-benefit ratio in patients without the risk factors mentioned above, the EP should calculate the CHADS2 (congestive heart failure [CHF], hypertension, age, diabetes mellitus [DM], prior stroke, transient ischemic attack [TIA], or thromboembolism [doubled]) score or CHA2DS2-VASC (CHF, hypertension, age 75 years or older [two scores], DM, previous stroke, TIA, or thromboembolism [doubled], vascular disease, age 65-74 years, sex [female]) score to help identify patients at risk for arterial embolic complications (Table).

Table.
Table.

 

 

For patients who have been in AF for less than 48 hours and who are at a very low-embolic risk (CHA2DS2-VASC score of 0), some experts suggest cardioversion without anticoagulation. However, other experts recommend anticoagulation prior to cardioversion—even in low-risk patients. Unfortunately, there is disagreement between professional organizations, with the American Heart Association/American College of Cardiology/Heart Rhythm Society stating that cardioversion may be performed with or without procedural anticoagulation,2 while the 2016 European Society of Cardiology guidelines recommend immediate initiation of anticoagulants in all such patients scheduled for cardioversion.3

The reasoning in favor of anticoagulation prior to cardioversion is supported by an observational study by Airaksinen et al4 of 2,481 patients undergoing cardioversion for AF of less than 48 hours duration. This study demonstrated a definite thromboembolic event in 38 (0.7%) of the patients within 30 days (median of 2 days). The thromboembolic event was stroke in 31 of the 38 patients.4 Airaksinen et al4 found that age older than 60 years, female sex, heart failure (HF), and DM were the strongest predictors of embolization. The risk of stroke in patients without HF and those younger than age 60 years was only 0.2%.4

In a similar observational study by Hansen et al5 of 16,274 patients in AF undergoing cardioversion with and without anticoagulation therapy, the absence of post­cardio­version anticoagulation increased the risk of thromboembolism 2-fold—regardless of CHA2DS2-VASC scores.

Summary

While the management of AF with a duration of more than 48 hours should always include some type of anticoagulation therapy (pre- or postcardioversion, or both), the role of anticoagulation in low-risk patients with AF of less than 48 hours is not as clear. As this situation is not uncommon, the emergency medicine and cardiology physicians should consider developing a mutually agreed upon protocol on how best to manage these patients at their institution. When considering cardioversion without pre- or postanticoagulation in low-risk patients with AF, EPs should always involve the patient in the decision-making process.

Case

A 56-year-old woman presented to the ED with palpitations and lightheadedness, which began upon awakening that morning. The patient had a history of atrial fibrillation (AF), and believed this was the cause of her symptoms. Over the past 18 months, the patient had twice undergone successful cardioversion for AF with a rapid ventricular response (RVR); both cardioversions were performed by her cardiologist.

The patient denied experiencing any chest pain, shortness of breath, nausea, or vomiting. Her medical history was significant only for AF. Regarding her medication history, the patient had been prescribed metoprolol, but admitted that she frequently forgot to take it. She further stated that she was not taking aspirin or anticoagulation therapy for AF. She denied past or current alcohol consumption or tobacco use.

On physical examination, the patient’s vital signs were: heart rate (HR), 186 beats/min; blood pressure, 137/82 mm Hg; respiratory rate, 20 breaths/min, and temperature, afebrile. Oxygen saturation was 96% on room air. The head, eye, ears, nose, and throat examination was normal. Auscultation of the lungs revealed clear breath sounds bilaterally. On examination of the heart, the patient had an irregularly irregular rhythm that was tachycardic; no murmurs, rubs, or gallops were appreciated. The abdomen was soft and nontender. There was no edema or redness of the lower extremities.

The emergency physician (EP) placed the patient on a cardiac monitor and administered 2 L of oxygen via nasal cannula. An electrocardiogram (ECG), portable chest X-ray (CXR), and laboratory evaluation were ordered, and an intravenous (IV) line was established. The ECG revealed AF with RVR, without evidence of ischemia. The CXR was interpreted as normal. Laboratory studies, including complete blood count, basic metabolic profile, and serum troponin levels, were likewise within normal limits.

Based on the patient’s history and evaluation, the EP decided to cardiovert the patient rather than attempt rate control with IV medications. The patient consented to the cardioversion, based on the two previous successful cardioversions performed by her cardiologist. The EP gave the patient midazolam 2 mg IV and performed synchronized cardioversion at 200 joules. The patient converted to normal sinus rhythm with an HR of 86 beats/min. She was observed in the ED for 1 hour, given metoprolol 50 mg by mouth, and discharged home with instructions to follow up with her cardiologist the following week.

The next day, the patient suffered a large ischemic stroke in the distribution of the left middle cerebral artery, resulting in a dense right hemiparesis. The neurological deficit was significant, necessitating the patient’s placement in a nursing home.

The patient and her family sued the EP for malpractice for not anticoagulating the patient prior to and following cardioversion. A $3.3 million settlement was agreed upon prior to trial.

Discussion

Patients commonly present to the ED for complaints related to AF. In some cases, the EP is the first to diagnose the patient’s AF; in other cases, the patient has a history of AF and is presenting with a complication. The focus of this discussion is solely on the management of AF with RVR.

When managing a patient in AF with RVR, the EP must consider three issues: ventricular rate control (VRC), rhythm control, and anticoagulation. Selecting the best treatment strategy will depend on the patient’s hemodynamic stability, duration of her or his symptoms, local custom and preference, and the length of time the AF has been present.

Ventricular Rate Control and Cardioversion

For many stable patients, VRC is frequently the treatment of choice, with a goal HR of less than 100 beats/min. Intravenous diltiazem, esmolol, or metoprolol can be used to achieve VRC in patients in AF. Because these drugs only control ventricular rate and do not typically cardiovert, the risk of embolization is small.

Synchronized cardioversion has the benefit of providing both rate and rhythm control, but at the expense of the increased risk of arterial embolization. Some patients, including those with rheumatic heart disease, mitral stenosis, prosthetic heart valves, severe left ventricular dysfunction, or a history of thromboembolism, are at a constant high risk of developing a thromboembolism.1

Risk-Benefit Ratio and Anticoagulation Therapy

To help determine the risk-benefit ratio in patients without the risk factors mentioned above, the EP should calculate the CHADS2 (congestive heart failure [CHF], hypertension, age, diabetes mellitus [DM], prior stroke, transient ischemic attack [TIA], or thromboembolism [doubled]) score or CHA2DS2-VASC (CHF, hypertension, age 75 years or older [two scores], DM, previous stroke, TIA, or thromboembolism [doubled], vascular disease, age 65-74 years, sex [female]) score to help identify patients at risk for arterial embolic complications (Table).

Table.
Table.

 

 

For patients who have been in AF for less than 48 hours and who are at a very low-embolic risk (CHA2DS2-VASC score of 0), some experts suggest cardioversion without anticoagulation. However, other experts recommend anticoagulation prior to cardioversion—even in low-risk patients. Unfortunately, there is disagreement between professional organizations, with the American Heart Association/American College of Cardiology/Heart Rhythm Society stating that cardioversion may be performed with or without procedural anticoagulation,2 while the 2016 European Society of Cardiology guidelines recommend immediate initiation of anticoagulants in all such patients scheduled for cardioversion.3

The reasoning in favor of anticoagulation prior to cardioversion is supported by an observational study by Airaksinen et al4 of 2,481 patients undergoing cardioversion for AF of less than 48 hours duration. This study demonstrated a definite thromboembolic event in 38 (0.7%) of the patients within 30 days (median of 2 days). The thromboembolic event was stroke in 31 of the 38 patients.4 Airaksinen et al4 found that age older than 60 years, female sex, heart failure (HF), and DM were the strongest predictors of embolization. The risk of stroke in patients without HF and those younger than age 60 years was only 0.2%.4

In a similar observational study by Hansen et al5 of 16,274 patients in AF undergoing cardioversion with and without anticoagulation therapy, the absence of post­cardio­version anticoagulation increased the risk of thromboembolism 2-fold—regardless of CHA2DS2-VASC scores.

Summary

While the management of AF with a duration of more than 48 hours should always include some type of anticoagulation therapy (pre- or postcardioversion, or both), the role of anticoagulation in low-risk patients with AF of less than 48 hours is not as clear. As this situation is not uncommon, the emergency medicine and cardiology physicians should consider developing a mutually agreed upon protocol on how best to manage these patients at their institution. When considering cardioversion without pre- or postanticoagulation in low-risk patients with AF, EPs should always involve the patient in the decision-making process.

References

1. Phang R, Manning WJ. Prevention of embolization prior to and after restoration of sinus rhythm in atrial fibrillation. UptoDate Web site. http://www.uptodate.com/contents/prevention-of-embolization-prior-to-and-after-restoration-of-sinus-rhythm-in-atrial-fibrillation. Updated October 10, 2016. Accessed March 6, 2017.
2. January CT, Wann LS, Alpert JS, et al; ACC/AHA Task Force Members. 2014 AHA/ACC/HRS guideline for the management of patients with atrial fibrillation: a report of the American College of Cardiology/American Heart Association Task Force on practice guidelines and the Heart Rhythm Society. Circulation. 2014;130(23):e199-e267. Erratum in Circulation. 2014;130(23):e272-e274. doi:10.1161/CIR.0000000000000041.
3. Kirchhof P, Benussi S, Kotecha D, et al. 2016 ESC Guidelines for the management of atrial fibrillation developed in collaboration with EACTS. Eur Heart J. 2016:37(38):2893-2962. doi:10.1093/eurheartj/ehw210.
4. Airaksinen KE, Grönberg T, Nuotio I, et al. Thromboembolic complications after cardioversion of acute atrial fibrillation: the FinCV (Finnish CardioVersion) study. J Am Coll Cardiol. 2013;62(13):1187-1192. doi:10.1016/j.jacc.2013.04.089.
5. Hansen ML, Jepsen RM, Olesen JB, et al. Thromboembolic risk in 16 274 atrial fibrillation patients undergoing direct current cardioversion with and without oral anticoagulant therapy. Europace. 2015;17(1):18-23. doi:10.1093/europace/euu189.

References

1. Phang R, Manning WJ. Prevention of embolization prior to and after restoration of sinus rhythm in atrial fibrillation. UptoDate Web site. http://www.uptodate.com/contents/prevention-of-embolization-prior-to-and-after-restoration-of-sinus-rhythm-in-atrial-fibrillation. Updated October 10, 2016. Accessed March 6, 2017.
2. January CT, Wann LS, Alpert JS, et al; ACC/AHA Task Force Members. 2014 AHA/ACC/HRS guideline for the management of patients with atrial fibrillation: a report of the American College of Cardiology/American Heart Association Task Force on practice guidelines and the Heart Rhythm Society. Circulation. 2014;130(23):e199-e267. Erratum in Circulation. 2014;130(23):e272-e274. doi:10.1161/CIR.0000000000000041.
3. Kirchhof P, Benussi S, Kotecha D, et al. 2016 ESC Guidelines for the management of atrial fibrillation developed in collaboration with EACTS. Eur Heart J. 2016:37(38):2893-2962. doi:10.1093/eurheartj/ehw210.
4. Airaksinen KE, Grönberg T, Nuotio I, et al. Thromboembolic complications after cardioversion of acute atrial fibrillation: the FinCV (Finnish CardioVersion) study. J Am Coll Cardiol. 2013;62(13):1187-1192. doi:10.1016/j.jacc.2013.04.089.
5. Hansen ML, Jepsen RM, Olesen JB, et al. Thromboembolic risk in 16 274 atrial fibrillation patients undergoing direct current cardioversion with and without oral anticoagulant therapy. Europace. 2015;17(1):18-23. doi:10.1093/europace/euu189.

Issue
Emergency Medicine - 49(4)
Issue
Emergency Medicine - 49(4)
Page Number
172-174
Page Number
172-174
Publications
Publications
Topics
Article Type
Display Headline
Malpractice Counsel: The Challenges of Cardioversion
Display Headline
Malpractice Counsel: The Challenges of Cardioversion
Sections
Disallow All Ads
Content Gating
No Gating (article Unlocked/Free)
Alternative CME
Article PDF Media

Malpractice Counsel: Pain in the Back, Pain in the Butt(ocks)

Article Type
Changed
Wed, 12/12/2018 - 20:55
Display Headline
Malpractice Counsel: Pain in the Back, Pain in the Butt(ocks)

A Pain in the Back

A 57-year-old man presented to the ED with a chief complaint of severe low back pain that radiated into his left buttock. The patient stated the pain started immediately after he had bent over to pick up a small refrigerator. He denied any abdominal pain, lower extremity numbness, weakness, or bowel or bladder dysfunction. The patient’s medical history was significant for hypertension, for which he was taking amlodipine, and sleep apnea. The patient stated that he had a continuous positive airway pressure (CPAP) machine for his sleep apnea, but did not use it regularly. Regarding his social history, the patient denied tobacco use, but did admit to daily alcohol consumption.

The patient’s vital signs were all normal. Physical examination was remarkable only for bilateral lumbar paraspinal muscle tenderness, which was greater on the left side. There was no midline tenderness. Straight leg-raise testing was negative bilaterally, and the patient had normal strength and deep tendon reflexes in the lower extremities. The abdomen was soft and nontender.

The emergency physician (EP) diagnosed the patient with muscle strain, and discharged him home with a prescription for hydrocodone, along with instructions to follow-up with his primary care physician (PCP) within the next few days.

Three days later, the patient presented again to the same ED complaining of increased lower back pain. He denied any new injury or overuse, and continued to deny any lower extremity numbness or weakness or bowel/bladder dysfunction. Similarly, the physical examination was unchanged. The patient was given an intramuscular (IM) injection of hydromorphone with promethazine, as well as oral diazepam, and discharged home with instructions to continue to take the hydrocodone as needed for pain.

According to his wife, the patient retired to bed shortly after arriving home from the ED. Approximately 90 minutes later, she discovered the patient unresponsive in bed and called emergency medical services (EMS). He was brought back to the same hospital ED via EMS and was emergently intubated upon arrival. Unfortunately, the patient had suffered an anoxic brain injury and never regained consciousness; he died 1 week later.

The patient’s wife sued the EP, claiming the anoxic brain injury was related to the drugs ordered by the EP in combination with the patient’s alcohol use. She alleged that if the EP had observed the patient in the ED for signs of respiratory distress, his condition would have been treated and the anoxic brain injury would have been prevented. The plaintiff also faulted the EP for not informing the patient of the risks of drinking alcohol while taking the prescription pain medication.

The EP asserted his care of the patient was appropriate, and that there was no reason to keep the patient for observation. Regarding counseling the patient about the risks associated with concomitant alcohol consumption and pain medication, the EP stated that he had relied on the nurse who administered the medications to provide such counsel. The EP further maintained the plaintiff’s death was due to the patient not using his CPAP machine as prescribed for sleep apnea, along with his alcohol consumption the evening of the event. At trial, a defense verdict was returned.

Discussion

Unfortunately, there are several unknowns in this case. Did the patient drink any alcohol after returning home from the second ED visit, prior to going to sleep? If so, how much did he consume? Did he take any of the narcotic pills prescribed from the first ED visit and, if so, how many did he take and in what time frame?

The combination of narcotics, benzodiazepines, and alcohol has long been known to be a potentially lethal combination, resulting in respiratory depression, respiratory arrest, anoxic brain injury and even death. As EPs, we are confronted with patients complaining of pain during every shift. Complicating matters, in national patient surveys concerning the care received in the ED, patients are specifically asked if their pain was adequately treated. At the same time, there is a national effort across all specialties to reduce the amount of opioids prescribed to patients. The EP should therefore attempt to select the least potent medication that will adequately control the patient’s pain.

The WHO Pain Ladder

In 1986, the World Health Organization (WHO) developed a three-step analgesic ladder to guide the management of cancer pain.1 This guide has since been expanded to include pain of noncancer etiology. Mild pain, defined on the numerical rating scale (NRS) as 1 to 3, is considered step 1.1,2 Moderate pain (NRS of 4-6) is considered step 2, and severe pain (NRS of 7-10) is step 3. For step 1 pain, acetaminophen or a nonsteroidal anti-inflammatory drug (NSAID) is recommended. For step 2 pain, a weak narcotic (eg, codeine, hydrocodone), with or without acetaminophen or an NSAID, is recommended. For step 3 pain, a strong narcotic agent such as morphine or hydromorphone is advised.1

The WHO’s ladder is not meant to serve as a strict protocol, but rather as a tool to guide the clinician in determining a reasonable starting point in pain management. Although the EP in this case did not ask the patient to rate his pain, from all indications it appeared to be severe (step 3) and as such, the choice of prescribing hydromorphone was a reasonable one. However, most experts agree that it is best to titrate an analgesic to the desired effect. In patients with severe pain, this means employing the intravenous (IV) route, not the IM route, which was used in this case. This is because the IM route can result in variable absorption and an unpredictable time of onset and duration of action.

 

 

Concomitant Antiemetic Therapy

It is common practice to administer an antiemetic simultaneously with a narcotic to prevent or lessen associated nausea and vomiting. The clinician must be aware, however, that all antiemetics act as central nervous system (CNS) depressants to some degree. The addition of diazepam in this case is problematic because all benzodiazepines cause sedation and anxiolysis. The combination of benzodiazepines with other CNS depressants, such as opioid analgesics, can lead to excessive sedation, resulting in partial airway obstruction, respiratory depression, and hypoxia.3 The risk of an adverse outcome significantly increases with concomitant alcohol consumption.

The EP must carefully consider the risks and benefits any time opioids and benzodiazepines are administered in combination. In addition, the underlying health of the patient must be considered. Risk factors for opioid-induced respiratory depression includes age older than 50 years, a history of sleep apnea, preexisting pulmonary disease (eg, chronic obstructive pulmonary disease), anatomic oral or airway abnormalities, and renal or hepatic impairment.3 Finally, patients should be informed of the dangers of mixing alcohol with opioids and benzodiazepines—whether such counsel is given by a physician, nurse, or pharmacist.

References

1. World Health Organization. Cancer Pain Relief. 2nd ed. Geneva, Switzerland: World Health Organization; 1996:1-69.
2. Todd KH. Pain assessment instruments for us in the emergency department. Emerg Med Clin North Am. 2005;23(2):285-295.
3. Jarzyna D, Jungquist CR, Pasero C, et al. American Society for Pain Management Nursing guidelines on monitoring for opioid-induced sedation and respiratory depression. Pain Management Nursing. 2011;12(3):118-145.

A Pain in the Buttock(s)

A 33-year-old Hispanic man, whose only spoken language was Spanish, presented to the ED for evaluation of pain in his right buttock. A coworker who accompanied the patient to the ED and served as his translator stated the patient’s pain began the previous day, immediately following a work-related injury in which the patient had slipped and fallen backward, landing on his buttocks. The patient denied any head injury, loss of consciousness, or neck pain. He further stated that he was otherwise in good health and was not taking any medications. Regarding social history, the patient denied any alcohol or drug use.

The patient’s vital signs were normal, as was his physical examination. The EP ordered an anteroposterior X-ray of the pelvis, which radiology services interpreted as normal. The EP diagnosed a buttock contusion secondary to fall, and discharged the patient home with instructions to take over-the-counter (OTC) ibuprofen for pain as needed.

Four days later, the patient presented to the same ED, complaining of low back pain radiating down his right leg. He denied any new injuries or falls, any lower extremity weakness, or bowel or bladder discomfort. Through a translator, the patient further noted that although he had been taking the OTC ibuprofen as prescribed, it had not alleviated his pain.

On physical examination, the patient’s vital signs were: blood pressure (BP), 112/62 mm Hg; heart rate (HR), 96 beats/min; respiratory rate (RR), 20 breaths/min; and temperature (T), 101.8˚F. Oxygen saturation was 98% on room air. The lung, heart, and abdominal examinations were normal. The patient was noted to be tender to palpation over the sacral and coccygeal region. There was no documentation of a lower extremity or neurological examination.

The EP ordered a computed tomography (CT) scan of the pelvis without contrast. This was interpreted by radiology services as demonstrating moderate facet arthropathy at L4/L5 on the left side, with a facet joint cyst extending in the central canal; no fracture was identified. The radiologist suggested that these findings could be better evaluated with a magnetic resonance imaging (MRI) study of the lumbar spine, if clinically indicated. The EP decided against ordering the MRI, diagnosed the patient with a contusion of the coccyx, and discharged him home with instructions to continue taking OTC ibuprofen; he also prescribed combination acetaminophen/oxycodone for severe pain, and instructed the patient to follow-up with his PCP in 2 days.

Forty eight hours later, the patient returned to the same ED via EMS, now with the complaint of inability to walk or urinate. He continued to have a fever and was tachycardic with an HR of 110 beats/min; BP, RR, and T were normal, as was his oxygen saturation on room air. The patient was noted to have significant lower extremity weakness. A Foley catheter was placed, and 1,200 cc of urine was obtained. An emergent MRI of the lumbar spine revealed an extensive lumbar epidural abscess along the lumbar spine. The patient was started on IV antibiotics and taken to the operating room for evacuation of the epidural abscess. Unfortunately, he had a prolonged inpatient stay due to persistent bilateral lower extremity weakness and neurogenic bowel and bladder dysfunction; he eventually was discharged to a skilled nursing facility.

The patient sued the EP and hospital for their failure to appreciate the significance of his presentation at the second ED visit. The attorneys for the plaintiff argued the defendants failed to order laboratory tests in the presence of fever, and failed to consult neurosurgery services. The plaintiff’s neurosurgeon expert stated the extensive lumbar epidural abscess seen on the MRI during the third ED visit would have been appreciable on the second ED visit if an MRI had been obtained at that time. The defendants settled the case for $1 million.

 

 

Discussion

Spinal epidural abscesses (SEAs) are an uncommon but serious infection that must be recognized and treated promptly to avoid permanent neurological complications. These abscesses occur most commonly in the thoracolumbar area, where the epidural space is larger. Since the epidural space is a vertical sheath, an abscess that begins at one level commonly extends to multiple levels; SEAs frequently range three to five spinal cord segments.1 The median age of onset for an SEA is approximately 50 years, and they are more common in men.1 Risk factors for the development of an SEA include epidural catheter placement, paraspinal injections of glucocorticoids or analgesics, IV drug abuse, human immunodeficiency virus infection, diabetes mellitus, alcohol abuse, trauma, tattoos, acupuncture, and hemodialysis.1,2 The most common pathogens causing an SEA are Staphylococcus aureus, gram-negative bacilli, and Streptococci.2 The percentage of S aureus that are methicillin-resistant (ie, MRSA) varies by geographic location, ranging from 40% to 68%.3,4

Signs and Symptoms

Typically, patients with an SEA initially present with fever, malaise, and nonspecific symptoms and, as seen in this case, generally present several times to a physician before the correct diagnosis is made.1 Unfortunately, the classic triad of fever, spinal pain, and neurological deficits is only infrequently observed. Fever is present in approximately two-thirds of patients, and spinal pain is present approximately 90% of the time.2

There are four stages of disease progression associated with SEAs. A typical scenario involves the initial complaint of back pain (stage I); followed by pain in the distribution of an affected nerve root (stage II); then motor weakness, sensory changes, and bladder or bowel dysfunction (stage III); and, finally, paralysis (stage IV).1,2

Diagnosis

Laboratory studies typically are not helpful in making the diagnosis. A complete blood count may show leukocytosis, but values can also be within the normal reference range. Acute phase reactants like erythrocyte sedimentation rate and C-reactive protein are commonly elevated with an SEA, but are neither sensitive nor specific.1

To make the diagnosis, the best test is a gadolinium-enhanced MRI of the spine.2 It may be prudent to image the entire spine because multiple skip lesions are common, and a patient may not have pain or tenderness in all affected areas. If MRI is not available, a CT scan of the spine with IV contrast is an acceptable alternative.1

Once an SEA is identified, it is important to determine the organism(s) responsible for the infection. The best culture source is from the abscess itself (90%) followed by blood cultures (62%) and cerebrospinal fluid, which are positive only 19% of the time.1

Treatment

Once an SEA is diagnosed, a multidisciplinary approach involving hospitalists, interventional radiology, neurosurgery, and/or orthopedics is best. The most effective management is to treat patients with a combination of surgical decompression and drainage with systemic antibiotic therapy, typically for a minimum of 4 weeks. A minority of select patients may be treated with antibiotics alone.

References

1. Sexton, DJ, Sampson JH. Spinal epidural abscess. UpToDate Web site. http://www.uptodate.com/contents/spinal-epidural-abscess. Updated June 23, 2016. Accessed January 9, 2017.
2. Darouiche RO. Spinal epidural abscess. N Engl J Med. 2006;355(19):2012-2020. doi:10.1056/NEJMra055111.
3. Chen WC, Wang JL, Wang JT, Chen YC, Chang SC. Spinal epidural abscess due to Staphylococcus aureus: clinical manifestations and outcomes. J Microbiol Immunol Infect. 2008;41(3):215-221.
4. Krishnamohan P, Berger JR. Spinal epidural abscess. Curr Infect Dis Rep. 2014;16(11):436. doi:10.1007/s11908-014-0436-7.

Article PDF
Author and Disclosure Information

Author’s Disclosure Statement: The author reports no actual or potential conflict of interest in relation to this article.

Issue
Emergency Medicine - 49(2)
Publications
Topics
Page Number
84-87
Sections
Author and Disclosure Information

Author’s Disclosure Statement: The author reports no actual or potential conflict of interest in relation to this article.

Author and Disclosure Information

Author’s Disclosure Statement: The author reports no actual or potential conflict of interest in relation to this article.

Article PDF
Article PDF

A Pain in the Back

A 57-year-old man presented to the ED with a chief complaint of severe low back pain that radiated into his left buttock. The patient stated the pain started immediately after he had bent over to pick up a small refrigerator. He denied any abdominal pain, lower extremity numbness, weakness, or bowel or bladder dysfunction. The patient’s medical history was significant for hypertension, for which he was taking amlodipine, and sleep apnea. The patient stated that he had a continuous positive airway pressure (CPAP) machine for his sleep apnea, but did not use it regularly. Regarding his social history, the patient denied tobacco use, but did admit to daily alcohol consumption.

The patient’s vital signs were all normal. Physical examination was remarkable only for bilateral lumbar paraspinal muscle tenderness, which was greater on the left side. There was no midline tenderness. Straight leg-raise testing was negative bilaterally, and the patient had normal strength and deep tendon reflexes in the lower extremities. The abdomen was soft and nontender.

The emergency physician (EP) diagnosed the patient with muscle strain, and discharged him home with a prescription for hydrocodone, along with instructions to follow-up with his primary care physician (PCP) within the next few days.

Three days later, the patient presented again to the same ED complaining of increased lower back pain. He denied any new injury or overuse, and continued to deny any lower extremity numbness or weakness or bowel/bladder dysfunction. Similarly, the physical examination was unchanged. The patient was given an intramuscular (IM) injection of hydromorphone with promethazine, as well as oral diazepam, and discharged home with instructions to continue to take the hydrocodone as needed for pain.

According to his wife, the patient retired to bed shortly after arriving home from the ED. Approximately 90 minutes later, she discovered the patient unresponsive in bed and called emergency medical services (EMS). He was brought back to the same hospital ED via EMS and was emergently intubated upon arrival. Unfortunately, the patient had suffered an anoxic brain injury and never regained consciousness; he died 1 week later.

The patient’s wife sued the EP, claiming the anoxic brain injury was related to the drugs ordered by the EP in combination with the patient’s alcohol use. She alleged that if the EP had observed the patient in the ED for signs of respiratory distress, his condition would have been treated and the anoxic brain injury would have been prevented. The plaintiff also faulted the EP for not informing the patient of the risks of drinking alcohol while taking the prescription pain medication.

The EP asserted his care of the patient was appropriate, and that there was no reason to keep the patient for observation. Regarding counseling the patient about the risks associated with concomitant alcohol consumption and pain medication, the EP stated that he had relied on the nurse who administered the medications to provide such counsel. The EP further maintained the plaintiff’s death was due to the patient not using his CPAP machine as prescribed for sleep apnea, along with his alcohol consumption the evening of the event. At trial, a defense verdict was returned.

Discussion

Unfortunately, there are several unknowns in this case. Did the patient drink any alcohol after returning home from the second ED visit, prior to going to sleep? If so, how much did he consume? Did he take any of the narcotic pills prescribed from the first ED visit and, if so, how many did he take and in what time frame?

The combination of narcotics, benzodiazepines, and alcohol has long been known to be a potentially lethal combination, resulting in respiratory depression, respiratory arrest, anoxic brain injury and even death. As EPs, we are confronted with patients complaining of pain during every shift. Complicating matters, in national patient surveys concerning the care received in the ED, patients are specifically asked if their pain was adequately treated. At the same time, there is a national effort across all specialties to reduce the amount of opioids prescribed to patients. The EP should therefore attempt to select the least potent medication that will adequately control the patient’s pain.

The WHO Pain Ladder

In 1986, the World Health Organization (WHO) developed a three-step analgesic ladder to guide the management of cancer pain.1 This guide has since been expanded to include pain of noncancer etiology. Mild pain, defined on the numerical rating scale (NRS) as 1 to 3, is considered step 1.1,2 Moderate pain (NRS of 4-6) is considered step 2, and severe pain (NRS of 7-10) is step 3. For step 1 pain, acetaminophen or a nonsteroidal anti-inflammatory drug (NSAID) is recommended. For step 2 pain, a weak narcotic (eg, codeine, hydrocodone), with or without acetaminophen or an NSAID, is recommended. For step 3 pain, a strong narcotic agent such as morphine or hydromorphone is advised.1

The WHO’s ladder is not meant to serve as a strict protocol, but rather as a tool to guide the clinician in determining a reasonable starting point in pain management. Although the EP in this case did not ask the patient to rate his pain, from all indications it appeared to be severe (step 3) and as such, the choice of prescribing hydromorphone was a reasonable one. However, most experts agree that it is best to titrate an analgesic to the desired effect. In patients with severe pain, this means employing the intravenous (IV) route, not the IM route, which was used in this case. This is because the IM route can result in variable absorption and an unpredictable time of onset and duration of action.

 

 

Concomitant Antiemetic Therapy

It is common practice to administer an antiemetic simultaneously with a narcotic to prevent or lessen associated nausea and vomiting. The clinician must be aware, however, that all antiemetics act as central nervous system (CNS) depressants to some degree. The addition of diazepam in this case is problematic because all benzodiazepines cause sedation and anxiolysis. The combination of benzodiazepines with other CNS depressants, such as opioid analgesics, can lead to excessive sedation, resulting in partial airway obstruction, respiratory depression, and hypoxia.3 The risk of an adverse outcome significantly increases with concomitant alcohol consumption.

The EP must carefully consider the risks and benefits any time opioids and benzodiazepines are administered in combination. In addition, the underlying health of the patient must be considered. Risk factors for opioid-induced respiratory depression includes age older than 50 years, a history of sleep apnea, preexisting pulmonary disease (eg, chronic obstructive pulmonary disease), anatomic oral or airway abnormalities, and renal or hepatic impairment.3 Finally, patients should be informed of the dangers of mixing alcohol with opioids and benzodiazepines—whether such counsel is given by a physician, nurse, or pharmacist.

References

1. World Health Organization. Cancer Pain Relief. 2nd ed. Geneva, Switzerland: World Health Organization; 1996:1-69.
2. Todd KH. Pain assessment instruments for us in the emergency department. Emerg Med Clin North Am. 2005;23(2):285-295.
3. Jarzyna D, Jungquist CR, Pasero C, et al. American Society for Pain Management Nursing guidelines on monitoring for opioid-induced sedation and respiratory depression. Pain Management Nursing. 2011;12(3):118-145.

A Pain in the Buttock(s)

A 33-year-old Hispanic man, whose only spoken language was Spanish, presented to the ED for evaluation of pain in his right buttock. A coworker who accompanied the patient to the ED and served as his translator stated the patient’s pain began the previous day, immediately following a work-related injury in which the patient had slipped and fallen backward, landing on his buttocks. The patient denied any head injury, loss of consciousness, or neck pain. He further stated that he was otherwise in good health and was not taking any medications. Regarding social history, the patient denied any alcohol or drug use.

The patient’s vital signs were normal, as was his physical examination. The EP ordered an anteroposterior X-ray of the pelvis, which radiology services interpreted as normal. The EP diagnosed a buttock contusion secondary to fall, and discharged the patient home with instructions to take over-the-counter (OTC) ibuprofen for pain as needed.

Four days later, the patient presented to the same ED, complaining of low back pain radiating down his right leg. He denied any new injuries or falls, any lower extremity weakness, or bowel or bladder discomfort. Through a translator, the patient further noted that although he had been taking the OTC ibuprofen as prescribed, it had not alleviated his pain.

On physical examination, the patient’s vital signs were: blood pressure (BP), 112/62 mm Hg; heart rate (HR), 96 beats/min; respiratory rate (RR), 20 breaths/min; and temperature (T), 101.8˚F. Oxygen saturation was 98% on room air. The lung, heart, and abdominal examinations were normal. The patient was noted to be tender to palpation over the sacral and coccygeal region. There was no documentation of a lower extremity or neurological examination.

The EP ordered a computed tomography (CT) scan of the pelvis without contrast. This was interpreted by radiology services as demonstrating moderate facet arthropathy at L4/L5 on the left side, with a facet joint cyst extending in the central canal; no fracture was identified. The radiologist suggested that these findings could be better evaluated with a magnetic resonance imaging (MRI) study of the lumbar spine, if clinically indicated. The EP decided against ordering the MRI, diagnosed the patient with a contusion of the coccyx, and discharged him home with instructions to continue taking OTC ibuprofen; he also prescribed combination acetaminophen/oxycodone for severe pain, and instructed the patient to follow-up with his PCP in 2 days.

Forty eight hours later, the patient returned to the same ED via EMS, now with the complaint of inability to walk or urinate. He continued to have a fever and was tachycardic with an HR of 110 beats/min; BP, RR, and T were normal, as was his oxygen saturation on room air. The patient was noted to have significant lower extremity weakness. A Foley catheter was placed, and 1,200 cc of urine was obtained. An emergent MRI of the lumbar spine revealed an extensive lumbar epidural abscess along the lumbar spine. The patient was started on IV antibiotics and taken to the operating room for evacuation of the epidural abscess. Unfortunately, he had a prolonged inpatient stay due to persistent bilateral lower extremity weakness and neurogenic bowel and bladder dysfunction; he eventually was discharged to a skilled nursing facility.

The patient sued the EP and hospital for their failure to appreciate the significance of his presentation at the second ED visit. The attorneys for the plaintiff argued the defendants failed to order laboratory tests in the presence of fever, and failed to consult neurosurgery services. The plaintiff’s neurosurgeon expert stated the extensive lumbar epidural abscess seen on the MRI during the third ED visit would have been appreciable on the second ED visit if an MRI had been obtained at that time. The defendants settled the case for $1 million.

 

 

Discussion

Spinal epidural abscesses (SEAs) are an uncommon but serious infection that must be recognized and treated promptly to avoid permanent neurological complications. These abscesses occur most commonly in the thoracolumbar area, where the epidural space is larger. Since the epidural space is a vertical sheath, an abscess that begins at one level commonly extends to multiple levels; SEAs frequently range three to five spinal cord segments.1 The median age of onset for an SEA is approximately 50 years, and they are more common in men.1 Risk factors for the development of an SEA include epidural catheter placement, paraspinal injections of glucocorticoids or analgesics, IV drug abuse, human immunodeficiency virus infection, diabetes mellitus, alcohol abuse, trauma, tattoos, acupuncture, and hemodialysis.1,2 The most common pathogens causing an SEA are Staphylococcus aureus, gram-negative bacilli, and Streptococci.2 The percentage of S aureus that are methicillin-resistant (ie, MRSA) varies by geographic location, ranging from 40% to 68%.3,4

Signs and Symptoms

Typically, patients with an SEA initially present with fever, malaise, and nonspecific symptoms and, as seen in this case, generally present several times to a physician before the correct diagnosis is made.1 Unfortunately, the classic triad of fever, spinal pain, and neurological deficits is only infrequently observed. Fever is present in approximately two-thirds of patients, and spinal pain is present approximately 90% of the time.2

There are four stages of disease progression associated with SEAs. A typical scenario involves the initial complaint of back pain (stage I); followed by pain in the distribution of an affected nerve root (stage II); then motor weakness, sensory changes, and bladder or bowel dysfunction (stage III); and, finally, paralysis (stage IV).1,2

Diagnosis

Laboratory studies typically are not helpful in making the diagnosis. A complete blood count may show leukocytosis, but values can also be within the normal reference range. Acute phase reactants like erythrocyte sedimentation rate and C-reactive protein are commonly elevated with an SEA, but are neither sensitive nor specific.1

To make the diagnosis, the best test is a gadolinium-enhanced MRI of the spine.2 It may be prudent to image the entire spine because multiple skip lesions are common, and a patient may not have pain or tenderness in all affected areas. If MRI is not available, a CT scan of the spine with IV contrast is an acceptable alternative.1

Once an SEA is identified, it is important to determine the organism(s) responsible for the infection. The best culture source is from the abscess itself (90%) followed by blood cultures (62%) and cerebrospinal fluid, which are positive only 19% of the time.1

Treatment

Once an SEA is diagnosed, a multidisciplinary approach involving hospitalists, interventional radiology, neurosurgery, and/or orthopedics is best. The most effective management is to treat patients with a combination of surgical decompression and drainage with systemic antibiotic therapy, typically for a minimum of 4 weeks. A minority of select patients may be treated with antibiotics alone.

References

1. Sexton, DJ, Sampson JH. Spinal epidural abscess. UpToDate Web site. http://www.uptodate.com/contents/spinal-epidural-abscess. Updated June 23, 2016. Accessed January 9, 2017.
2. Darouiche RO. Spinal epidural abscess. N Engl J Med. 2006;355(19):2012-2020. doi:10.1056/NEJMra055111.
3. Chen WC, Wang JL, Wang JT, Chen YC, Chang SC. Spinal epidural abscess due to Staphylococcus aureus: clinical manifestations and outcomes. J Microbiol Immunol Infect. 2008;41(3):215-221.
4. Krishnamohan P, Berger JR. Spinal epidural abscess. Curr Infect Dis Rep. 2014;16(11):436. doi:10.1007/s11908-014-0436-7.

A Pain in the Back

A 57-year-old man presented to the ED with a chief complaint of severe low back pain that radiated into his left buttock. The patient stated the pain started immediately after he had bent over to pick up a small refrigerator. He denied any abdominal pain, lower extremity numbness, weakness, or bowel or bladder dysfunction. The patient’s medical history was significant for hypertension, for which he was taking amlodipine, and sleep apnea. The patient stated that he had a continuous positive airway pressure (CPAP) machine for his sleep apnea, but did not use it regularly. Regarding his social history, the patient denied tobacco use, but did admit to daily alcohol consumption.

The patient’s vital signs were all normal. Physical examination was remarkable only for bilateral lumbar paraspinal muscle tenderness, which was greater on the left side. There was no midline tenderness. Straight leg-raise testing was negative bilaterally, and the patient had normal strength and deep tendon reflexes in the lower extremities. The abdomen was soft and nontender.

The emergency physician (EP) diagnosed the patient with muscle strain, and discharged him home with a prescription for hydrocodone, along with instructions to follow-up with his primary care physician (PCP) within the next few days.

Three days later, the patient presented again to the same ED complaining of increased lower back pain. He denied any new injury or overuse, and continued to deny any lower extremity numbness or weakness or bowel/bladder dysfunction. Similarly, the physical examination was unchanged. The patient was given an intramuscular (IM) injection of hydromorphone with promethazine, as well as oral diazepam, and discharged home with instructions to continue to take the hydrocodone as needed for pain.

According to his wife, the patient retired to bed shortly after arriving home from the ED. Approximately 90 minutes later, she discovered the patient unresponsive in bed and called emergency medical services (EMS). He was brought back to the same hospital ED via EMS and was emergently intubated upon arrival. Unfortunately, the patient had suffered an anoxic brain injury and never regained consciousness; he died 1 week later.

The patient’s wife sued the EP, claiming the anoxic brain injury was related to the drugs ordered by the EP in combination with the patient’s alcohol use. She alleged that if the EP had observed the patient in the ED for signs of respiratory distress, his condition would have been treated and the anoxic brain injury would have been prevented. The plaintiff also faulted the EP for not informing the patient of the risks of drinking alcohol while taking the prescription pain medication.

The EP asserted his care of the patient was appropriate, and that there was no reason to keep the patient for observation. Regarding counseling the patient about the risks associated with concomitant alcohol consumption and pain medication, the EP stated that he had relied on the nurse who administered the medications to provide such counsel. The EP further maintained the plaintiff’s death was due to the patient not using his CPAP machine as prescribed for sleep apnea, along with his alcohol consumption the evening of the event. At trial, a defense verdict was returned.

Discussion

Unfortunately, there are several unknowns in this case. Did the patient drink any alcohol after returning home from the second ED visit, prior to going to sleep? If so, how much did he consume? Did he take any of the narcotic pills prescribed from the first ED visit and, if so, how many did he take and in what time frame?

The combination of narcotics, benzodiazepines, and alcohol has long been known to be a potentially lethal combination, resulting in respiratory depression, respiratory arrest, anoxic brain injury and even death. As EPs, we are confronted with patients complaining of pain during every shift. Complicating matters, in national patient surveys concerning the care received in the ED, patients are specifically asked if their pain was adequately treated. At the same time, there is a national effort across all specialties to reduce the amount of opioids prescribed to patients. The EP should therefore attempt to select the least potent medication that will adequately control the patient’s pain.

The WHO Pain Ladder

In 1986, the World Health Organization (WHO) developed a three-step analgesic ladder to guide the management of cancer pain.1 This guide has since been expanded to include pain of noncancer etiology. Mild pain, defined on the numerical rating scale (NRS) as 1 to 3, is considered step 1.1,2 Moderate pain (NRS of 4-6) is considered step 2, and severe pain (NRS of 7-10) is step 3. For step 1 pain, acetaminophen or a nonsteroidal anti-inflammatory drug (NSAID) is recommended. For step 2 pain, a weak narcotic (eg, codeine, hydrocodone), with or without acetaminophen or an NSAID, is recommended. For step 3 pain, a strong narcotic agent such as morphine or hydromorphone is advised.1

The WHO’s ladder is not meant to serve as a strict protocol, but rather as a tool to guide the clinician in determining a reasonable starting point in pain management. Although the EP in this case did not ask the patient to rate his pain, from all indications it appeared to be severe (step 3) and as such, the choice of prescribing hydromorphone was a reasonable one. However, most experts agree that it is best to titrate an analgesic to the desired effect. In patients with severe pain, this means employing the intravenous (IV) route, not the IM route, which was used in this case. This is because the IM route can result in variable absorption and an unpredictable time of onset and duration of action.

 

 

Concomitant Antiemetic Therapy

It is common practice to administer an antiemetic simultaneously with a narcotic to prevent or lessen associated nausea and vomiting. The clinician must be aware, however, that all antiemetics act as central nervous system (CNS) depressants to some degree. The addition of diazepam in this case is problematic because all benzodiazepines cause sedation and anxiolysis. The combination of benzodiazepines with other CNS depressants, such as opioid analgesics, can lead to excessive sedation, resulting in partial airway obstruction, respiratory depression, and hypoxia.3 The risk of an adverse outcome significantly increases with concomitant alcohol consumption.

The EP must carefully consider the risks and benefits any time opioids and benzodiazepines are administered in combination. In addition, the underlying health of the patient must be considered. Risk factors for opioid-induced respiratory depression includes age older than 50 years, a history of sleep apnea, preexisting pulmonary disease (eg, chronic obstructive pulmonary disease), anatomic oral or airway abnormalities, and renal or hepatic impairment.3 Finally, patients should be informed of the dangers of mixing alcohol with opioids and benzodiazepines—whether such counsel is given by a physician, nurse, or pharmacist.

References

1. World Health Organization. Cancer Pain Relief. 2nd ed. Geneva, Switzerland: World Health Organization; 1996:1-69.
2. Todd KH. Pain assessment instruments for us in the emergency department. Emerg Med Clin North Am. 2005;23(2):285-295.
3. Jarzyna D, Jungquist CR, Pasero C, et al. American Society for Pain Management Nursing guidelines on monitoring for opioid-induced sedation and respiratory depression. Pain Management Nursing. 2011;12(3):118-145.

A Pain in the Buttock(s)

A 33-year-old Hispanic man, whose only spoken language was Spanish, presented to the ED for evaluation of pain in his right buttock. A coworker who accompanied the patient to the ED and served as his translator stated the patient’s pain began the previous day, immediately following a work-related injury in which the patient had slipped and fallen backward, landing on his buttocks. The patient denied any head injury, loss of consciousness, or neck pain. He further stated that he was otherwise in good health and was not taking any medications. Regarding social history, the patient denied any alcohol or drug use.

The patient’s vital signs were normal, as was his physical examination. The EP ordered an anteroposterior X-ray of the pelvis, which radiology services interpreted as normal. The EP diagnosed a buttock contusion secondary to fall, and discharged the patient home with instructions to take over-the-counter (OTC) ibuprofen for pain as needed.

Four days later, the patient presented to the same ED, complaining of low back pain radiating down his right leg. He denied any new injuries or falls, any lower extremity weakness, or bowel or bladder discomfort. Through a translator, the patient further noted that although he had been taking the OTC ibuprofen as prescribed, it had not alleviated his pain.

On physical examination, the patient’s vital signs were: blood pressure (BP), 112/62 mm Hg; heart rate (HR), 96 beats/min; respiratory rate (RR), 20 breaths/min; and temperature (T), 101.8˚F. Oxygen saturation was 98% on room air. The lung, heart, and abdominal examinations were normal. The patient was noted to be tender to palpation over the sacral and coccygeal region. There was no documentation of a lower extremity or neurological examination.

The EP ordered a computed tomography (CT) scan of the pelvis without contrast. This was interpreted by radiology services as demonstrating moderate facet arthropathy at L4/L5 on the left side, with a facet joint cyst extending in the central canal; no fracture was identified. The radiologist suggested that these findings could be better evaluated with a magnetic resonance imaging (MRI) study of the lumbar spine, if clinically indicated. The EP decided against ordering the MRI, diagnosed the patient with a contusion of the coccyx, and discharged him home with instructions to continue taking OTC ibuprofen; he also prescribed combination acetaminophen/oxycodone for severe pain, and instructed the patient to follow-up with his PCP in 2 days.

Forty eight hours later, the patient returned to the same ED via EMS, now with the complaint of inability to walk or urinate. He continued to have a fever and was tachycardic with an HR of 110 beats/min; BP, RR, and T were normal, as was his oxygen saturation on room air. The patient was noted to have significant lower extremity weakness. A Foley catheter was placed, and 1,200 cc of urine was obtained. An emergent MRI of the lumbar spine revealed an extensive lumbar epidural abscess along the lumbar spine. The patient was started on IV antibiotics and taken to the operating room for evacuation of the epidural abscess. Unfortunately, he had a prolonged inpatient stay due to persistent bilateral lower extremity weakness and neurogenic bowel and bladder dysfunction; he eventually was discharged to a skilled nursing facility.

The patient sued the EP and hospital for their failure to appreciate the significance of his presentation at the second ED visit. The attorneys for the plaintiff argued the defendants failed to order laboratory tests in the presence of fever, and failed to consult neurosurgery services. The plaintiff’s neurosurgeon expert stated the extensive lumbar epidural abscess seen on the MRI during the third ED visit would have been appreciable on the second ED visit if an MRI had been obtained at that time. The defendants settled the case for $1 million.

 

 

Discussion

Spinal epidural abscesses (SEAs) are an uncommon but serious infection that must be recognized and treated promptly to avoid permanent neurological complications. These abscesses occur most commonly in the thoracolumbar area, where the epidural space is larger. Since the epidural space is a vertical sheath, an abscess that begins at one level commonly extends to multiple levels; SEAs frequently range three to five spinal cord segments.1 The median age of onset for an SEA is approximately 50 years, and they are more common in men.1 Risk factors for the development of an SEA include epidural catheter placement, paraspinal injections of glucocorticoids or analgesics, IV drug abuse, human immunodeficiency virus infection, diabetes mellitus, alcohol abuse, trauma, tattoos, acupuncture, and hemodialysis.1,2 The most common pathogens causing an SEA are Staphylococcus aureus, gram-negative bacilli, and Streptococci.2 The percentage of S aureus that are methicillin-resistant (ie, MRSA) varies by geographic location, ranging from 40% to 68%.3,4

Signs and Symptoms

Typically, patients with an SEA initially present with fever, malaise, and nonspecific symptoms and, as seen in this case, generally present several times to a physician before the correct diagnosis is made.1 Unfortunately, the classic triad of fever, spinal pain, and neurological deficits is only infrequently observed. Fever is present in approximately two-thirds of patients, and spinal pain is present approximately 90% of the time.2

There are four stages of disease progression associated with SEAs. A typical scenario involves the initial complaint of back pain (stage I); followed by pain in the distribution of an affected nerve root (stage II); then motor weakness, sensory changes, and bladder or bowel dysfunction (stage III); and, finally, paralysis (stage IV).1,2

Diagnosis

Laboratory studies typically are not helpful in making the diagnosis. A complete blood count may show leukocytosis, but values can also be within the normal reference range. Acute phase reactants like erythrocyte sedimentation rate and C-reactive protein are commonly elevated with an SEA, but are neither sensitive nor specific.1

To make the diagnosis, the best test is a gadolinium-enhanced MRI of the spine.2 It may be prudent to image the entire spine because multiple skip lesions are common, and a patient may not have pain or tenderness in all affected areas. If MRI is not available, a CT scan of the spine with IV contrast is an acceptable alternative.1

Once an SEA is identified, it is important to determine the organism(s) responsible for the infection. The best culture source is from the abscess itself (90%) followed by blood cultures (62%) and cerebrospinal fluid, which are positive only 19% of the time.1

Treatment

Once an SEA is diagnosed, a multidisciplinary approach involving hospitalists, interventional radiology, neurosurgery, and/or orthopedics is best. The most effective management is to treat patients with a combination of surgical decompression and drainage with systemic antibiotic therapy, typically for a minimum of 4 weeks. A minority of select patients may be treated with antibiotics alone.

References

1. Sexton, DJ, Sampson JH. Spinal epidural abscess. UpToDate Web site. http://www.uptodate.com/contents/spinal-epidural-abscess. Updated June 23, 2016. Accessed January 9, 2017.
2. Darouiche RO. Spinal epidural abscess. N Engl J Med. 2006;355(19):2012-2020. doi:10.1056/NEJMra055111.
3. Chen WC, Wang JL, Wang JT, Chen YC, Chang SC. Spinal epidural abscess due to Staphylococcus aureus: clinical manifestations and outcomes. J Microbiol Immunol Infect. 2008;41(3):215-221.
4. Krishnamohan P, Berger JR. Spinal epidural abscess. Curr Infect Dis Rep. 2014;16(11):436. doi:10.1007/s11908-014-0436-7.

Issue
Emergency Medicine - 49(2)
Issue
Emergency Medicine - 49(2)
Page Number
84-87
Page Number
84-87
Publications
Publications
Topics
Article Type
Display Headline
Malpractice Counsel: Pain in the Back, Pain in the Butt(ocks)
Display Headline
Malpractice Counsel: Pain in the Back, Pain in the Butt(ocks)
Sections
Disallow All Ads
Content Gating
No Gating (article Unlocked/Free)
Alternative CME
Article PDF Media

Malpractice Counsel: Acute Pulmonary Embolism Masquerading as Acute Coronary Syndrome

Article Type
Changed
Wed, 12/12/2018 - 20:52
Display Headline
Malpractice Counsel: Acute Pulmonary Embolism Masquerading as Acute Coronary Syndrome

Case

A 37-year-old woman presented to the ED with a 90-minute history of chest tightness and shortness of breath. She admitted to feeling anxious but denied nausea, vomiting, or diaphoresis. The patient was in good health overall and had no history of similar symptoms. The only medication she took on a regular basis was a combination oral contraceptive (OC). Regarding the patient’s social history, she admitted to smoking one-half of a pack of cigarettes per day and occasional alcohol use.

On physical examination, the patient’s vital signs were: heart rate (HR), 102 beats/min; blood pressure, 118/64 mm Hg; respiratory rate, 20 breaths/min; and temperature, 98.6˚F. Oxygen saturation was 95% on room air. The head, eyes, ears, nose, and throat examination was normal. The cardiopulmonary examination revealed slight tachycardia with a regular rhythm but no murmurs, rubs, or gallops; the lungs were clear to auscultation bilaterally. The abdominal examination revealed a soft, nontender abdomen, without mass, and no guarding or rebound was present. An examination of the lower extremities was not documented.

The emergency physician (EP) ordered laboratory studies, which included a complete blood count (CBC), basic metabolic profile (BMP), and troponin I level. A chest X-ray and electrocardiogram (ECG) were also ordered. The chest X-ray was interpreted as normal, and the ECG revealed mild sinus tachycardia with nonspecific ST-T segment changes in V1 through V3. The CBC and BMP were all within normal limits, but the troponin I level was slightly elevated.

Given the patient’s clinical presentation and slightly elevated troponin I level, the EP was concerned for an acute coronary syndrome (ACS) and admitted the patient to the care of the on-call cardiologist. Prior to transfer, the patient was given 325 mg of aspirin by mouth, but no anticoagulation therapy was ordered. The cardiologist, who evaluated the patient after she was admitted to the inpatient floor, was concerned the patient had a pulmonary embolism (PE), and ordered a stat computed tomography angiography (CTA) scan of the chest. While the patient was undergoing the chest CTA scan, she went into cardiac arrest. Despite aggressive resuscitative measures, the patient could not be revived and was pronounced dead. An autopsy revealed a PE as the cause of death.

Plaintiff’s Claim

The patient’s estate sued the EP for failure to properly diagnose the PE, stating the hospital was vicariously liable for the EP’s actions. The emergency medicine (EM) expert for the plaintiff opined that the decedent’s symptoms should have prompted the EP to suspect she was suffering from a PE, and he should have immediately ordered anticoagulation, a D-dimer test, or a chest CTA scan. The expert cardiologist for the plaintiff stated the EP should have immediately started the patient on anticoagulation prior to the chest CTA scan.

The Defense

The defense EM expert stated the defendant’s diagnosis of ACS was appropriate given the patient’s overall clinical presentation, and the defense expert cardiologist stated the standard of care did not require the EP to administer anticoagulation prior to her diagnosis of PE, since the bleeding risks outweighed the benefits.

Verdict

At trial, the jury returned a defense verdict.

Discussion

This is not the first (nor probably the last) malpractice case in this column to involve a missed PE. While there have been improvements to the tools we currently possess to evaluate patients for suspected PE, it remains a difficult condition to reliably and timely identify in the ED. Although the two predominating symptoms—shortness of breath and chest pain—are common presentations in the ED, each is associated with large differential diagnoses.

Acute Coronary Syndrome Versus Pulmonary Embolism

From what we know of the published details of this case, the patient had only one risk factor for ACS (cigarette smoking) and two risk factors for PE (cigarette smoking and estrogen-containing contraceptive use). The only abnormal physical finding (tachycardia) was slightly more suggestive of PE than ACS. This patient’s primary complaint was chest fullness and shortness of breath. According to the Prospective Investigation of Pulmonary Embolism Diagnosis II study, shortness of breath is the most common complaint in PE (73%), followed by pleuritic chest pain (44%).1

In ACS, which is more common in men versus women and in patients of both sexes over age 55 years, the clinical presentation most commonly involves chest pain that patients describe as a pressure or fullness (as demonstrated in this patient). Unfortunately, in certain patient populations (eg, women, elderly patients, patients with diabetes mellitus) the presenting complaint can be shortness of breath, weakness, or nausea and vomiting. In a study evaluating how frequently an acute PE can mimic ACS, Kukla et al2 found that one-third of patients with an acute PE can present with all of the manifestations suggestive of ACS (ie, chest pain, ECG changes, and elevated troponin).

It is probably safe to assume the elevated troponin I level played a factor in influencing the EP to diagnose ACS, rather than pursuing an alternative diagnosis such as PE. Unfortunately, since both serum troponin T and I can be markers of right ventricle dysfunction, they are elevated in 30% to 50% of patients with moderate-to-large PE.3 However, neither serum troponin T nor troponin I is specific for myocardial infarction or unstable angina.

 

 

Pretest Probability: Wells Criteria

Determining pretest probability for any disease process is important when evaluating complaints in the ED; this is especially true for PE. One of the most frequently used tools for determining the likelihood of PE in ED patients is the Wells criteria (Table 1).4

Table 1.
Pulmonary embolism is unlikely in patients with a Wells score of 4 points or less; PE is likely in patients with a score above 4.

Based on the published information available, the patient in this case would have scored a 1.5, placing her in the unlikely or low-risk category for PE. Patients whose Wells score places them in the low-risk group can benefit from serum D-dimer testing to help diagnose PE. However, serum D-dimer testing should not be ordered for patients in the likely or high-risk categories; these patients should instead be sent directly for imaging studies such as a chest CTA scan.

Pulmonary Embolism Rule-Out Criteria

For patients whose Wells criteria score places them in the “unlikely group,” the PE rule-out criteria (PERC) can be used to determine the need for ordering a D-dimer. If all eight criteria are met, no further testing is necessary to exclude PE from the differential diagnosis (Table 2).5

Table 2.
In one multicenter, prospective cohort study of 8,138 patients presenting to the ED with shortness of breath or chest pain, less than 1% of patients who were ruled out by the PERC rule developed a PE or deep vein thrombosis within the subsequent 45 days.5 If we apply the PERC rule to the patient in this case, she would have failed to “PERC-out” because her HR was greater than 100 beats/minute, and she was taking an estrogen-containing OC. If the EP had considered PE in the differential diagnosis, D-dimer testing would have been indicated, which might possibly have led to the correct diagnosis earlier.

Summary

Evaluating chest pain and shortness of breath in the ED is a humbling experience for even the most seasoned EP. Thoroughly reviewing the patient’s history and physical examination, and determining the pretest probability of disease entities high on the differential diagnoses list, go a long way in helping make the correct diagnosis—and in turn initiating possible life-saving interventions and treatment.

References

1. Stein PD, Beemath A, Matta F, et al. Clinical characteristics of patients with acute pulmonary embolism: data from PIOPED II. Am J Med. 2007;120(10):871-879.

2. Kukla P, Dlugopolski R, Krupa E, et al. How often pulmonary embolism mimics acute coronary syndrome? Kardiol Pol. 2011;69(3):235-240.

3. Meyer T, Binder L, Hruska N, Luthe H, Buchwald AB. Cardiac troponin I elevation in acute pulmonary embolism is associated with right ventricular dysfunction. J Am Coll Cardiol. 2000;36(5):1632-1636.

4. Wells PS, Anderson DR, Rodger M, et al. Derivation of a simple clinical model to categorize patients probability of pulmonary embolism: increasing the models utility with the SimpliRED D-dimer. Thromb Haemost. 2000;83(3):416-420.

5. Kline JA, Courtney DM, Kabrhel C, et al. Prospective multicenter evaluation of the pulmonary embolism rule-out criteria. J Thromb Haemost. 2008;6(5):772-780. doi: 10.1111/j.1538-7836.2008.02944.x.

Article PDF
Author and Disclosure Information

Author’s Disclosure Statement: The author reports no actual or potential conflict of interest in relation to this article.

Issue
Emergency Medicine - 48(10)
Publications
Topics
Page Number
440-442
Sections
Author and Disclosure Information

Author’s Disclosure Statement: The author reports no actual or potential conflict of interest in relation to this article.

Author and Disclosure Information

Author’s Disclosure Statement: The author reports no actual or potential conflict of interest in relation to this article.

Article PDF
Article PDF

Case

A 37-year-old woman presented to the ED with a 90-minute history of chest tightness and shortness of breath. She admitted to feeling anxious but denied nausea, vomiting, or diaphoresis. The patient was in good health overall and had no history of similar symptoms. The only medication she took on a regular basis was a combination oral contraceptive (OC). Regarding the patient’s social history, she admitted to smoking one-half of a pack of cigarettes per day and occasional alcohol use.

On physical examination, the patient’s vital signs were: heart rate (HR), 102 beats/min; blood pressure, 118/64 mm Hg; respiratory rate, 20 breaths/min; and temperature, 98.6˚F. Oxygen saturation was 95% on room air. The head, eyes, ears, nose, and throat examination was normal. The cardiopulmonary examination revealed slight tachycardia with a regular rhythm but no murmurs, rubs, or gallops; the lungs were clear to auscultation bilaterally. The abdominal examination revealed a soft, nontender abdomen, without mass, and no guarding or rebound was present. An examination of the lower extremities was not documented.

The emergency physician (EP) ordered laboratory studies, which included a complete blood count (CBC), basic metabolic profile (BMP), and troponin I level. A chest X-ray and electrocardiogram (ECG) were also ordered. The chest X-ray was interpreted as normal, and the ECG revealed mild sinus tachycardia with nonspecific ST-T segment changes in V1 through V3. The CBC and BMP were all within normal limits, but the troponin I level was slightly elevated.

Given the patient’s clinical presentation and slightly elevated troponin I level, the EP was concerned for an acute coronary syndrome (ACS) and admitted the patient to the care of the on-call cardiologist. Prior to transfer, the patient was given 325 mg of aspirin by mouth, but no anticoagulation therapy was ordered. The cardiologist, who evaluated the patient after she was admitted to the inpatient floor, was concerned the patient had a pulmonary embolism (PE), and ordered a stat computed tomography angiography (CTA) scan of the chest. While the patient was undergoing the chest CTA scan, she went into cardiac arrest. Despite aggressive resuscitative measures, the patient could not be revived and was pronounced dead. An autopsy revealed a PE as the cause of death.

Plaintiff’s Claim

The patient’s estate sued the EP for failure to properly diagnose the PE, stating the hospital was vicariously liable for the EP’s actions. The emergency medicine (EM) expert for the plaintiff opined that the decedent’s symptoms should have prompted the EP to suspect she was suffering from a PE, and he should have immediately ordered anticoagulation, a D-dimer test, or a chest CTA scan. The expert cardiologist for the plaintiff stated the EP should have immediately started the patient on anticoagulation prior to the chest CTA scan.

The Defense

The defense EM expert stated the defendant’s diagnosis of ACS was appropriate given the patient’s overall clinical presentation, and the defense expert cardiologist stated the standard of care did not require the EP to administer anticoagulation prior to her diagnosis of PE, since the bleeding risks outweighed the benefits.

Verdict

At trial, the jury returned a defense verdict.

Discussion

This is not the first (nor probably the last) malpractice case in this column to involve a missed PE. While there have been improvements to the tools we currently possess to evaluate patients for suspected PE, it remains a difficult condition to reliably and timely identify in the ED. Although the two predominating symptoms—shortness of breath and chest pain—are common presentations in the ED, each is associated with large differential diagnoses.

Acute Coronary Syndrome Versus Pulmonary Embolism

From what we know of the published details of this case, the patient had only one risk factor for ACS (cigarette smoking) and two risk factors for PE (cigarette smoking and estrogen-containing contraceptive use). The only abnormal physical finding (tachycardia) was slightly more suggestive of PE than ACS. This patient’s primary complaint was chest fullness and shortness of breath. According to the Prospective Investigation of Pulmonary Embolism Diagnosis II study, shortness of breath is the most common complaint in PE (73%), followed by pleuritic chest pain (44%).1

In ACS, which is more common in men versus women and in patients of both sexes over age 55 years, the clinical presentation most commonly involves chest pain that patients describe as a pressure or fullness (as demonstrated in this patient). Unfortunately, in certain patient populations (eg, women, elderly patients, patients with diabetes mellitus) the presenting complaint can be shortness of breath, weakness, or nausea and vomiting. In a study evaluating how frequently an acute PE can mimic ACS, Kukla et al2 found that one-third of patients with an acute PE can present with all of the manifestations suggestive of ACS (ie, chest pain, ECG changes, and elevated troponin).

It is probably safe to assume the elevated troponin I level played a factor in influencing the EP to diagnose ACS, rather than pursuing an alternative diagnosis such as PE. Unfortunately, since both serum troponin T and I can be markers of right ventricle dysfunction, they are elevated in 30% to 50% of patients with moderate-to-large PE.3 However, neither serum troponin T nor troponin I is specific for myocardial infarction or unstable angina.

 

 

Pretest Probability: Wells Criteria

Determining pretest probability for any disease process is important when evaluating complaints in the ED; this is especially true for PE. One of the most frequently used tools for determining the likelihood of PE in ED patients is the Wells criteria (Table 1).4

Table 1.
Pulmonary embolism is unlikely in patients with a Wells score of 4 points or less; PE is likely in patients with a score above 4.

Based on the published information available, the patient in this case would have scored a 1.5, placing her in the unlikely or low-risk category for PE. Patients whose Wells score places them in the low-risk group can benefit from serum D-dimer testing to help diagnose PE. However, serum D-dimer testing should not be ordered for patients in the likely or high-risk categories; these patients should instead be sent directly for imaging studies such as a chest CTA scan.

Pulmonary Embolism Rule-Out Criteria

For patients whose Wells criteria score places them in the “unlikely group,” the PE rule-out criteria (PERC) can be used to determine the need for ordering a D-dimer. If all eight criteria are met, no further testing is necessary to exclude PE from the differential diagnosis (Table 2).5

Table 2.
In one multicenter, prospective cohort study of 8,138 patients presenting to the ED with shortness of breath or chest pain, less than 1% of patients who were ruled out by the PERC rule developed a PE or deep vein thrombosis within the subsequent 45 days.5 If we apply the PERC rule to the patient in this case, she would have failed to “PERC-out” because her HR was greater than 100 beats/minute, and she was taking an estrogen-containing OC. If the EP had considered PE in the differential diagnosis, D-dimer testing would have been indicated, which might possibly have led to the correct diagnosis earlier.

Summary

Evaluating chest pain and shortness of breath in the ED is a humbling experience for even the most seasoned EP. Thoroughly reviewing the patient’s history and physical examination, and determining the pretest probability of disease entities high on the differential diagnoses list, go a long way in helping make the correct diagnosis—and in turn initiating possible life-saving interventions and treatment.

Case

A 37-year-old woman presented to the ED with a 90-minute history of chest tightness and shortness of breath. She admitted to feeling anxious but denied nausea, vomiting, or diaphoresis. The patient was in good health overall and had no history of similar symptoms. The only medication she took on a regular basis was a combination oral contraceptive (OC). Regarding the patient’s social history, she admitted to smoking one-half of a pack of cigarettes per day and occasional alcohol use.

On physical examination, the patient’s vital signs were: heart rate (HR), 102 beats/min; blood pressure, 118/64 mm Hg; respiratory rate, 20 breaths/min; and temperature, 98.6˚F. Oxygen saturation was 95% on room air. The head, eyes, ears, nose, and throat examination was normal. The cardiopulmonary examination revealed slight tachycardia with a regular rhythm but no murmurs, rubs, or gallops; the lungs were clear to auscultation bilaterally. The abdominal examination revealed a soft, nontender abdomen, without mass, and no guarding or rebound was present. An examination of the lower extremities was not documented.

The emergency physician (EP) ordered laboratory studies, which included a complete blood count (CBC), basic metabolic profile (BMP), and troponin I level. A chest X-ray and electrocardiogram (ECG) were also ordered. The chest X-ray was interpreted as normal, and the ECG revealed mild sinus tachycardia with nonspecific ST-T segment changes in V1 through V3. The CBC and BMP were all within normal limits, but the troponin I level was slightly elevated.

Given the patient’s clinical presentation and slightly elevated troponin I level, the EP was concerned for an acute coronary syndrome (ACS) and admitted the patient to the care of the on-call cardiologist. Prior to transfer, the patient was given 325 mg of aspirin by mouth, but no anticoagulation therapy was ordered. The cardiologist, who evaluated the patient after she was admitted to the inpatient floor, was concerned the patient had a pulmonary embolism (PE), and ordered a stat computed tomography angiography (CTA) scan of the chest. While the patient was undergoing the chest CTA scan, she went into cardiac arrest. Despite aggressive resuscitative measures, the patient could not be revived and was pronounced dead. An autopsy revealed a PE as the cause of death.

Plaintiff’s Claim

The patient’s estate sued the EP for failure to properly diagnose the PE, stating the hospital was vicariously liable for the EP’s actions. The emergency medicine (EM) expert for the plaintiff opined that the decedent’s symptoms should have prompted the EP to suspect she was suffering from a PE, and he should have immediately ordered anticoagulation, a D-dimer test, or a chest CTA scan. The expert cardiologist for the plaintiff stated the EP should have immediately started the patient on anticoagulation prior to the chest CTA scan.

The Defense

The defense EM expert stated the defendant’s diagnosis of ACS was appropriate given the patient’s overall clinical presentation, and the defense expert cardiologist stated the standard of care did not require the EP to administer anticoagulation prior to her diagnosis of PE, since the bleeding risks outweighed the benefits.

Verdict

At trial, the jury returned a defense verdict.

Discussion

This is not the first (nor probably the last) malpractice case in this column to involve a missed PE. While there have been improvements to the tools we currently possess to evaluate patients for suspected PE, it remains a difficult condition to reliably and timely identify in the ED. Although the two predominating symptoms—shortness of breath and chest pain—are common presentations in the ED, each is associated with large differential diagnoses.

Acute Coronary Syndrome Versus Pulmonary Embolism

From what we know of the published details of this case, the patient had only one risk factor for ACS (cigarette smoking) and two risk factors for PE (cigarette smoking and estrogen-containing contraceptive use). The only abnormal physical finding (tachycardia) was slightly more suggestive of PE than ACS. This patient’s primary complaint was chest fullness and shortness of breath. According to the Prospective Investigation of Pulmonary Embolism Diagnosis II study, shortness of breath is the most common complaint in PE (73%), followed by pleuritic chest pain (44%).1

In ACS, which is more common in men versus women and in patients of both sexes over age 55 years, the clinical presentation most commonly involves chest pain that patients describe as a pressure or fullness (as demonstrated in this patient). Unfortunately, in certain patient populations (eg, women, elderly patients, patients with diabetes mellitus) the presenting complaint can be shortness of breath, weakness, or nausea and vomiting. In a study evaluating how frequently an acute PE can mimic ACS, Kukla et al2 found that one-third of patients with an acute PE can present with all of the manifestations suggestive of ACS (ie, chest pain, ECG changes, and elevated troponin).

It is probably safe to assume the elevated troponin I level played a factor in influencing the EP to diagnose ACS, rather than pursuing an alternative diagnosis such as PE. Unfortunately, since both serum troponin T and I can be markers of right ventricle dysfunction, they are elevated in 30% to 50% of patients with moderate-to-large PE.3 However, neither serum troponin T nor troponin I is specific for myocardial infarction or unstable angina.

 

 

Pretest Probability: Wells Criteria

Determining pretest probability for any disease process is important when evaluating complaints in the ED; this is especially true for PE. One of the most frequently used tools for determining the likelihood of PE in ED patients is the Wells criteria (Table 1).4

Table 1.
Pulmonary embolism is unlikely in patients with a Wells score of 4 points or less; PE is likely in patients with a score above 4.

Based on the published information available, the patient in this case would have scored a 1.5, placing her in the unlikely or low-risk category for PE. Patients whose Wells score places them in the low-risk group can benefit from serum D-dimer testing to help diagnose PE. However, serum D-dimer testing should not be ordered for patients in the likely or high-risk categories; these patients should instead be sent directly for imaging studies such as a chest CTA scan.

Pulmonary Embolism Rule-Out Criteria

For patients whose Wells criteria score places them in the “unlikely group,” the PE rule-out criteria (PERC) can be used to determine the need for ordering a D-dimer. If all eight criteria are met, no further testing is necessary to exclude PE from the differential diagnosis (Table 2).5

Table 2.
In one multicenter, prospective cohort study of 8,138 patients presenting to the ED with shortness of breath or chest pain, less than 1% of patients who were ruled out by the PERC rule developed a PE or deep vein thrombosis within the subsequent 45 days.5 If we apply the PERC rule to the patient in this case, she would have failed to “PERC-out” because her HR was greater than 100 beats/minute, and she was taking an estrogen-containing OC. If the EP had considered PE in the differential diagnosis, D-dimer testing would have been indicated, which might possibly have led to the correct diagnosis earlier.

Summary

Evaluating chest pain and shortness of breath in the ED is a humbling experience for even the most seasoned EP. Thoroughly reviewing the patient’s history and physical examination, and determining the pretest probability of disease entities high on the differential diagnoses list, go a long way in helping make the correct diagnosis—and in turn initiating possible life-saving interventions and treatment.

References

1. Stein PD, Beemath A, Matta F, et al. Clinical characteristics of patients with acute pulmonary embolism: data from PIOPED II. Am J Med. 2007;120(10):871-879.

2. Kukla P, Dlugopolski R, Krupa E, et al. How often pulmonary embolism mimics acute coronary syndrome? Kardiol Pol. 2011;69(3):235-240.

3. Meyer T, Binder L, Hruska N, Luthe H, Buchwald AB. Cardiac troponin I elevation in acute pulmonary embolism is associated with right ventricular dysfunction. J Am Coll Cardiol. 2000;36(5):1632-1636.

4. Wells PS, Anderson DR, Rodger M, et al. Derivation of a simple clinical model to categorize patients probability of pulmonary embolism: increasing the models utility with the SimpliRED D-dimer. Thromb Haemost. 2000;83(3):416-420.

5. Kline JA, Courtney DM, Kabrhel C, et al. Prospective multicenter evaluation of the pulmonary embolism rule-out criteria. J Thromb Haemost. 2008;6(5):772-780. doi: 10.1111/j.1538-7836.2008.02944.x.

References

1. Stein PD, Beemath A, Matta F, et al. Clinical characteristics of patients with acute pulmonary embolism: data from PIOPED II. Am J Med. 2007;120(10):871-879.

2. Kukla P, Dlugopolski R, Krupa E, et al. How often pulmonary embolism mimics acute coronary syndrome? Kardiol Pol. 2011;69(3):235-240.

3. Meyer T, Binder L, Hruska N, Luthe H, Buchwald AB. Cardiac troponin I elevation in acute pulmonary embolism is associated with right ventricular dysfunction. J Am Coll Cardiol. 2000;36(5):1632-1636.

4. Wells PS, Anderson DR, Rodger M, et al. Derivation of a simple clinical model to categorize patients probability of pulmonary embolism: increasing the models utility with the SimpliRED D-dimer. Thromb Haemost. 2000;83(3):416-420.

5. Kline JA, Courtney DM, Kabrhel C, et al. Prospective multicenter evaluation of the pulmonary embolism rule-out criteria. J Thromb Haemost. 2008;6(5):772-780. doi: 10.1111/j.1538-7836.2008.02944.x.

Issue
Emergency Medicine - 48(10)
Issue
Emergency Medicine - 48(10)
Page Number
440-442
Page Number
440-442
Publications
Publications
Topics
Article Type
Display Headline
Malpractice Counsel: Acute Pulmonary Embolism Masquerading as Acute Coronary Syndrome
Display Headline
Malpractice Counsel: Acute Pulmonary Embolism Masquerading as Acute Coronary Syndrome
Sections
Disallow All Ads
Article PDF Media